You are on page 1of 67

IDMP TEST SERIES-2023

High Yielding PRELIMS TEST SERIES

Test-24
Science & Tech, Environment Revision
Topic Covered:
 Science & Tech Revision
 Environment revision

IDMP-2023
Contact us :info@onlyias.com

OnlyIAS Nothing Else Visit :dpp.onlyias.in


Contact : +91-7007 931 912

Q.1) Consider the following statements regarding Q.4) Consider the following statements regarding
Marsquake : Global Methane Initiative (GMI):
1. NASA’s Perseverance Mars lander has detected the 1. GMI is a voluntary Government and an informal
largest quake on Mars ever observed on another international partnership created to achieve global
planet. reduction in anthropogenic methane emission.
2. Mars have tectonic plates; its crust is a giant plate 2. Methane is the second-most abundant greenhouse
and quakes are caused by shifts in tectonic plates gas in the atmosphere, after carbon dioxide and having
Which of the statements given above is /are not 25-28 times more harmful effect than carbon dioxide.
correct? 3. India is not a member of this initiative.
a) 1 only Which of the statements given above is/are correct?
b) 2 only a) 1 and 2 only
c) Both 1 and 2 b) 2 and 3 only
d) Neither 1 nor 2 c) 1 and 3 only
d) All of the above
Q.2) Consider the following statements regarding the
key findings of the India State of Forest Report (ISFR) Q.5) Consider the following statements regarding the
2021: Mission Coking Coal (MCC):
1. It is an assessment of India’s forest and tree cover, 1. Domestic coking coal is low ash coal and is suitable
published every four years by the Forest Survey of for direct use in blast furnaces.
India. 2. Coking coal is used in manufacturing of steel through
2. The total forest cover in India’s biodiversity-rich blast furnace route.
north-eastern states has increased. 3. It will reduce imports and lead to AtmaNirbhar
3. Nagarjunasagar-Srisailam Tiger Reserve has the Bharat through increasing production and utilization of
Largest Forest Cover in Tiger reserves (TR). domestic coking coal.
Which of the statements given above is/are Incorrect? Which of the statements given above is/are correct?
a) 1 and 2 only a) 1 and 2 only
b) 2 and 3 only b) 2 and 3 only
c) 1 and 3 only c) 1 and 3 only
d) 2 only d) All of the above

Q.3) Consider the following statements about the Q.6) Consider the following statements regarding
Action Plan for Introduction of Cheetah in India: Climate Hazards and Vulnerability Atlas :
1. It will enhance genetic diversity and capacity to 1. The atlas provides a range of vulnerability with risks
sequester carbon. ranging from nil, low, moderate, high and very high
2. Cheetah is the only large carnivore that got categories for selected Indian districts.
completely wiped out from India in 1952. 2. It is launched by NITI Aayog with United Nations
3. Reintroduction will be sent from Zambia or Environment Programme (UNEP).
Botswana to the Bandhavgarh National Park because Which of the statements given above is/are correct?
of its suitable habitat and adequate prey base. a) 1 only
Which of the statements given above is/are correct? b) 2 only
a) 1 only c) Both 1 and 2
b) 2 and 3 only d) Neither 1 nor 2
c) 1 and 2 only
d) 2 only Q.7) Consider the following statements in context of
the Global Tiger Forum (GTF) :

DPP 2023 DAY 167 1


Contact us :info@onlyias.com

OnlyIAS Nothing Else Visit :dpp.onlyias.in


Contact : +91-7007 931 912

1. Global Tiger forum is the only intergovernmental Q.10) Consider the following statements regarding
platform of tiger range countries which has been Dugong (a species of sea cow) :
consolidating Tiger Action Plans of the range countries. 1. Dugong is a herbivorous mammal which is listed
2. St. Petersburg Declaration on Tiger Conservation Endangered in IUCN Red List.
(2010) aimed at tripling the wild tiger population by 2. Dugong populations in an area can be used as an
2030. indicator of general coastal marine ecosystem health.
3. The Global Tiger Forum (GTF) and World Wildlife 3. Andra Pradesh has declared India’s first dugong
Fund India are the two implementing partners of the conservation reserve in the Gulf of Mannar and the
National Tiger Conservation Authority for Conservation adjacent Palk Bay on the southeast coast of India.
Assured | Tiger Standards (CA|TS) assessment in India. Which of the statements given above is/are correct?
Which of the statements given above are Correct? a) 1 only
a) 1 and 2 only b) 2 only
b) 2 and 3 only c) 2 and 3 only
c) 1 and 3 only d) 1 and 3 only
d) All of the above
Q.11) Consider the following Wetland/National
Q.8) Consider the following statements regarding Park/Wildlife sanctuary with respect to their relevant
blending hydrogen into natural gas system: states:
1. The Oil and Natural Gas Corporation(ONGC) has 1. Nauradehi Wildlife Sanctuary Rajasthan
begun India’s maiden project of mixing hydrogen into 2. Mukundara Tiger Reserve Madhya Pradesh
the natural gas system. 3. Haiderpur Wetland Maharashtra
2. Blending integrates concentrations of hydrogen into 4. Khijadiya Bird Sanctuary Gujarat
existing natural gas pipelines and reduces carbon How many of the above pairs are matched correctly?
intensity of carbon dioxide. a) Only one pair
3. Hydrogen-enriched compressed natural gas (HCNG) b) Only two pairs
will ensure 70% more reduction in carbon monoxide c) Only three pairs
emissions compared to CNG. d) All four pairs
Which of the statements given above is/are Incorrect?
a) 2 only Q.12) Consider the following statements in context of
b) 3 only the Red Sanders (Red Sandalwood):
c) 1 and 2 only 1. It is endemic to a distinct tract of forests in Western
d) None of the above and Eastern Ghats of India.
2. It is used as a flavouring in alcoholic beverages and
Q.9) Consider the following statements regarding for making musical instruments.
Ken-Betwa river interlinking project : 3. It has been assessed as ‘Near Threatened’ as per the
1. It is the first project under the National Perspective IUCN criteria and scheduled in appendix I of CITES.
Plan (NPP) for the interlinking of rivers. Which of the statements given above is/are Incorrect?
2. The project envisages transferring water from the a) 1 only
Betwa river to the Ken river, both tributaries of the b) 2 only
Narmada. c) 1 and 3 only
Which of the statements given above is/are correct? d) 2 and 3 only
a) 1 only
b) 2 only Q.13) ‘BiodiverCities by 2030: Transforming cities’ a
c) Both 1 and 2 recently published report is released by?
d) Neither 1 nor 2 a) United Nations Environment Programme (UNEP)

DPP 2023 DAY 167 2


Contact us :info@onlyias.com

OnlyIAS Nothing Else Visit :dpp.onlyias.in


Contact : +91-7007 931 912

b) World Economic Forum (WEF) b) 2 only


c) Intergovernmental Panel on Climate Change (IPCC) c) Both 1 and 2
d) UN-Habitat d) Neither 1 nor 2

Q.14) Which of the following conventions/protocols Q.17) “It is an off-patent organochlorine pesticide
are related to Access to Genetic Resources and the developed in 1954. It was used widely on crops until
Fair and Equitable Sharing of Benefits? the Supreme Court banned its production and
1. Cartagena Protocol distribution. It became a highly controversial
2. Convention on Biological Diversity (CBD) agrichemical due to its acute toxicity, potential for
3. Convention on International Trade in Endangered bioaccumulation, and role as an endocrine disruptor.
Species of Wild Fauna and Flora(CITES) A global ban on the manufacture and use of this
4. Nagoya Protocol pesticide was negotiated under the Stockholm
Which of the options given above is/are correct? Convention in April 2011. It is still used extensively in
a) 1 and 3 only India and China despite laws against its use”. Which
b) 2 and 4 only of the following pesticides is described in the above
c) 1, 2 and 4 only lines?
d) All of the above a) Methyl parathion
b) Mancozeb
Q.15) Consider the following statements regarding c) Endosulfan
the Convention on International Trade in Endangered d) Sulphur
Species of Wild Fauna and Flora (CITES):
1. Appendix III lists species that are in danger of Q.18) Which one of the following best describes the
extinction and prohibits commercial trade of these term “Deep learning” ?
plants and animals except in extraordinary situations a) It is an interactive experience of a real-world
for scientific or educational reasons. environment where the objects that reside in the
2. It is administered by the United Nations Framework real world are enhanced by computer-generated
Convention on Climate Change(UNFCCC). perceptual information
3. CITES are legally binding on the Parties but it does b) It is the on-demand availability of computer system
not take the place of national laws. resources, especially data storage and computing
Which of the statements given above is/are Incorrect? power, without direct active management by the
a) 1 only user
b) 2 and 3 only c) It is computer software that is released under a
c) 1 and 3 only license in which the copyright holder grants users
d) 1 and 2 only the rights to use, change, and distribute the
software and source code
Q.16) Consider the following statements regarding d) It is a type of artificial intelligence (AI) that imitates
Protection of Plant Varieties and Farmers’ Rights the way humans gain certain types of knowledge
(PPV&FR) Act: and is an important element of data science, which
1. The Act introduced the world’s only IPR legislation includes statistics and predictive modelling.
which grants intellectual property rights to the farmers
but not to the plant breeders. Q.19) Recently, China rocket debris fell in the Indian
2. It was enacted under Trade-Related Aspects of Ocean near Maldives. In the view of Space debris
Intellectual Property Rights (TRIPS) of WTO. consider the following statements -
Which of the statements given above is/are Correct? 1. Space debris consists of meteoroid which orbits
a) 1 only the sun and artificial satellites that orbits the Earth

DPP 2023 DAY 167 3


Download From - https://upscmaterial.online/

Contact us :info@onlyias.com

OnlyIAS Nothing Else Visit :dpp.onlyias.in


Contact : +91-7007 931 912

2. Kessler syndrome is related to space junk. 2. It will be launched by a Polar Satellite Launch Vehicle
Which of the statements given above is/are correct? (PSLV) of ISRO in Low earth orbit(LEO).
a) 1 only 3. ISRO will send humanoid Vyommitra in unmanned
b) 2 only Gaganyaan spacecraft ahead of human spaceflight.
c) Both 1 and 2 Which of the statements given above is/are correct?
d) Neither 1 nor 2 a) 2 only
b) 2 and 3 only
Q.20) Consider the following statements regarding c) 1 and 3 only
Norovirus: d) All of the above
1. Norovirus is highly contagious and the most
common cause of Sinusitis in all age groups Q.23) Consider the following statements with
2. Norovirus is resistant to many disinfectants and reference to the Kessler Syndrome:
can heat up to 60°C and can also survive many 1. It is a scenario in which the density of objects in Low
common hand sanitisers. Earth Orbit (LEO) is high enough that collisions
3. Developing immunity to one type of Norovirus between objects could cause a cascade where each
provide protection against other varieties of collision generates space debris.
Norovirus 2. ISRO has come up with Project NETRA, an early
Which of the statements given above is/ are not warning system in space to detect debris.
correct? 3. The space debris is found only in low Earth orbit due
a) 3 only to artificial objects in the orbit.
b) 1 and 3 only Which of the statements given above is/are correct?

e
c) 2 and 3 only a) 1 only

n
d) 1 only b) 2 and 3 only

li
c) 1 and 2 only
Q.21) On 20 December 2013, at its 68th session, the d) All of the above

n
United Nations General Assembly (UNGA) proclaimed o
3 March as UN World Wildlife Day to celebrate and Q.24) Consider the following statements with
l.
raise awareness of the world’s wild animals and reference to the positron recently in news:
a

plants. The date chosen coincides with which of the 1. The behavior of positrons between 10 and 300 GeV
ri

following events ? is known as the positron excess.


a) The day of signature of the Convention on 2. Cosmic rays which are produced in supernovae
te

International Trade in Endangered Species of Wild explosions and propagating through the giant celestial
Fauna and Flora (CITES) clouds are a viable contributor to the origin of the
a

b) UN Conference on the Human Environment in positron excess phenomenon.


m

Stockholm in 1972 was held on this day 3. Positron is not the antiparticle of the electron but a
c

c) Commemoration of the adoption of the sub-particle of it.


Convention on Biological Diversity (CBD) in 1992. Which of the statements given above is/are correct?
s

d) The day on which International Union for a) 1 only


p

Conservation of Nature (IUCN) was established b) 1 and 2 only


.u

c) 2 and 3 only
Q.22) Consider the following statements regarding d) All of the above
w

the Gaganyaan Mission of India :


w

1. It is the first indigenous mission that will send Indian Q.25) Consider the following statements with
astronauts to space. reference to the Venus missions:
w

DPP 2023 DAY 167 4


https://upscmaterial.online/
Download From - https://upscmaterial.online/

Contact us :info@onlyias.com

OnlyIAS Nothing Else Visit :dpp.onlyias.in


Contact : +91-7007 931 912

1. EnVision is an European Space Agency led mission to Q.28) Consider the following statements with
planet Venus. reference to the Nano Sniffer recently in news:
2. DAVINCI (Deep Atmosphere Venus Investigation of 1. India has developed the world's first micro-sensor
Noble gasses, Chemistry, and Imaging) mission is based explosive trace detector (ETD) named as nano
announced by NASA. sniffer.
3. Venus spins on its axis from east to west and also 2. Nano Sniffers used a Micro-Electromechanical
does not have rings. System to detect explosives.
Which of the statements given above is/are correct? 3. It can also detect nanograms quantities of
a) 2 only explosives.
b) 1 and 3 only Which of the statements given above is/are correct?
c) 1 and 2 only a) 3 only
d) All of the above b) 2 and 3 only
c) 1 and 2 only
Q.26) Consider the following statements with d) All of the above
reference to the Project P-75 recently in news:
1. It is the Submarine construction programme to Q.29) Consider the following statements with
acquire national competence in submarine reference to the PYTHON-5 recently in news:
construction. 1. It is an Indian short-range ballistic missile with a
2. A hypersonic missile traveling at speeds of Mach 5 range of 150 km
and higher are developed under it. 2. It is a third-generation anti-tank guided missile with
3. Under this project advanced versions of agni missiles a speed of Mach 4 and an operational range of more

e
are developed. than 20 km.

n
Which of the statements given above is/are incorrect? 3. It is an air-to-air and surface-to-air missile developed

li
a) 3 only by Israel.
b) 2 and 3 only Which of the statements given above is/are correct?

n
c) 1 and 2 only a) 1 only o
d) 1 and 3 only b) 3 only
l.
c) 2 and 3 only
a

Q.27) Consider the following statements with d) All of the above


ri

reference to the different types of engines used in


rockets: Q.30) Consider the following statements with
te

1. The Ramjet engine uses Hydrogen as fuel and the references to the new development related to the
Oxygen from the atmospheric air as the defence field:
a

Oxidiser. 1. Project Seabird is the largest naval infrastructure


m

2. The Scramjet is a air-breathing jet engine that uses project for India and also involves creation of a naval
c

the vehicle’s forward motion to compress incoming air base.


for combustion without a rotating compressor. 2. Smart anti-airfield weapon (SAAW) is indigenously
s

3. A hypersonic cruise missile is boosted by a rocket to designed smart weapon, with 10 km range and can
p

hypersonic speed and then uses an air-breathing target enemy airfield assets such as radars, bunkers,
.u

engine called a scramjet to sustain that speed taxiways and runways.


Which of the statements given above is/are correct? 3. P-8I is a long-range, multi-mission maritime patrol
w

a) 1 only aircraft based on the Boeing Next-Generation 737-800


w

b) 3 only aircraft.
c) 1 and 2 only Which of the statements given above are correct?
w

d) 2 and 3 only a) 1 and 3 only

DPP 2023 DAY 167 5


https://upscmaterial.online/
Download From - https://upscmaterial.online/

Contact us :info@onlyias.com

OnlyIAS Nothing Else Visit :dpp.onlyias.in


Contact : +91-7007 931 912

b) 2 and 3 only c) 1 and 2 only


c) 1 and 2 only d) All of the above
d) All of the above
Q.34) Consider the following statements with
Q.31) Consider the following statements with reference to the Chimeric Antigen Receptor T-
reference to the blockchain technology: cells(CAR-T) therapy recently in the news.
1. It is a digital ledger of transactions that is duplicated 1. It is a way to get immune cells called T cells to fight
and distributed across the entire network of computer cancer by editing them in the lab so they can find and
systems. destroy cancer cells.
2. Copy of each transaction along with its hash is stored 2. Large numbers of the CAR-T cells are given to the
in the ledger shared across all participants of the patient by the infusion process.
network. 3. T cells are always taken from a healthy person not
3. Blockchain can be used in Permission less models but from a cancer patient.
not in Permissioned models. Which of the statements given above is/are correct?
Which of the statements given above are correct? a) 1 only
a) 1 and 2 only b) 2 and 3 only
b) 2 and 3 only c) 1 and 2 only
c) 1 and 3 only d) All of the above
d) All of the above
Q.35) Consider the following statements with
Q.32) Consider the following statements with reference to the Genetically modified food crops:

e
references to the term ‘’DearCry’’ recently in news: 1. Bt cotton is the only GM crop that has been

n
1. It is an instant messaging platform launched by approved for commercial cultivation by the

li
National Informatics Center (NIC). Government of India.
2. It is a new family of ransomware attacking business 2. FSSAI issued an order setting the permissible limit for

n
email servers. genetically modified organisms (GMO) in imported
o
3. It provides a secure method of communication that food crops at 10%.
l.
would not be vulnerable to attack from any amount of 3. Globally GM crops were commercially introduced in
a

computational power. 1996 but in India commercial cultivation of GM crop


ri

Which of the statements given above is/are correct? was approved in 2002 by the Government of India.
a) 3 only Which of the statements given above is/are correct?
te

b) 1 and 3 only a) 1 and 3 only


c) 2 only b) 2 and 3 only
a

d) 2 and 3 only c) 1 and 2 only


m

d) All of the above


c

Q.33) Consider the following statements with


reference to the Li Fi technology recently in news: Q.36) Consider the following statements with
s

1. Li-Fi uses high frequency radio waves for data references to the Indi Gau recently in news:
p

transmission. 1. It is India’s first Cattle Genomic Chip for the


.u

2. LiFi is strictly based on a unique line-of-sight conservation of pure varieties of indigenous cattle
technology. breeds.
w

3. Its network coverage distance is about 100 meters . 2. It is the largest cattle chip in the world with 11,496
w

Which of the statements given above is/are incorrect? markers.


a) 3 only 3. This indigenous chip was developed by the National
w

b) 2 and 3 only Institute of Animal Biotechnology.

DPP 2023 DAY 167 6


https://upscmaterial.online/
Download From - https://upscmaterial.online/

Contact us :info@onlyias.com

OnlyIAS Nothing Else Visit :dpp.onlyias.in


Contact : +91-7007 931 912

Which of the statements given above is/are correct? 3. Vehicles running on HFCs are also suited for heavy
a) 1 only duty vehicles as electric vehicles are only limited to
b) 2 and 3 only lighter vehicles.
c) 1 and 2 only Which of the statements given above is/are correct?
d) All of the above a) 3 only
b) 2 and 3 only
Q.37) Consider the following statements with c) 1 and 2 only
references to the Ribonucleic acid RNA: d) 1 and 3 only
1. RNA interference (RNAi) technology can be used for
manufacturing pesticides. Q.40) Consider the following statements with
2. RNA is less resistant to damage from UV light than reference to the Aluminium air batteries:
DNA 1. This battery uses aluminium alloy plates as cathode
3. RNA nucleotides contain the nitrogenous bases and air electrode as anode.
adenine, cytosine, and guanine but they do not contain 2. They can be recharged many times like lithium-ion
thymine, which is instead replaced by uracil. batteries.
Which of the statements given above is/are correct? 3. They are costlier than Lithium-ion batteries.
a) 1 only Which of the statements given above is/are incorrect?
b) 1 and 3 only a) 1 only
c) 2 and 3 only b) 2 only
d) All of the above c) 1 and 2 only
d) 1, 2 and 3

e
Q.38) Consider the following statements with

n
references to the International solar alliance: Q.41) Consider the following statements with

li
1. The ISA, is an Indian initiative that was launched by references to the term Tokmak recently in news:
the Prime Minister of India and the Prime Minister of 1. The tokamak is an experimental machine designed

n
Britain in India. to harness the energy of fusion. o
2. ISA is the nodal agency for implementing One Sun 2. The device uses electric fields to contain and control
l.
One World One Grid initiative. the hot plasma, which enables the fusion between
a

3. The United States was founding members of ISA but deuterium and tritium nuclei to produce great
ri

later the US left this alliance. amounts of energy.


Which of the statements given above is/are incorrect? Which of the statements given above is/are correct?
te

a) 1 only a) 1 only
b) 1 and 3 only b) 2 only
a

c) 1 and 2 only c) Both 1 and 2


m

d) 2 and 3 only d) Neither 1 nor 2


c

Q.39) Consider the following statements with Q.42) Consider the following statements regarding
s

references to the hydrogen fuel cell (HFCs): Antimicrobial Resistance (AMR):


p

1. A hydrogen fuel cell is an electrochemical device that 1. AMR surveillance and research network (AMRSN)
.u

converts chemical energy of a hydrogen into comprises 30 tertiary care government hospitals
mechanical energy. only.
w

2. Only water is the remaining part of HFCs which can 2. It is mainly caused due to poor hygiene and
w

be used in the water deficient areas. overuse of antimicrobials.


w

DPP 2023 DAY 167 7


https://upscmaterial.online/
Download From - https://upscmaterial.online/

Contact us :info@onlyias.com

OnlyIAS Nothing Else Visit :dpp.onlyias.in


Contact : +91-7007 931 912

3. A separate Schedule H-1 has been incorporated in b) 1, 2 and 4 only


Drug and Cosmetic rules to regulate the sale of c) 2, 3 and 4 only
antimicrobials in the country d) All of the above
Which of the statements given above is/are correct?
a) 1 only Q.46) Seen in news, “Presidio Principles”, are related
b) 2 and 3 only to, which of the following:
c) 1 and 2 only a) Decentralized future with blockchain technology
d) 2 only b) Free and open navigation through sea routes
c) Food subsidies to the developing nations under
Q.43) Which of the statements given below about WTO
Agni Prime are incorrect? d) Climate funding and mitigation
1. It is a canisterised missile and will replace Agni-1
and Agni-2 missiles. Q.47) With reference to Metaverse, consider the
2. The Agni-P and Agni-5 ballistic missiles trace their following statements:
origins back to the Integrated Guided Missile 1. It includes exclusively virtual reality and
Development Programme (IGMDP). augmented reality.
Select the correct answer using the code given below: 2. It is a virtual world with real-time events and an
a) 1 only online infrastructure.
b) 2 only 3. It will provide virtual communities, activities,
c) Both 1 and 2 events and all seamless access without the need to
d) Neither 1 nor 2 sign into multiple apps.

e
Which of the statements given above is/are correct?

n
Q.44) Consider the following statements regarding a) 2 only

li
Pralay Missile: b) 1 and 3 only
1. It is a surface-to-surface missile bought from c) 2 and 3 only

n
France. d) All of the above o
2. It is a derivative of the Prahaar missile programme,
l.
which was tested for the first time in 2011. Q.48) Consider the following statements regarding
a

Which of the statements given above is/are correct? Radioisotope Thermoelectric Generator (RTG):
ri

a) 1 only 1. It is a type of Nuclear-based power system that is


b) 2 only generally used for power generation and thermal
te

c) Both 1 and 2 management of space missions.


d) Neither 1 nor 2 2. It converts electrical energy into thermal energy
a

through the device known as thermocouples.


m

Q.45) With reference to Coal-Based Hydrogen 3. RTG can prove to be a good alternative to solar
c

Production, consider the following statements: energy in space.


1. Coal is one of the important sources of hydrogen. Which of the statements given above is/are correct?
s

2. Almost 100% of hydrogen produced in India is a) 1 and 2 only


p

through natural gas. b) 2 only


.u

3. Cost of hydrogen produced from coal can be c) 2 and 3 only


costlier and more sensitive to imports. d) 1 and 3 only
w

4. The currently produced hydrogen is consumed


w

highly by oil refineries. Q.49) ‘Photodynamic Therapy’ and ‘Hormone


Which of the statements given above are correct? therapy’ are related to which of the following?
w

a) 2 and 3 only a) Tuberculosis Treatment

DPP 2023 DAY 167 8


https://upscmaterial.online/
Download From - https://upscmaterial.online/

Contact us :info@onlyias.com

OnlyIAS Nothing Else Visit :dpp.onlyias.in


Contact : +91-7007 931 912

b) Cancer Treatment 2. It causes symptoms similar to smallpox but are


c) AIDS Treatment more severe.
d) Genetically modified Crops 3. It is a zoonotic disease and can be sexually
transmitted.
Q.50) Consider the following statements regarding Which of the statements given above are correct?
Intellectual Property Rights (IPR): a) 1 and 2 only
1. India is a member of the World Trade Organisation b) 1 and 3 only
and committed to the Agreement on Trade Related c) 2 and 3 only
Aspects of Intellectual Property (TRIPS d) 1, 2 and 3
Agreement).
2. Department of Industrial Policy & Promotion Q.53) The Ministry of Health and Family Welfare has
(DIPP), Ministry of Corporate Affairs, has been approved the National Rare Disease Policy 2021. In
appointed as the nodal department to coordinate, the view of this, Consider the following statements
guide and oversee the implementation and future about Rare disease :
development of IPRs in India. 1. WHO defines rare disease as often debilitating
3. India’s rank in the Global Innovation Index (GII) lifelong disease or disorder with a prevalence of 1
issued by World Intellectual Property Organization or less, per 10000 population.
(WIPO) in 2021 is 46th. 2. Rashtriya Arogya Nidhi provides financial
Which of the statements given above is/are correct? assistance of 20 Lakhs for people suffering from
a) 1 and 2 only rare diseases
b) 2 only Which of the statements given above is/are Correct?

e
c) 1 and 3 only a) 1 only

n
d) All of the above b) 2 only

li
c) Both 1 and 2
Q.51) Consider the following statements with d) Neither 1 nor 2

n
regarding to Vaccines: o
1. Nucleic Acid uses genetic material from a disease- Q.54) Recently, Many countries including India
l.
causing virus or bacterium to stimulate an immune demanded for Patent waiver of Covid vaccines. In the
a

response against it. context of this, consider the following statements


ri

2. mRNA are highly efficacious because of their regarding Evergreening and Compulsory licencing of
inherent capability of being translatable into the Patents -
te

protein structure inside the cell cytoplasm. 1. The government can give licence for the use of
3. ZyCov-D is the world's first DNA vaccine against patented invention to a third party without the
a

COVID-19. consent of the patent holder under Compulsory


m

Which of the statements given above are correct? licensing


c

a) 1 and 2 only 2. Compulsory licencing is not allowed under any


b) 1 and 3 only circumstances as per the policies of Trips
s

c) 2 and 3 only agreement of WTO


p

d) 1, 2 and 3 3. Evergreening of patents deal with renewable of


.u

patents with minor changes


Q.52) Consider the following statements with Which of the statements given above is/are Correct?
w

reference to Monkeypox: a) 1 and 2 only


w

1. It is a member of the Orthopoxvirus genus in the b) 1 and 3 only


c) 2 and 3 only
family Poxviridae.
w

d) All of the above

DPP 2023 DAY 167 9


https://upscmaterial.online/
Download From - https://upscmaterial.online/

Contact us :info@onlyias.com

OnlyIAS Nothing Else Visit :dpp.onlyias.in


Contact : +91-7007 931 912

Q.55) Consider the following statements in the 1. Soyameals will be distributed through Anganwadi
context of World’s first Genetically Modified (GM) institutions to increase Protein intake among
Rubber Sapling: - children
1. The genetically modified rubber has been planted
2. GM soyabean contains living modified organisms
for the first time in Kerala.
2. Natural rubber is a native of Amazon forests and is and has no negative impact on human health as
naturally suited for the colder conditions of well as on the environment.
northeast India. Which of the statements given above is/are Correct?
Which of the statements given above is/are Correct? a) 1 only
a) 1 only b) 2 only
b) 2 only c) Both 1 and 2
c) Both 1 and 2
d) Neither 1 nor 2
d) Neither 1 nor 2

Q.56) Consider the following statement about Q.59) Recently, the National Strategy on Additive
Synthetic Biology: Manufacturing has been released . In the view of
1. It refers to the science of using genetic sequencing, consider the following statements -
editing, and modification to create unnatural 1. Additive manufacturing is a process of joining
organisms or organic molecules that can function materials to build objects from 3D model data,
in living and non-living systems.
usually layer after layer.
2. It can be used for treatment of malaria and Car
Tcell therapy for cancer treatment. 2. The National Strategy on Additive Manufacturing
3. The Department of Science and Technology has was released by Niti Aayog with an aim to increase
released a National policy for Synthetic Biology. India’s share in global additive manufacturing.

e
Which of the statements given above is/are Incorrect? Which of the statements given above is/are Correct?
a) 1 only

n
a) 1 only
b) 1 and 3 only

li
b) 2 only
c) 2 and 3 only
c) Both 1 and 2

n
d) 1, 2 and 3
d) Neither 1 nor 2 o
Q.57) WHO released the third progress report on
l.
Global trans-fat elimination 2021 titled ‘Countdown Q.60) The Union Government has constituted an
a

to 2023’. In the context of this consider the following expert panel to probe the recent series of battery
statements -
ri

explosions in electric vehicles (EVs). In the view of


1. Trans Fats are saturated fatty acids that belong to
this, select the correct statements why Li-ion
te

both natural and industrial sources


2. FASSI (Food Safety and Standards Authority of batteries are most preferred:
1. They have high power-to-weight ratio
a

India) has launched ‘REPLACE ‘strategy to eliminate


industrially-produced artificial trans fats from the 2. They have high self-discharge.
m

global food supply 3. Its low reactivity causes thermal runaway


Which of the statements given above is/are Correct?
c

4. They don’t have any memory effect.


a) 1 only
Which of the statements given above is/are Correct?
s

b) 2 only
a) 1, 2 and 3 only
p

c) Both 1 and 2
d) Neither 1 nor 2 b) 1 and 4 only
.u

c) 2 and 3 only
d) 3 and 4 only
w

Q.58) Recently the government decided to import


genetically modified Soyameal for the first time.
w

Consider the following statements regarding this - Q.61) In 2022, the Ministry of Power (MoP)
announced a Green Hydrogen Policy (GHP). In the
w

DPP 2023 DAY 167 10


https://upscmaterial.online/
Download From - https://upscmaterial.online/

Contact us :info@onlyias.com

OnlyIAS Nothing Else Visit :dpp.onlyias.in


Contact : +91-7007 931 912

context of this, consider the following statements Q.64) Consider the following statements regarding
about Hydrogen: Neutrino:
1. Hydrogen is colourless, non-toxic and has a salty 1. Neutrinos are tiny, neutral, elementary particles
taste and pungent odour. which interact with matter via magnetic and
2. Hydrogen gas generated through hydrolysis is electric forces.
called grey hydrogen. 2. Neutrinos are almost opaque to the matter.
3. India’s first 99.999% pure Green Hydrogen pilot Which of the statements given above is/are Correct?
plant has been commissioned by the Oil India a) 1 only
Limited (OIL) at its Jorhat Pump Station in Assam. b) 2 only
Which of the statements given above is/are Incorrect? c) Both 1 and 2
a) 1, 2 and 3 d) Neither 1 nor 2
b) 1 and 3 only
c) 2 and 3 only Q.65) Consider the following statements about the
d) 1 and 2 only Liquid Nano Urea:
1. It has been developed by International Centre for
Q.62) Recently, Scientists in the UK have managed to Nanobiotechnology (ICN), Kanyakumari with nano
produce the largest amount of energy so far from a nitrogen particles.
nuclear fusion reaction. Select the correct statement 2. It is sprayed directly on the root of plants and
about in the context of Nuclear Fusion hence improves agricultural produce with reduced
1. Fusion processes generate long-lived radioactive urea consumption
waste while releasing energy. 3. It has higher efficiency but lower shelf life as

e
2. Fusion process does not produce greenhouse compared to conventional Urea because of being

n
gases. liquid in nature.

li
3. Fusion processes can sustain for long periods of Which of the statements given above is/are incorrect?
time due to formation of large nuclei. a) 3 only

n
Which of the statements given above is/are Incorrect? b) 1 only
a) 1 and 2 only c) 2 and 3 only
o
l.
d) 1, 2 and 3
b) 2 only
a

c) 1 and 3 only
Q.66) Consider the following statements in the light of
ri

d) All of the above


Nano technology:
te

1. Conversion of a material at nanoscale results in


Q.63) In the context of India’s Nuclear Programme
alternation of Physicochemical and Biochemical
consider the following statements -
a

properties but the Optical properties remain the


1. Indian Nuclear Power Generation envisages a
m

same.
Three Stage Programme.
2. Nano alloys are alloys with one metal at core and
2. India's 1962 Atomic Energy Act says nothing about
c

another stays on the surface as a shell


liability or compensation in the event of an
s

Which of the statements given above is/are Correct?


accident
p

a) 1 only
3. Fast breeder reactors use heavy water as
.u

b) 2 only
moderator.
c) Both 1 and 2
Which of the statements given above is/are Correct?
w

a) 1 and 2 only d) Neither 1 nor 2


b) 1 only
w

c) 1 and 3 only Q.67) The Ministry of Civil Aviation (MoCA) has


released the updated Drone Rules, 2021. In the
w

d) All of the above


context of this, consider the following statements -
DPP 2023 DAY 167 11
https://upscmaterial.online/
Download From - https://upscmaterial.online/

Contact us :info@onlyias.com

OnlyIAS Nothing Else Visit :dpp.onlyias.in


Contact : +91-7007 931 912

1. Import of drones and drone components shall be at the following statements regarding Tiger and its
regulated by the Directorate General of Civil conservation:
Aviation 1. ‘Project Tiger’ is a Centrally sponsored scheme
which provides central assistance to tiger range
2. Drones have been classified according to the
States for in-situ and ex- situ conservation of tigers.
weight of drone and its payload 2. Under St. Petersburg Declaration, The tiger
3. Drones’ rules will not be applied to the naval, inhabits countries pledged to double number of
military or air force. tigers by 2022
Which of the statements given above is/are Correct? 3. Conservation Assured Tiger Standards (CATS)
a) 2 only recommends for the translocation of Tigers from
b) 1 and 3 only one tiger reserve to other tiger reserve.
c) 2 and 3 only Which of the following statements given above is/are
d) 1, 2 and 3 Correct?
a) 2 only
Q.68) Recently, “PARAM Ganga” has been established b) 1 and 2 only
at IIT Roorkee under National Supercomputing c) 2 and 3 only
Mission (NSM). In the context of this consider the d) 1 and 3 only
following statements-
Q.71) Guidelines for Groundwater Regulation -2020
1. ‘FLOPS’ are used to measure the memory capacity have been issued for sustainable use of Groundwater.
of the Super Computer. In the context of this consider following statements
2. National Supercomputing Mission (NSM) has been for Central Ground Water Board (CGWB) -
steered jointly by Ministry of Electronics & 1. Central Ground Water Board (CGWB) is a statutory
Information Technology and NITI Aayog. body constituted under the Environmental

e
Which of the statements given above is/are Correct? (Protection) Act ,1986.
2. It has been entrusted with the responsibility of

n
a) 1 only
regulating and controlling ground water

li
b) 2 only development and management in the country

n
c) Both 1 and 2 Which of the statements given above is/are Incorrect?
d) Neither 1 nor 2 a) 1 only o
b) 2 only
l.
Q.69) Recently, the First Internet Governance Forum c) Both 1 and 2
a

was held in India. In the context of this consider the d) Neither 1 nor 2
ri

following statements -
te

1. Internet Governance Forum (IGF) is a UN-based Q.72) Consider the following statements regarding
intergovernmental platform to discuss public Bose-Einstein Condensate (BEC):
a

policy for deliverance of better governance 1. It exists when atoms of certain elements are cooled
m

through the Internet. to temperatures near absolute zero.


2. India Internet Governance Forum (IIGF) has been 2. It has been created at the Cold Atom Laboratory, a
c

constituted in conformance to the Tunis Agenda of device on board the International Space Station
s

UN Internet Governance. (ISS).


p

Which of the statements given above is/are Correct? 3. It is used to study quantum mechanics on a
a) 1 only
.u

macroscopic level.
b) 2 only Which of the statements given above is/are correct?
w

c) Both 1 and 2 a) 3 only


d) Neither 1 nor 2 b) 2 and 3 only
w

c) 1 and 2 only
Q.70) ‘Project Tiger’ will celebrate the Golden jubilee d) 1, 2 and 3
w

of its establishment in 2023. In the view of this, look

DPP 2023 DAY 167 12


https://upscmaterial.online/
Download From - https://upscmaterial.online/

Contact us :info@onlyias.com

OnlyIAS Nothing Else Visit :dpp.onlyias.in


Contact : +91-7007 931 912

Q.73) Consider the following statements regarding b) 3 only


the Hasdeo Aranya Forest: c) 1 and 3 only
1. They are called the lungs of Odisha. d) 1, 2 and 3
2. They provide a large migratory corridor for
elephants. Q.77) Consider the following statements regarding
3. In 2010, it was categorized as a “no-go” zone for India Meteorological Department (IMD):
mining by the Government of India. 1. The administrative responsibilities of the
Which of the statements given above is/are correct? Department are under the supervision of the
a) 2 only Ministry of Environment, Forest and Climate
b) 1 and 3 only Change of the Indian Government.
c) 2 and 3 only 2. It uses the Long Period Average (LPA) as a base to
d) All of the above predict the expected amount of Monsoon rainfall
in a particular year.
Q.74) Recently seen in news, “Peninsular Rock Which of the statements given above is/are correct?
Agama” is a: a) 1 only
a) Bird b) 2 only
b) Sedimentary Rock c) Both 1 and 2
c) Metamorphic Rock d) Neither 1 nor 2
d) Lizard
Q.78) With reference to Arctic Amplification, consider
Q.75) With reference to Loktak Lake, consider the the following statements:
1. It occurs when the atmosphere’s net radiation

e
following statements:
balance is affected by an increase in greenhouse

n
1. It is the largest freshwater lake in Meghalaya. gasses.

li
2. It houses the only floating national park in the 2. The main factor contributing to it is the low albedo.
world. 3. As per the ‘State of Global Climate’ Report by the

n
3. Instead of Carbon Dioxide, Nitrogen is the major United Nations Framework Convention on Climate
o
pollutant in the lake. Change, sea level along the Indian coast is rising
l.
Which of the statements given above are correct? faster than the global average rate.
Which of the statements given above are correct?
a

a) 2 only
a) 1 and 2 only
ri

b) 2 and 3 only b) 2 and 3 only


c) 1 and 3 only
te

c) 1 and 3 only
d) All of the above d) All of the above
a

Q.76) With reference to the Godavari river, consider Q.79) Recently seen in the news, Oil and Natural Gas
m

the following statements: Corporation (ONGC) for the first time drilled to
generate electricity using geothermal power at this
c

1. Its basin extends to Maharashtra, Andhra Pradesh,


place. The place is which of the following:
Chhattisgarh, Odisha, Madhya Pradesh, Karnataka
s

a) Uttarakhand
and Puducherry. b) Ladakh
p

2. The Polavaram Project on the river Godavari river, c) Jammu and Kashmir
.u

is the reason behind the dispute between d) Himachal Pradesh


Telangana and Andhra Pradesh.
w

Q.80) Which of the following lakes of Iran are


3. Pushkaram Mela is held on the banks of river
experiencing shrinking/drying up due to
w

Godavari. anthropological reasons?


Which of the statements given above is/are correct? 1. Lake Baikal
w

a) 2 only 2. Lake Sawa

DPP 2023 DAY 167 13


https://upscmaterial.online/
Download From - https://upscmaterial.online/

Contact us :info@onlyias.com

OnlyIAS Nothing Else Visit :dpp.onlyias.in


Contact : +91-7007 931 912

3. Lake Urmia c) 1 and 3 only


4. Lake Superior d) All of the above
Select the correct answer using the codes given
below: Q.84) Consider the following pairs:
a) 2 and 3 only (Volcanoes) (Country)
b) 1 and 4 only 1. Kavachi : Solomon Islands
c) 1, 2 and 3 only 2. Tofua : Italy
d) All of the above 3. Etna : Tonga
4. Semeru : Sri Lanka
Q.81) Consider the following pairs: Which of the pairs given above is/are correctly
(Mountain) (River) matched?
1. Amarkantak : Narmada a) Only one pair
2. Satpura : Chambal b) Only two pairs
3. Shivalik : Yamuna c) Only three pairs
4. Vindhya : Mahanadi d) All four pairs
Which of the pairs given above is/are incorrectly
matched? Q.85) Recently the Aridity Anomaly Outlook Index
a) 4 only was released, which of the following statements are
b) 2 and 4 only correct about it?
c) 1 and 3 only 1. It was released by the World Meteorological
d) None of the above Organization.
2. It mainly monitors agricultural drought.
Q.82) Consider the following statements regarding 3. Bihar had the second highest number of districts
Vanilla Islands: experiencing arid conditions in India.

e
1. It is formed by the islands of Seychelles, Select the correct answer using the codes given
Madagascar, Reunion (France), Mauritius, below:

n
Comoros, and Mayotte (France) in the Pacific a) 2 and 3 only

li
Ocean. b) 1 and 3 only

n
2. For the first time the Vice-President of India visited c) 1 and 2 only
Comoros. d) 1, 2 and 3 o
3. In the colonial era, Vanilla islands were the object
l.
of rivalry among the European powers. Q.86) Consider the following statements regarding
a

Which of the statements given above is/are correct? Rat Hole Mining:
a) 2 only 1. The government does not have any Act in place to
ri

b) 1 and 3 only regulate mining.


te

c) 2 and 3 only 2. It is a major source of air, water and soil pollution


d) All of the above and National Green Tribunal (NGT) banned it in
a

2014.
Q.83) With reference to Electromagnetic Field (EMF) 3. It is prevalent in Meghalaya due to the thinness of
m

Emissions, consider the following statements: coal found over here.


1. EMF are present everywhere in our environment Which of the statements given above are correct?
c

but are invisible to the human eye. a) 1 and 2 only


s

2. As per WHO, the emissions from mobile towers in b) 2 and 3 only


p

India are ten times more stringent than the safe c) 1 and 3 only
limits prescribed by the International Commission d) All of the above
.u

on Non-Ionizing Radiation Protection (ICNIRP) .


3. It is resulting in various serious health problems in Q.87) Consider the following statements regarding
w

humans like leukemia, depression, miscarriages, Abysmal State of Heritage Conservation:


w

etc. 1. The Abysmal State of Heritage Conservation was


Which of the statements given above is/are correct? revealed under a report by the Archaeological
w

a) 1 only Survey of India.


b) 2 and 3 only
DPP 2023 DAY 167 14
https://upscmaterial.online/
Download From - https://upscmaterial.online/

Contact us :info@onlyias.com

OnlyIAS Nothing Else Visit :dpp.onlyias.in


Contact : +91-7007 931 912

2. As per the report, there is no national policy on d) West Bengal


archaeological exploration and excavation.
3. The National Culture Fund, established to rope in Q.92) Consider the following pairs with respect to
corporate groups and individuals to fund Harvest Festival of India
conservation efforts, has utilized only 14% of its (Festival) (State)
funds because of the lack of coordination with the 1. Nuakhai : West Bengal
ASI. 2. Pongal : Tamil Nadu
Which of the statements given above is/are correct? 3. Nabanna : Odisha
a) 1 only 4. Onam : Kerala
b) 2 and 3 only Which of the pairs given above is/are correctly
c) 1 and 2 only matched?
d) 1, 2 and 3 a) One pair only
b) Two pairs only
Q.88) What was the theme of “World Tribal Day” c) Three pairs only
2022? d) All four pairs
a) The Role of Indigenous Women in the Preservation
and Transmission of Traditional Knowledge. Q.93) With reference to Mandala Art, consider the
b) Leaving no one behind: Indigenous peoples and the following statements:
call for a new social contract. 1. It can be formed in circular and square shape and
c) Preserving Indigenous Languages pattern is interconnected.
d) Global Efforts for Social Inclusion 2. It first appeared in Rig Veda, in Hinduism.
3. The art gained worldwide attention when it was
Q.89) Consider the following statements regarding made with leaves and rocks by artist James Brunt
Wancho Community: in the United States.

e
1. They are tribal people inhabiting Meghalaya. Which of the statements given above are correct?
2. The prime festival of the Wancho is Oriah, held for a) 1 and 2 only

n
a period of six to twelve days interspersed with b) 2 and 3 only

li
prayer, songs and dances. c) 1 and 3 only

n
3. They are traditionally governed by a council of d) All of the above
elderly chieftains, known as Wangham or Wangsa. o
Which of the statements given above are correct? Q.94) Which of the following countries border the
l.
a) 1 and 2 only Gulf of Aqaba?
a

b) 2 and 3 only
1. Saudi Arabia
c) 1 and 3 only
ri

d) All of the above 2. Sudan


te

3. Israel
Q.90) With reference to the Navroz Festival, consider 4. Yeman
a

the following statements: Select the correct answer using the codes given
1. It is celebrated at the time of the vernal equinox. below:
m

2. It is celebrated as per Shahenshahi calendar, which a) 1 and 3 only


was created by Persian king Jamshed.
c

b) 2 and 4 only
Which of the statements given above is/are correct?
s

a) 1 only c) 1, 3 and 4 only


p

b) 2 only d) All of the above


c) Both 1 and 2
.u

d) Neither 1 nor 2 Q.95) Consider the following pairs:


(Geographical Indications Tag) (Region)
w

Q.91) Seen in news, ‘Temple of Vedic Planetarium’, is 1. Kandhamal Haladi : Odisha


2. Mithila Makhan : Gujarat
w

in which state?
a) Uttar Pradesh 3. Gulbarga Tur Dal : Karnataka
w

b) Bihar 4. Rasagola : Odisha


c) Karnataka Which of the pairs given above are correctly matched?
DPP 2023 DAY 167 15
https://upscmaterial.online/
Download From - https://upscmaterial.online/

Contact us :info@onlyias.com

OnlyIAS Nothing Else Visit :dpp.onlyias.in


Contact : +91-7007 931 912

a) 2 and 3 only a) 1 only


b) 1, 3 and 4 only b) 2 only
c) 1, 2 and 4 only c) Both 1 and 2
d) 2, 3 and 4 only
d) Neither 1 nor 2

Q.96) Consider the following statements in context of


Q.99) Recently seen is news ‘Operation Skylight’ is
Electoral Bonds:
associated with:
1. An Electoral Bond is like a promissory note that may
a) Biennial multinational large force employment
be purchased by a person who is a citizen of India or
exercise.
company incorporated or established in India.
b) Curbing of artificial lights for clear sky astronomical
2. Only the Political Parties registered under Section
research.
29A of the Representation of the People Act (RPA),
c) Testing the operational readiness of satellite-based
1951 and has secured at least five per cent of the votes
systems.
polled in the most recent General elections or
d) Reducing air pollution and limiting greenhouse gas
Assembly elections are eligible to receive electoral
emission.
bonds.
3. The bonds are like banknotes that are payable to the
Q.100) Which of the following best describes the term
bearer on demand and carry interest.
“Cantillon Effect”?
Which of the statements given above is/are Incorrect?
a) Changes in money supply in an economy causes
a) 2 only
redistribution of purchasing power among people.
b) 2 and 3 only
b) refers to a graphical representation that shows the
c) 1 and 3 only

e
relationship between the unemployment rate (on
d) 1 and 2 only

n
the horizontal axis) and the job vacancy rate (on

li
the vertical axis) in an economy.
Q.97) What is/are the impacts of counterfeit currency
c) shows a low-income, non-luxury product for which

n
on the economy?
demand increases as the price increases and vice
o
1. An artificial increase in money supply
versa.
l.
2. Decrease in the acceptability of paper money
d) shows the relationship between government
3. A decrease in the price of the goods
a

expenditure and economic growth.


4. leads to currency devaluation
ri

Select the correct answer using the codes given


te

below:
a) 1 and 3 only
a

b) 2 and 4 only
m

c) 1, 2 and 4 only
d) All of the above
c
s

Q.98) Consider the following statements regarding


p

Rudram-1:
.u

1. It is the first indigenous air-to-surface anti-radiation


missile of the country.
w

2. It is designed to detect, track and neutralise the


adversary effects of radiation spread due to a chemical
w

attack.
w

Which of the statements given above is/are Incorrect?

DPP 2023 DAY 167 16


https://upscmaterial.online/
Download From - https://upscmaterial.online/

Contact us :info@onlyias.com

OnlyIAS Nothing Else Visit :dpp.onlyias.in


Contact : +91-7007 931 912

Q.1) Ans: C scientists are discovering on Mars, due to less


Exp: seismic activity.As marsquakes aren't typically
● Statement 1 is not correct : NASA’s InSight as violent as earthquakes, they're more
Mars lander has detected the largest quake difficult to detect, and other vibrations – from
on Mars ever observed on another the wind, for example can interfere with
planet.NASA has reported that on May 4, its readings.
InSight Mars lander detected the largest quake
ever observed on another planet. The rover Reference:
first landed on Mars in November 2018, and https://indianexpress.com/article/explained/nasa-
has since heard 1,313 quakes.InSight is part of mars-mission-insight-rover-marsquake-7910915/
NASA’s Discovery Program.This mission is part
of NASA's Discovery Program for highly
focused science missions that ask critical
questions in solar system science. It will be the Q.2) Ans: A
first mission to peer deep beneath the Martian Exp:
surface, studying the planet’s interior by ● Statement 1 is incorrect: The Ministry of
measuring its heat output and listening for Environment, Forests and Climate Change
marsquakes, which are seismic events similar (MoEFCC) recently released the India State of
to earthquakes on Earth. It will use the seismic Forest Report (ISFR) 2021.It is an assessment of
waves generated by marsquakes to develop a India’s forest and tree cover, published every
map of the planet’s deep interior. With InSight, two years by the Forest Survey of India under

e
scientists hope to compare Earth and Mars, the Ministry of Environment, Forests and

n
and better understand how a planet’s starting Climate Change. The first survey was published

li
materials make it more or less likely to support in 1987, and ISFR 2021 is the 17th report. With
life. data computed through wall-to-wall mapping of

n
● Statement 2 is not correct : On Earth, quakes India’s forest cover through remote sensing
o
are caused by shifts in tectonic plates. Mars, techniques, the ISFR is used in planning and
l.
however, does not have tectonic plates, and formulation of policies in forest management as
a

its crust is a giant plate. Therefore, well as forestry and agroforestry sectors.
ri

‘marsquakes’ are caused due to stresses that ● Statement 2 is incorrect: As par findings of ISFR
cause rock fractures or faults in its crust.The report,the total forest cover in India’s
te

findings of Mars’ formation will help better biodiversity-rich north-eastern states has
understand how other rocky planets, including dropped owing to natural calamities, particularly
a

Earth, were and are created. InSight would landslides and heavy rains, in the region as well
m

delve deep beneath the surface of Mars, as to anthropogenic activities such as shifting
c

detecting the fingerprints of the processes of agriculture, pressure of developmental activities


terrestrial planet formation, as well as and felling of trees.
s

measuring the planet’s “vital signs”: Its “pulse” ➢ Top 5 states in Total Forest cover:
p

(seismology), “temperature” (heat flow Madhya Pradesh, Arunachal Pradesh,


.u

probe), and “reflexes” (precision tracking. This Chhattisgarh, Odisha, Maharashtra.


Marsquake is estimated to have hit magnitude ➢ Top 5 states by percent of State’s
w

5 on the scale used on Earth. A magnitude-5 Geographic Area under Forest Cover:
w

quake on Earth would be classed as moderate, Mizoram (84.53%), Arunachal Pradesh


only causing minor damage. However, it's right (79.33%), Meghalaya, Manipur,
w

at the upper end of the size of quakes that Nagaland.

DPP 2023 DAY 167 17


https://upscmaterial.online/
Download From - https://upscmaterial.online/

Contact us :info@onlyias.com

OnlyIAS Nothing Else Visit :dpp.onlyias.in


Contact : +91-7007 931 912

● Statement 3 is correct: Forest Cover assessment conservation areas and thereby contribute
in Tiger reserves & Tiger corridor areas of the towards the global climate change mitigation
country and Mapping of Climate Change goals.
Hotspots in Indian Forests are the new Chapters ● Statement 2 is correct: Recently, The
introduced in ISFR, 2021. Nagarjunasagar- Government of India has decided to reintroduce
Srisailam Tiger Reserve,Andhra Pradesh has the Cheetahs in National Parks over five years, under
Largest Forest Cover in Tiger reserves (TR). 'Action Plan for Introduction of Cheetah in India’.
Forest cover in the TR is 55,666.27 sq km (7.80% Cheetah is the only large carnivore that got
of the country's total forest cover and 74.51% of completely wiped out from India in 1952,
the total area of TRs). Forest cover in the Tiger mainly due to over-hunting and habitat loss.The
corridors is 11,575.12 sq km (1.62 % of the locally extinct cheetah-subspecies of India is
country's total forest cover).As per the Climate found in Iran and is categorized as critically
Hotspot projections for the studied periods i.e. endangered. It is not possible to source the
2030, 2050 and 2085- States/UTs projected to critically endangered Asiatic cheetah from Iran
witness highest temperature increase: Ladakh, without affecting this sub-species.
Jammu & Kashmir, Himachal Pradesh and ● Statement 3 is incorrect: India will source
Uttarakhand. cheetahs from Namibia or South Africa, which
can provide India with substantial numbers of
Reference: https://fsi.nic.in/forest-report-2021- suitable cheetah for several years. Amongst the
details 10 surveyed sites of the central Indian states,
https://indianexpress.com/article/explained/takeaw Kuno Palpur National Park (KNP) in Madhya

e
ays-from-india-state-of-forest-report-7722163/ Pradesh has been rated the highest. This is

n
because of its suitable habitat and adequate

li
prey base. KNP is 748 sq. km. in area, devoid of
Q.3) Ans: C human settlements, forms part of Sheopur-

n
Exp: Shivpuri deciduous open forest landscape and is
o
● Statement 1 is correct: Cheetah estimated to have a capacity to sustain 21
l.
reintroduction/conservation translocation is an cheetahs. Kuno also offers the prospect of
a

appropriate conservation strategy to restore the housing four big cats of India - tiger, lion, leopard
ri

integrity of ecosystems.A top-down effect of a and cheetah - and allowing them to coexist as in
large predator that enhances and maintains the the past.
te

genetic diversity in lower trophic levels of the


ecosystems. Reintroducing cheetahs in India will Reference:
a

help relieve pressure on the species by creating https://pib.gov.in/PressReleaseIframePage.aspx?PRI


m

additional habitat, which the cheetah D=1788373


c

desperately needs to survive. Having cheetahs in https://www.thehindu.com/sci-tech/energy-and-


India will boost ecotourism and enhance environment/50-cheetahs-to-be-introduced-in-india-
s

livelihood options and living conditions among in-next-5-years-environment-


p

the local communities where the cheetah is minister/article38142455.ece


.u

introduced. Among large carnivores, conflict


with human interests is lowest for cheetahs, as Q.4) Ans: A
w

they are not a threat to humans and usually do Exp:


w

not attack large livestock. It will help to enhance ● Statement 1 is correct: Global Methane
India’s capacity to sequester carbon through Initiative (GMI) is a voluntary Government and
w

ecosystem restoration activities in cheetah an informal international partnership created

DPP 2023 DAY 167 18


https://upscmaterial.online/
Download From - https://upscmaterial.online/

Contact us :info@onlyias.com

OnlyIAS Nothing Else Visit :dpp.onlyias.in


Contact : +91-7007 931 912

to achieve global reduction in anthropogenic the value of coking coal imported in FY 2020-21
methane emission through partnership among was Rs.45435 crore.
developed and developing countries having ● Statement 2 is correct: Coking refers to the
economies in transition.It has members from coal's ability to be converted into coke, a pure
45 countries including the United States and form of carbon that can be used in basic oxygen
Canada. furnaces. Coking coal is used in manufacturing
● Statement 2 is correct:Emission of methane is of steel through blast furnace route. Mission
a big concern as it is a greenhouse gas having coking coal (MCC) is based on the
25-28 times more harmful effect than carbon recommendations of the InterMinisterial
dioxide. Approximately 40% of methane Committee set up to strategize augmentation of
emitted is from natural sources and about 60% coking coal production in India.
comes from human-influenced sources, ● Statement 3 is correct: Objective of Mission
including livestock farming, rice agriculture, coking coal(MCC):
biomass burning and so forth.It is 80 times ➢ To prepare an action plan to increase
more potent than carbon dioxide in terms of its production and utilisation of domestic
global warming capacity. It is also the second- coking coal.
most abundant greenhouse gas in the ➢ Adoption of new technologies.
atmosphere, after carbon dioxide. ➢ Allocation of coking coal blocks for
● Statement 3 is incorrect: The forum was private sector development.
created in 2004 and India is one of the
➢ Setting up of new coking coal
members since its inception and has taken up
washeries, enhanced R&D activities

e
Vice-Chairmanship for the first time in the
and improvement of quality

n
Steering Leadership along with USA. The parameters.

li
Chairperson of the Steering Leadership is from It helps to boost production and strengthen
Canada. Recently India has co-chaired along

n
in-house capabilities which thereby will
with Canada the Global Methane Initiative o
reduce imports and lead to AtmaNirbhar
leadership meet held virtually.
Bharat.
l.
a

Reference:
Reference: https://coal.nic.in/en/major-
ri

https://pib.gov.in/PressReleaseIframePage.aspx?PRI statistics/mission-coking-coal
D=1780151
te

https://www.livemint.com/industry/energy/mission
https://www.indiaspend.com/explainers/chasing-
-coking-coal-spells-out-roadmap-for-boosting-
methane-why-curbing-methane-emissions-is-crucial-
a

production-11638865815603.html
to-fighting-climate-change-828127
m
c

Q.6) Ans: D
Q.5) Ans: B
s

Exp:
Exp:
p

● Statement 1 is incorrect: Climate Hazards and


● Statement 1 is incorrect: Domestic coking coal
.u

Vulnerability Atlas is based on several extreme


is high ash coal (between 18% - 49%) and is not weather events (extreme rainfall, drought,
suitable for direct use in blast furnaces.
w

coldwave, heatwave, thunderstorm, cyclones,


Therefore, it is blended with imported coking
lightning etc) and the risks they pose to the local
w

coal (<9% ash). About 50 MT coking coal is population, livelihoods and economy of each
imported by the country on an annual basis and district. The atlas provides a range of
w

vulnerability with risks ranging from nil, low,


DPP 2023 DAY 167 19
https://upscmaterial.online/
Download From - https://upscmaterial.online/

Contact us :info@onlyias.com

OnlyIAS Nothing Else Visit :dpp.onlyias.in


Contact : +91-7007 931 912

moderate, high and very high categories for network of habitats of the tiger and facilitate
every Indian district.The hazards and their inter passages in the range countries.
vulnerability values have been calculated ● Statement 2 is incorrect: The 13 Tiger Range
utilising the Met department’s historical climate Countries(TRC) consist of Malaysia, Bangladesh,
data. Each weather phenomenon’s month-wise Bhutan, Cambodia, China, India, Indonesia, Laos,
hazard levels posed to the respective district has Myanmar, Nepal, Russia, Thailand, and Vietnam.
been enlisted in this one-of-its-kind atlas. In all, India will facilitate Tiger Range Countries
640 maps are available in the online atlas. towards finalisation of New Delhi declaration for
● Statement 2 is incorrect: Recently, the Indian the Global Tiger Summit to be held at
Meteorological Department (IMD),an agency of Vladivostok, Russia later this year. According to
the Ministry of Earth Sciences, launched India’s the St. Petersburg Declaration on Tiger
1st Climate Hazards and Vulnerability Atlas.It is Conservation (2010), TRC agreed to the Global
developed by the scientists at the Climate Tiger Recovery Programme (GTRP) which
Research and Services (CRS) office of the India envisages doubling the wild tiger population by
Meteorological Department (IMD), Pune. The 2022.
Atlas will help in understanding region specific ● Statement 3 is correct: Conservation Assured
impact of certain extreme weather events, also Tiger Standards CA|TS is a set of criteria which
help disaster management sectors to identify allows tiger sites to check if their management
the vulnerable districts for taking preventive and will lead to successful tiger conservation. 14
adaptive measures. Tiger Reserves in India have already been
awarded with international CA|TS

e
Reference: accreditation. The Global Tiger Forum (GTF) and

n
https://www.thehindu.com/news/cities/chennai/im World Wildlife Fund India are the two

li
d-launches-climate-hazards-e-book-and- implementing partners of the National Tiger
vulnerability-e-atlas/article65252683.ece Conservation Authority for CATS assessment in

n
https://indianexpress.com/article/cities/pune/imd- India. o
launches-indias-1st-weather-hazard-and-
l.
vulnerability-atlas-7723913/ Reference:
a

https://pib.gov.in/PressReleseDetailm.aspx?PRID=17
ri

91473
Q.7) Ans: C https://tigers.panda.org/?5804966/Forth-Asia-Tiger-
te

Exp: Ministerial-Conference-concludes
● Statement 1 is correct: Global Tiger forum is the
a

only intergovernmental platform of tiger range


m

countries which has been consolidating Tiger Q.8) Ans: C


c

Action Plans of the range countries.The forum Exp:


was formed in 1993 on recommendations of an ● Statement 1 is incorrect: In line with the
s

international symposium on Tiger Conservation. National Hydrogen Mission, Gas Authority of


p

Global Tiger forum is headquartered in New India Limited (GAIL) has commenced India's
.u

Delhi, India.The Global Tiger Forum was set up first-of-its-kind project of mixing hydrogen into
to promote a worldwide campaign to save the the natural gas system to establish the techno-
w

tiger, its prey and its habitat. The Global Tiger commercial feasibility of blending hydrogen in
w

Forum has plans to promote a legal framework City Gas Distribution (CGD) network. CGD refers
in the countries involved for biodiversity to transportation of natural gas to enable supply
w

conservation and to increase the protected area of clean cooking fuel (like PNG) to households,

DPP 2023 DAY 167 20


https://upscmaterial.online/
Download From - https://upscmaterial.online/

Contact us :info@onlyias.com

OnlyIAS Nothing Else Visit :dpp.onlyias.in


Contact : +91-7007 931 912

industrial and commercial units as well as Exp:


transportation fuel (like Compressed Natural ● Statement 1 is correct: The governments of
Gas) to vehicles through a network of pipelines. Uttar Pradesh and Madhya Pradesh signed a
Government is planning to blend 15% green memorandum of agreement with the Union Jal
hydrogen with piped natural gas (PNG) for Shakti Ministry to implement the Ken-Betwa
domestic, commercial and industrial river interlinking project. The Ken-Betwa Link
consumption. Project is the first project under the National
● Statement 2 is incorrect: Blending integrates Perspective Plan (NPP) for the interlinking of
concentrations of hydrogen into existing rivers.The project needs approval on the share
natural gas pipelines and reduces carbon of Centre and State in funding, forming a new
intensity of methane. Hydrogen is a clean fuel organisation the Ken Betwa Link Project
and produces only water as by- Authority to execute the project and obtaining a
product.Depending on the source, hydrogen is stage 2 forest clearance for constructing the
classified as brown (derived from coal Daudhan dam. The project is expected to be
gasification), blue (produced from natural gas ready in 8 years. An SPV called Ken-Betwa Link
with a process of steam methane reforming), Project Authority (KBLPA) will be set up to
green (produced from electrolysis) and grey implement the project.
(produced from natural gas). GAIL has started ● Statement 2 is incorrect: The project involves
injecting grey hydrogen. This grey hydrogen the transfer of surplus water from the Ken river
would subsequently be replaced by green in Panna district in Madhya Pradesh to the Betwa
hydrogen. river in Uttar Pradesh. Both these rivers are

e
● Statement 3 is correct: Hydrogen-enriched tributaries of river Yamuna. This will be done

n
compressed natural gas (HCNG) will ensure 70% through the construction of the Dhaudan Dam

li
more reduction in carbon monoxide emissions and a 221 km-long canal. The project will be of
compared to CNG. It is easier and safer to use immense benefit to the water-starved region of

n
than hydrogen as it contains very low energy Bundelkhand. There is a hydropower component
o
content from hydrogen i.e., up to 30% by to this project as well which will generate just
l.
volume. Power output of HCNG is also better over 100 MW of electricity.
a

than CNG ones.India has committed to achieving


ri

net-zero carbon emissions by 2070 and Reference:


hydrogen together with renewable energy is https://economictimes.indiatimes.com/news/india/
te

seen as key to achieving that goal. centre-constitutes-ken-betwa-link-project-


authority/articleshow/89580333.cms?from=mdr
a

Reference: https://theprint.in/india/govt-forms-ken-betwa-link-
m

https://economictimes.indiatimes.com/industry/ren project-authority-to-implement-first-river-
c

ewables/gail-starts-indias-maiden-project-of- interlinking-project/832978/
blending-hydrogen-into-natural-gas-system-at-
s

indore/articleshow/89249620.cms?from=mdr
p

https://pib.gov.in/Pressreleaseshare.aspx?PRID=179 Q.10) Ans: B


.u

4428#:~:text=In%20line%20with%20the%20National, Exp:
based%20and%20carbon%20neutral%20future. ● Statement 1 is incorrect: Dugong (Dugong
w

dugon) also called ‘Sea Cow’ is one of the four


w

surviving species in the Order Sirenia and it is the


only existing species of herbivorous mammal
w

Q.9) Ans: A that lives exclusively in the sea including in

DPP 2023 DAY 167 21


https://upscmaterial.online/
Download From - https://upscmaterial.online/

Contact us :info@onlyias.com

OnlyIAS Nothing Else Visit :dpp.onlyias.in


Contact : +91-7007 931 912

India.It is listed vulnerable in IUCN Red List and them understand the benefits of conserving
comes under Schedule I of wildlife protection dugongs and their habitat.
act, 1972. Dugongs are long-living animals that
have a low reproductive rate, long generation Reference:
time, and high investment in each offspring.They https://www.iucn.org/resources/publication/dugon
are found in over 30 countries and in India are g-status-reports-and-action-plans-countries-and-
seen in the Gulf of Mannar, Gulf of Kutch, Palk territories
Bay, and the Andaman and Nicobar Islands. https://www.downtoearth.org.in/news/wildlife-
● Statement 2 is correct: Dugongs are an biodiversity/better-late-than-never-experts-on-
important part of the marine ecosystem and tamil-nadu-s-decision-to-go-ahead-with-dugong-
their depletion will have effects all the way up reserve-in-palk-bay-81712
the food chain.They can tolerate the brackish
waters found in coastal wetlands, and large
numbers are also found in wide and shallow Q.11) Ans: A
mangrove channels and around leeward sides of Exp:
large inshore islands, where seagrass beds are ● Nauradehi Wildlife Sanctuary is arguably one
common. Dugong play an important ecological of the largest wildlife reserves in India in terms
role in coastal marine ecosystems, and dugong of landmass. Established in the year 1975, it
populations in an area can be used as an spreads across a total expanse of 1197 square
indicator of general ecosystem health.The Gulf kilometres in the heart of Madhya
of Mannar is presently facing threats of climate Pradesh.The sites recommended for holding

e
change apart from destructive fishing practices and conservation breeding of cheetah in

n
and industrial pollution. India, in controlled wild conditions are:

li
● Statement 3 is incorrect: Recently, Tamil Nadu Nauradehi Wildlife Sanctuary Madhya
has declared India’s first dugong conservation Pradesh,Gandhi Sagar Wildlife Sanctuary -

n
reserve in the Gulf of Mannar and the adjacent Bhainsrorgarh Wildlife Sanctuary complex ,
o
Palk Bay on the southeast coast of India.The Madhya Pradesh, Shahgarh bulge in Jaisalmer,
l.
dugong is a species of sea cow found throughout Rajasthan and Mukundara Tiger
a

the warm latitudes of the Indian and western Reserve,Rajasthan.


ri

Pacific Oceans.The rich marine diversity of the ● The Mukundra Tiger Reserve (TR) is located
Gulf of Mannar would completely come under near Kota, Rajasthan.The park is situated in a
te

the protection in the conservation reserve. The valley formed by two parallel mountains viz.
Gulf of Mannar and Palk Bay are habitats for a Mukundra and Gargola.The 4 rivers (Ramzan,
a

wide range of marine fauna, including rare Ahu, Kali and Chambal) form the boundary of
m

fishes, sea turtles, seahorses, and sea the valley.It is located on the eastern bank of
c

cucumbers. A National Level Task Force for the chambal river and is drained by its
Dugong Conservation has been established and tributaries.Mukundra Tiger Reserve is the third
s

a National Conservation Action Plan for Dugongs tiger reserve of Rajasthan is also known as
p

and their habitats in India has been drafted. Darrah wildlife sanctuary.
.u

Dugong and Seagrass Conservation Project ● Haiderpur Wetland in Uttar Pradesh has been
focuses on the dugong range states of Indonesia, added as the 47th Ramsar Site in December
w

Madagascar, Malaysia, Mozambique, Solomon 2021. It is a human-made wetland formed in


w

Islands, Sri Lanka, Timor-Leste and Vanuatu. It 1984 and is located within the boundaries of
seeks to work with local communities to help Hastinapur Wildlife Sanctuary.
w

DPP 2023 DAY 167 22


https://upscmaterial.online/
Download From - https://upscmaterial.online/

Contact us :info@onlyias.com

OnlyIAS Nothing Else Visit :dpp.onlyias.in


Contact : +91-7007 931 912

● Khijadiya Bird Sanctuary in Gujarat and treating digestive tract problems, fluid retention,
Bakhira Wildlife Sanctuary in Uttar Pradesh cough and for blood purification. It can be used
were announced as Ramsar sites (Wetlands of for removal of tan, blemishes, dullness and acne.
International Importance) on the occasion of In Ayurveda, it is used to treat eye disorders. It
World Wetlands Day 2022 (2nd February 2022) can be used as an antipyretic, anti-inflammatory,
held at Sultanpur National Park, a Ramsar site anthelmintic agent. It is also used in cosmetics. It
in Haryana. Khijadiya Wildlife Sanctuary is used as a flavouring in alcoholic beverages. In
merges with the famous Marine National Park addition, the timber is also exploited for the
on one end, and the Dhunvav river drains fresh extraction of santalin (a red pigment used as dye
water on the other end. It holds a remarkable and colourant in food).
habitat with salt pans and freshwater on either ● Statement 3 is incorrect: In 2022, Red Sanders
side. Furthermore, a large creek flows on the (Red Sandalwood) has fallen back into the
northern end streams from the Gulf of Kutch, ‘Endangered’ category in the International
supporting marine diversity and mangroves. Union for Conservation of Nature’s (IUCN) Red
List. It was earlier classified as ‘Near Threatened’
Reference:
in 2018 and has again been classified as
https://indianexpress.com/article/cities/rajkot/inter
Endangered. The IUCN has assessed that over
national-wetlands-day-sanctuaries-gujarat-up-
the last three generations, the species has
ramsar-7752917/
experienced a population decline of 50-80 per
https://www.hindustantimes.com/india-news/3-
cent. It has been assessed as ‘Endangered’ as
sites-in-mp-selected-for-hosting-cheetah-officials-to-
per the IUCN criteria and scheduled in appendix
train-in-africa-101611051513469.html

e
II of CITES and Wildlife Protection Act.

n
li
Reference:
Q.12) Ans: C
https://www.downtoearth.org.in/news/wildlife-

n
Exp:
biodiversity/red-sanders-falls-back-in-iucn-s-
o
● Statement 1 is incorrect: Red Sanders (Red
endangered-category-81053
l.
Sandalwood) is an Indian endemic tree species,
https://timesofindia.indiatimes.com/city/hyderabad
with a restricted geographical range in the
a

/red-sanders-back-on-iucn-red-list-of-endangered-
Eastern Ghats of India. It is endemic to a distinct
ri

species/articleshow/88864267.cms
tract of forests in Andhra Pradesh.Red Sanders
te

smuggling continues despite the physical


deterrents and patrols in place in Andhra
Q.13) Ans: B
a

Pradesh, as well as international, national and


Exp:
m

state-level laws preventing the cutting and


● The World Economic Forum (WEF) recently
transport of species.
c

published a report titled ‘BiodiverCities by


● Statement 2 is correct: The timber of Red
2030: Transforming cities’ relationship with
s

Sanders is highly demanded domestically and


nature’ developed as an output of the
p

internationally, and especially in East Asian


BiodiverCities by 2030 initiative. BiodiverCities
.u

countries like China and Japan. Red Sandalwood


by 2030 initiative is a joint initiative of the
timber is used for making furniture, woodcraft
World Economic Forum and the Government
w

and musical instruments. The rare wavy grain


of Colombia. It aims to support city
variant is highly valued in Japan for its acoustic
w

governments, businesses and citizens, to


properties.The wood at the centre of the trunk
enable cities to live in harmony with nature by
w

(heartwood) is used as medicine. It is used for


2030. The initiative brings together

DPP 2023 DAY 167 23


https://upscmaterial.online/
Download From - https://upscmaterial.online/

Contact us :info@onlyias.com

OnlyIAS Nothing Else Visit :dpp.onlyias.in


Contact : +91-7007 931 912

multidisciplinary expertise, combines existing modified organisms (LMOs) resulting from


initiatives and surfaces innovative solutions to modern biotechnology that may have adverse
promote sustainable, inclusive and nature- effects on biological diversity, taking also into
positive urban development at a global scale. account risks to human health. It was adopted
The report examines the relationship between on 29 January 2000 and entered into force on
cities and nature and provides a clear path for 11 September 2003.
city leaders to embrace the BiodiverCities by
● Convention on Biological Diversity (CBD), a
2030 vision and increase urban
legally binding multilateral treaty adopted at
competitiveness and liveability through
the Earth Summit held in Rio de Janeiro,
nature.
1992.Objectives:
These are the other major reports that have been 1. Conservation of biological diversity;
released by the World Economic Forum:
2. Sustainable use of biological resources ( or
➢ Engaging Tomorrow's Consumer Report its components)

➢ Global Gender Gap Report 3. Fair & equitable sharing of benefits arising
➢ Global Information Technology Report from the use of genetic resources.
➢ Human Capital Report ● Convention on International Trade in
➢ Global Risk Report Endangered Species of Wild Fauna and
➢ Travel and Tourism Competitiveness Report
Flora(CITES) aims to control or prevent

e
international commercial trade in endangered
➢ Global Competitive Index

n
species or products derived from them.The
➢ Global Energy Architecture Performance Index Convention does not seek to directly protect

li
Report endangered species, rather it seeks to reduce

n
➢ Inclusive growth & Development Report the economic incentive to poach endangered
o
➢ Inclusive growth & Development Report
species and destroy their habitat by closing off
l.
the international market.
➢ Environmental Performance Index
a

● The Nagoya Protocol on Access to Genetic


ri

Reference: Resources and the Fair and Equitable Sharing


te

https://www.weforum.org/communities/biodivercit of Benefits Arising from their Utilization to the


ies-by-2030 Convention on Biological Diversity is an
a

https://www.businesstoday.in/latest/economy/stor international agreement which aims at sharing


m

y/nearly-half-of-gdp-in-cities-at-risk-from-nature- the benefits arising from the utilization of


loss-wef-survey-319339-2022-01-17 genetic resources in a fair and equitable way. It
c

entered into force on 12 October 2014, 90 days


s

after the date of deposit of the fiftieth


p

instrument of ratification.
.u

Q.14) Ans: B Reference:


Exp: https://indianexpress.com/article/cities/delhi/biolo
w

● The Cartagena Protocol on Biosafety to the gical-diversity-bill-referred-to-joint-committee-of-


parliament-7682731/
w

Convention on Biological Diversity is an


international agreement which aims to ensure https://prsindia.org/billtrack/the-biological-
w

the safe handling, transport and use of living diversity-amendment-bill-2021

DPP 2023 DAY 167 24


https://upscmaterial.online/
Download From - https://upscmaterial.online/

Contact us :info@onlyias.com

OnlyIAS Nothing Else Visit :dpp.onlyias.in


Contact : +91-7007 931 912

Q.15) Ans: D of endangered wild species and several


Exp: measures are in place to control threats from
● Statement 1 is incorrect: CITES(the Convention invasive alien species. Recently Wild Life
on International Trade in Endangered Species of (Protection) Amendment Bill-2021 that seeks to
Wild Fauna and Flora, also known as the provide for implementation of the Convention
Washington Convention) is a multilateral treaty on International Trade in Endangered Species of
to protect endangered plants and animals. It was Wild Fauna and Flora (CITES) is passed by lok
drafted as a result of a resolution adopted in sabha.
1963 at a meeting of members of the
International Union for Conservation of Nature Reference:
(IUCN). The convention was opened for https://indianexpress.com/article/india/lok-sabha-
signature in 1973 and CITES entered into force passes-bill-to-implement-global-norms-on-
on 1 July 1975. It classifies plants and animals endangered-species-8066792/
according to three categories, or appendices, https://www.downtoearth.org.in/news/wildlife-
based on how threatened. They are. biodiversity/wildlife-protection-amendment-bill-
1. Appendix I: It lists species that are in danger why-religious-or-any-other-purpose-clause-has-
of extinction. It prohibits commercial trade of enraged-animal-activists-84191
these plants and animals except in
extraordinary situations for scientific or
educational reasons. Q.16) Ans: B
2. Appendix II species: They are those that are not Exp:

e
threatened with extinction but that might ● Statement 1 is incorrect: The PPV&FR Act

n
suffer a serious decline in number if trade is introduced intellectual property protection in

li
not restricted. Their trade is regulated by Indian agriculture and is the world’s only IPR
permit. legislation which grants intellectual property

n
3. Appendix III species: They are protected in at rights not only to the plant breeders but also to
o
least one country that is a CITES member state the farmers. The act will recognize and protect
l.
and that has petitioned others for help in the rights of farmers in respect of their
a

controlling international trade in that species. contributions in conserving, improving and


ri

making available plant genetic resources for the


● Statement 2 is incorrect: The CITES Secretariat development of new plant varieties. Recently,
te

is administered by UNEP (The United Nations Protection of Plant Varieties and Farmers’ Rights
Environment Programme) and is located at Authority (PPV&FRA) under the PPV&FR Act has
a

Geneva, Switzerland. It plays a coordinating, revoked a plant variety protection certificate


m

advisory and servicing role in the working of the granted to PepsiCo India Holding on FC-5 potato
c

Convention(CITES). Appendices I, II and III to the variety (also called as FL-2027) on multiple
Convention Are lists of species afforded different grounds.
s

levels or types of protection from over- ● Statement 2 is correct: The Protection of Plant
p

exploitation. Varieties and Farmers' Rights (PPV&FR) Act, was


.u

● Statement 3 is correct: CITES is legally binding enacted in 2001 under Article 27(3) (b) of the
on state parties to the convention, which are Agreement on Trade-Related Aspects of
w

obliged to adopt their own domestic legislation Intellectual Property Rights (TRIPS). The act is in
w

to implement its goals (it does not take the conformity with International Union for the
place of national laws).India, being a CITES Protection of New Varieties of Plants (UPOV),
w

Party, actively prohibits the international trade 1978 (an intergovernmental organization, to

DPP 2023 DAY 167 25


https://upscmaterial.online/
Download From - https://upscmaterial.online/

Contact us :info@onlyias.com

OnlyIAS Nothing Else Visit :dpp.onlyias.in


Contact : +91-7007 931 912

provide and promote an effective system of problems. If Endosulfan is released to water, it


plant variety protection, with the aim of is expected to absorb to the sediment and may
encouraging the development of new varieties bioconcentrate in aquatic organisms.A global
of plants, for the benefit of society). In order to ban on the manufacture and use of endosulfan
fulfill WTO obligation, India enacted the was negotiated under the Stockholm
Protection of Plant Varieties and Farmers’ Rights Convention in April 2011. It is still used
Act in October, 2001 to protect the new plant extensively in India and China despite laws
varieties. The act covers all categories of plants against its use.The Supreme Court has
except micro-organisms. slammed the Kerala government for doing
“virtually nothing” for Endosulfan pesticide
Reference: exposure victims. The court said the State’s
https://indianexpress.com/article/cities/ahmedabad inaction was “appalling” and amounted to a
/registration-certificate-of-potato-variety-under- breach of the apex court’s 2017 judgment,
pepsico-india-revoked-7655214/ which had ordered the State to pay Rs 5 lakh
https://theprint.in/theprint-essential/case-against- each to the victims in three months. Five years
farmers-poor-documentation-why-pepsico-india- since the judgment, the court has realised that
lost-lays-variety-potato-rights/776644/ only eight out of 3,704 victims have been paid
compensation.

Q.17) Ans: C
Exp: Q.18) Ans: D

e
● Option 3 is correct : The Supreme Court has Exp:

n
slammed the Kerala government for doing ● Option D is correct: Deep learning is a type of

li
virtually nothing for endosulfan pesticide machine learning and artificial intelligence
exposure victims. The apex court in its 2017 (AI) that imitates the way humans gain certain

n
order in a PIL had directed that Rs 5 lakh be types of knowledge. Deep learning is an
o
paid to all victims for Rehabilitation of important element of data science, which
l.
Endosulfan Victims. It is an off-patent includes statistics and predictive
a

organochlorine pesticide developed in 1954. modeling.More commonly known as active


ri

It was used widely on crops like cashew, neural network (ANN) or deep neural network
cotton, tea, paddy, fruits and others until (DNN).It is a subset of machine learning and
te

2011. In 2011, the Supreme Court banned its comes under the broader umbrella of AI. It
production and distribution. This pesticide is a combines a series of computer commands or
a

known carcinogen, neurotoxin and genotoxin algorithms that stimulate activity and brain
m

(damages DNA). The health effects of the structure. Deep Learning is a subfield of
c

chemical include late sexual maturity, physical machine learning concerned with algorithms
deformities, poisoning etc. People, especially inspired by the structure and function of the
s

new-borns, have suffered deformities, health brain called artificial neural networks. Deep
p

complications and loss of family members due learning (aka deep structured learning or
.u

to exposure to the agrochemical. It became a hierarchical learning) is part of a broader family


highly controversial agrichemical due to its of machine learning methods based on
w

acute toxicity, potential for bioaccumulation, learning data representations, as opposed to


w

and role as an endocrine disruptor. Endosulfan task-specific algorithms. Deep learning neural
in the environment gets accumulated in food network consists of two terms: 1) Neural
w

chains leading to higher doses causing networks, a biologically-inspired programming

DPP 2023 DAY 167 26


https://upscmaterial.online/
Download From - https://upscmaterial.online/

Contact us :info@onlyias.com

OnlyIAS Nothing Else Visit :dpp.onlyias.in


Contact : +91-7007 931 912

paradigm which enables a computer to learn ● ISRO has started ‘Project NETRA’ , an early
from observational data. 2) Deep learning, a warning system in space to detect debris and
powerful set of techniques for learning in other hazards to Indian satellites.
neural networks. Application of Deep learning Source::-
are in Self-driving cars where the neural https://www.nhm.ac.uk/discover/what-is-space-
networks can be trained over parameters such junk-and-why-is-it-a-problem.html
as traffic patterns, traffic rules, weather and https://www.nasa.gov/mission_pages/station/news
road quality etc. and it can self-improvise to /orbital_debris.html
drive efficiently ; Weather forecasting where
training parameters could be wind pattern, air-
pressure, temperature and previous weather Q.20) Ans: B
records of the year etc. so that it could predict Exp:
weather phenomena without human ● Statement 1 is not correct : Norovirus has
intervention ; Automatic machine translation, become the most common cause of
deciphering complicated scripts and language gastroenteritis in all age groups, including
modeling. children. NoV is an important cause of
gastroenteritis in children as well as adults
Reference: worldwide. There is no specific treatment,
https://www.livemint.com/education/news/machine- rest and rehydration with warm fluids should
learning-becomes-most-acquired-skill-in-india-on- be enough.Transmission occurs predominantly
coursera-in-2021-11638783681363.html by the farcaloral route, directly or indirectly,

e
https://www.thehindu.com/sci-tech/technology/the- through the ingestion of contaminated water

n
rise-of-ai-chips/article65446670.ece or food or surfaces.People with norovirus

li
illness can shed billions of norovirus particles.
Q.19) Ans: C And only a few virus particles can make other

n
Exp: people sick. Because norovirus is highly
o
● Statement 1 is correct: - Space debris contagious, infection can easily be spread from
l.
encompasses both natural (meteoroid) and person to person. Norovirus causes most cases
a

artificial (man-made) particles. Meteoroids are of gastroenteritis epidemics that occur on


ri

in orbit about the sun, while most artificial cruise ships and in nursing homes. Sinusitis is
debris is in orbit about the Earth which is often a chronic (ongoing) disease that includes
te

commonly referred to as orbital debris. the symptoms of nasal obstruction (difficulty


● Statement 2 is correct: Kessler syndrome is an breathing through the nose), nasal drainage,
a

idea proposed by NASA scientist Donald decreased sense of smell, facial pressure, and
m

Kessler in 1978. He said that if there was too frequent sinus infections. Allergies and related
much space junk in orbit, it could result in a
c

respiratory problems such as asthma can also


chain reaction where more and more objects be associated with chronic sinusitis.
s

collide and create new space junk in the ● Statement 2 is correct : Norovirus is resistant
p

process, to the point where Earth's orbit to many disinfectants and can heat up to 60°C.
.u

becomes unusable. This situation would be Therefore, merely steaming food or


extreme, but some experts worry that a variant chlorinating water does not kill the virus. The
w

of this could be a problem one day, and steps virus can also survive many common hand
w

should be taken to avoid it ever happening. sanitisers.According to study, NoV was found
This idea was also popularised in the movie to be the second most predominant virus after
w

Gravity. rotavirus.NoV is a highly contagious virus and

DPP 2023 DAY 167 27


https://upscmaterial.online/
Download From - https://upscmaterial.online/

Contact us :info@onlyias.com

OnlyIAS Nothing Else Visit :dpp.onlyias.in


Contact : +91-7007 931 912

transmission occurs at a rapid pace because of ● This year, World Wildlife Day (WWD) has been
heavy viral shedding by the infected person. celebrated under the theme “Recovering key
Even the lowest infectious dose is enough to species for ecosystem restoration”. The
set off extensive outbreaks. Experts also point celebrations sought to draw attention to the
out that NoV could increasingly become a conservation status of some of the most
public health concern because of rapid critically endangered species of wild fauna and
expansion of the food industry. flora, and to drive discussions towards
● Statement 3 is not correct : A person can be imagining and implementing solutions to
infected with different varieties of norovirus conserve them. All conversations have been
several times in their lives, but developing inspired by and sought to inform efforts
immunity to one type doesn't provide towards the achievement of UN Sustainable
protection against other varieties.The virus Development Goals 1 (No Poverty), 2 (Zero
can be transferred from an infected to a hunger) 12 (Ensure sustainable consumption
healthy individual through contaminated and production patterns), 13 (Climate Action)
surfaces or food.The virus can primarily be 14 (Life Below Water) and 15 (Life on Land).
transmitted from oral-faecal.Disease Reference :
outbreaks typically occur aboard cruise ships, https://indianexpress.com/article/lifestyle/life-
in nursing homes, dormitories, and other style/world-wildlife-day-2022-theme-history-
closed spaces.Norovirus is sometimes called significance-7798784/
the stomach flu or stomach bug. However,
norovirus illness is not related to the flu which

e
is caused by influenza virus. Q.22) Ans: C

n
Reference : https://www.hindustantimes.com/india- Exp:

li
news/norovirus-confirmed-in-kerala-s-wayanad-here- ● Statement 1 is correct:The current plan is to
s-what-you-need-to-know-101636717225374.html have two unmanned and one manned flight

n
under Gaganyaan Programme. Gaganyaan is
o
significant because it is the first indigenous
l.
mission that will send Indian astronauts to
a

Q.21) Ans: A space. If it succeeds, India will be the fourth


ri

Exp: country to have sent a human to space, the other


● Option 1 is correct : On 20 December 2013, at three being the US, Russia and China. The main
te

its 68th session, the United Nations General objective of the Gaganyaan mission is
Assembly (UNGA) proclaimed 3 March – the technology demonstration. The programme will
a

day of signature of the Convention on source 60-70% components and value-added


m

International Trade in Endangered Species of services from Indian industries.The spacecraft


c

Wild Fauna and Flora (CITES) in 1973 – as UN consists of a service module and a crew module,
World Wildlife Day to celebrate and raise collectively known as the Orbital Module.
s

awareness of the world’s wild animals and ● Statement 2 is incorrect: The launch vehicle
p

plants. The UNGA resolution also designated used for this mission will be the
.u

the CITES Secretariat as the facilitator for the Geosynchronous Satellite Launch Vehicle GSLV
global observance of this special day for Mk III. GSLV Mk III is a 3 stage heavy lift launch
w

wildlife on the UN calendar. World Wildlife Day vehicle developed by ISRO. GSLV Mk III has the
w

has now become the most important global required payload capacity for the mission. It can
annual event dedicated to wildlife. carry satellites weighing 10 tons to Low Earth
w

Orbit (LEO) – at an altitude of 600 kms.It will

DPP 2023 DAY 167 28


https://upscmaterial.online/
Download From - https://upscmaterial.online/

Contact us :info@onlyias.com

OnlyIAS Nothing Else Visit :dpp.onlyias.in


Contact : +91-7007 931 912

circle Earth at a low-earth-orbit at an altitude of Situational Awareness’ to predict collision from


300-400 km from the earth for 5-7 days. space debris.
● Statement 3 is correct: As part of this ● Statement 3 is incorrect: Much of the debris is in
programme, two unmanned missions and one low Earth orbit (LEO), though some debris can be
manned mission will be carried out.ISRO is found in geostationary orbit. There are about
planning to launch the first uncrewed mission 500,000 pieces of debris the size of a marble or
under Gaganyaan in 2022, following which the larger orbiting the Earth, traveling at speeds up to
second unmanned mission “Vyommitra” will 17,500 mph.
carry a robot and this will then be followed by
the manned mission. The uncrewed missions are References:
for technology demonstration, safety and https://www.thehindu.com/news/national/kerala/isr
reliability verification and will aim to study the o-to-step-up-tracking-of-space-
performance of systems before crewed flight. debris/article65274219.ece#:~:text=With%20space%2
The robot will mimic the space crew activities set 0junk%20posing%20increasing,and%20Analysis%20(N
for the human flight mission in order to assess all ETRA)%20project
the possible challenges prior to the final mission.

Reference: Q.24) Ans: B


https://indianexpress.com/article/technology/scienc Exp:
e/isro-targets-gaganyaan-launch-before-august-15- ● Statement 1 is correct: Astronomers have
7704957/ observed an excess of positrons having energy of

e
https://www.hindustantimes.com/science/in-new- more than 10 giga-electron volts, or 10 GeV.

n
year-message-isro-chief-talks-about-2022-plans- Positrons with energy more than 300 GeV,

li
gaganyaan-venus-mission-101641266084861.html however, are lower in comparison to what
astronomers expect. This behavior of positrons

n
between 10 and 300 GeV is what astronomers call
o
Q.23) Ans: C the positron excess. GeV (countable and
l.
Exp: uncountable, plural GeVs) is giga-electron volt , a
a

● Statement 1 is correct: The Kessler Syndrome is unit used for measuring the energy of subatomic
ri

a condition in which the density of objects in Low particles


Earth Orbit (LEO) is high enough that collisions ● Statement 2 is correct: The Milky Way is a huge
te

between objects could cause a cascade where each collection of stars, dust, and gas. It is called a spiral
collision generates space debris that increases the galaxy because if viewed from the top or bottom,
a

likelihood of further collisions. It may lead to a it will look like a spinning pinwheel.
m

situation in which orbit would become impassable 1. Cosmic rays are produced in supernovae
c

in the long run. explosions. These waves propagate through


● Statement 2 is correct: : Space debris is tracked by these giant clouds before they reach the Earth.
s

a number of countries, including Germany, France, Cosmic rays interact with molecular hydrogen
p

UK and USA. ISRO has come up with ‘Project and can give rise to other cosmic rays primarily
.u

NETRA’, an early warning system in space to detect electrons and positrons. Further, cosmic rays
debris and other hazards to Indian satellites. propagate through these clouds and decay
w

Indraprastha Institute of Information Technology from their original forms and lose their wave
w

Delhi is working on a project titled ‘Orbit energy by energising the clouds. Some may
computation of Resident Space Objects for Space also get re-energised.
w

DPP 2023 DAY 167 29


https://upscmaterial.online/
Download From - https://upscmaterial.online/

Contact us :info@onlyias.com

OnlyIAS Nothing Else Visit :dpp.onlyias.in


Contact : +91-7007 931 912

2. Researchers from the Raman Research /nasa-picks-venus-as-hot-spot-for-two-new-robotic-


Institute(RRI) found out that Cosmic rays missions-7341904/
interaction with giant clouds is a viable
contributor to the origin of positron excess
phenomenon. Q.26) Ans: B
● Statement 3 is incorrect: Positron is the Exp:
antiparticle of the electron. Also known as anti- ● Statement 1 is correct: The Defence Acquisition
electron, it has the same properties as the electron Council, chaired by the Defence Minister, approved
with the exception of electric charge.. Electrons a request for proposal for construction of six
have a negative charge while the positron has a Conventional Submarines under Project P 75 (I). As
positive charge. part of a 30-year Submarine construction
programme to acquire national competence in
Reference: submarine construction, these submarines will be
https://pib.gov.in/PressReleaseIframePage.aspx?PRID made under Strategic Partnership Model. The
=17151111. submarines will include indigenous technologies
like Air Independent Propulsion as well as the new
Q.25) Ans: D technologies and manufacturing capabilities to be
Exp: brought by foreign partners, helping India to
● Statement 1 is correct: EnVision is an European achieve self-reliance.
Space Agency led mission to planet Venus, with ● Statement 2 is incorrect: Under BrahMos
contributions from NASA. It is likely to be launched Aerospace, India is developing hypersonic missiles

e
sometime in the 2030s. It will study the planet’s and not under project 75. China, Russia and the

n
atmosphere and surface, monitor trace gasses in U.S. are all pursuing hypersonic weapons

li
the atmosphere and analyze its surface technologies, setting stage for an arms race. A
composition. Venus is the hottest planet in the hypersonic missile travels at speeds of Mach 5 and

n
solar system because of the heat that is trapped by faster than speed of sound (3836 mph). They are
o
its thick cloud cover. highly maneuverable and do not follow a
l.
● Statement 2 is correct: NASA has announced two predictable arc as they travel..
a

missions to Venus, Earth’s closest planetary ● Statement 3 is incorrect: The Agni series of ballistic
ri

neighbor, as part of its ‘Discovery Program’ that missiles is being developed under the Integrated
aims to explore and study the solar system. Guided Missile Development Programme of the
te

DAVINCI (Deep Atmosphere Venus Investigation of Defense Research and Development Organization
Noble gases, Chemistry, and Imaging) mission of India. Agni is a surface to surface ballistic missile.
a

VERITAS (Venus Emissivity, Radio Science, InSAR,


m

Topography, and Spectroscopy) . The last US probe Reference:


c

to visit the planet was the Magellan orbiter in 1990. https://www.thehindu.com/news/national/defence-


● Statement 3 is correct: Venus is the hottest planet ministry-clears-deal-for-6-
s

in the solar system because of the heat that is submarines/article34725695.ece


p

trapped by its thick cloud cover. Venus spins on its


.u

axis from east to west. Venus also does not have a


moon and no rings. It is called the Earth’s twin Q.27) Ans: B
w

because of their similar sizes. Exp:


w

● Statement 1 is incorrect: A scramjet engine is an


Reference: improvement over the ramjet engine as it
w

https://indianexpress.com/article/technology/science efficiently operates at hypersonic speeds and

DPP 2023 DAY 167 30


https://upscmaterial.online/
Download From - https://upscmaterial.online/

Contact us :info@onlyias.com

OnlyIAS Nothing Else Visit :dpp.onlyias.in


Contact : +91-7007 931 912

allows supersonic combustion. Thus it is known as ● All Statements are correct: India has developed
Supersonic Combustion Ramjet, or Scramjet. The the world's first micro-sensor based explosive trace
Scramjet engine designed by ISRO uses Hydrogen detector (ETD), using a micro-electromechanical
as fuel and the Oxygen from the atmospheric air as system (MEMS), called NanoSniffer. The
the oxidiser. In an air-breathing scramjet engine, NanoSniffer was developed by an IIT Bombay
air from the atmosphere is rammed into the incubated startup called NanoSniff Technologies. It
engine’s combustion chamber at a supersonic is marketed by the IIT Delhi incubated startup
speed of more than Mach two. In the chamber, Kritikal Solutions. It provides trace detection of
the air mixes with the fuel to ignite a supersonic nano-gram quantities of explosives & delivers
combustion but the cruiser’s flight will be at a results in seconds. The NanoSniffer can detect
hypersonic speed of Mach six to seven. So it is explosives in less than ten seconds. The device is
called supersonic combustion ramjet or Scramjet. priced nearly Rs 10 lakhs, this is one-third of the
● Statement 2 is incorrect: A ramjet is a form of air- price of existing devices. NanoSniffer uses a trace
breathing jet engine that uses the vehicle’s forward detection technique that can detect nanogram
motion to compress incoming air for combustion quantities of explosives, consequently
without a rotating compressor. Fuel is injected in anticipating a wide range of military, commercial
the combustion chamber where it mixes with the and homemade explosives threats.
hot compressed air and ignites. A ramjet-powered
vehicle requires an assisted take-off like a rocket References:
assist to accelerate it to a speed where it begins to https://www.tribuneindia.com/news/nation/ramesh-
produce thrust. Ramjets work most efficiently at pokhriyal-launches-nanosniffer-a-microsensor-based-

e
supersonic speeds around Mach 3. explosive-trace-detector-236848

n
● Statement 3 is correct: Hypersonic defines speed https://iit-techambit.in/nanosniffer/

li
of about Mach 5 or at least at 1.6 km per second or
higher. Hypersonic weapons travel within the

n
atmosphere and can maneuver midway which Q.29) Ans: B o
combined with their high speeds makes their Exp:
l.
detection and interception extremely difficult. ● Statement 1 is incorrect: The Prithvi-I is an Indian
a

They use scramjet technology, which is a type of Air short-range ballistic missile with a range of 150
ri

Breathing propulsion System. A hypersonic cruise km. It is the first in a lineage of indigenous land and
missile is boosted by a rocket to hypersonic speed sea-based missiles which includes the Prithvi-II, the
te

and then uses an air-breathing engine called a ship-launched Dhanush, and the Prithvi Air
scramjet to sustain that speed. Defense interceptor. The Indian Army has
a

withdrawn the Prithvi-I from service and replaced


m

References: it with the Prahaar missile.


c

https://www.thehindu.com/news/national/drdo- ● Statement 2 is incorrect: The Helina (the Army


successfully-tests-hypersonic-technology- version) and Dhruvastra (Indian Airforce version)
s

demonstrator-vehicle/article61709465.ece are helicopter-launched versions of third-


p

https://www.isro.gov.in/launchers/isro’s-scramjet- generation anti-tank guided missiles (the Nag


.u

engine-technology-demonstrator-successfully-flight- missile system). The missile system has all-


tested weather day-and-night capability and can defeat
w

battle tanks with conventional armor and explosive


w

reactive armor.
Q.28) Ans: D ● Statement 3 is correct: India's indigenous Light
w

Exp: Combat Aircraft, Tejas, has added the fifth

DPP 2023 DAY 167 31


https://upscmaterial.online/
Download From - https://upscmaterial.online/

Contact us :info@onlyias.com

OnlyIAS Nothing Else Visit :dpp.onlyias.in


Contact : +91-7007 931 912

generation Python-5 Air-to-Air Missile (AAM) to its Tupolev Tu-142 aircraft. It can conduct anti-
weapons capability. The Python-5 is the second submarine warfare (ASW), anti-surface warfare
AAM of Israeli origin to be integrated on the Tejas (AsuW), intelligence, maritime patrol, and
fighter. It is an infra-red guided missile with a range surveillance and reconnaissance missions.
of at least 20km. The first missile was the Beyond
Visual Range (BVR) AAM Derby which is a radar- Reference:
guided weapon with a range of over 50km. It is a https://www.thehindu.com/news/national/rajnath-
dual use missile suitable for air-to-air and surface- singh-begins-visit-to-karwar-naval-base-
to-air missions. It is powered by a solid propellant kochi/article34944019.ece
rocket engine. The propulsion system provides a
speed of Mach 4 and an operational range of Q.31) Ans: A
more than 20 km. It is also equipped with lock-on- Exp:
before launch (LOBL) and lock-on-after launch ● Statement 1 is correct: Ministry of Electronics and
(LOAL) capabilities. Information and Technology (MeitY) has released a
draft National Strategy on Blockchain. A blockchain
References: is essentially a digital ledger of transactions (DLT)
https://pib.gov.in/PressReleaseIframePage.aspx?PRID that is duplicated and distributed across the entire
=1714563 network of computer systems on the blockchain.
Blockchain is a specific type of DLT in which blocks
are linearly connected to each other.
Q.30) Ans: A ● Statement 2 is correct: Copy of each transaction

e
Exp: along with its hash is stored in the ledger shared

n
● Statement 1 is correct: ‘Project Seabird’ Is the across all participants of the network. Blockchains

li
largest naval infrastructure project for India, create trust by acting as a shared database,
Project Seabird involves creation of a naval base at distributed across vast peer-to-peer networks that

n
Karwar (Karnataka) on the west coast of India. This have no single point of failure and no single source
o
will provide the Indian Navy with its largest naval of truth, implying that no individual entity can own
l.
base on the west coast and also the largest naval a blockchain network, and no single entity can
a

base east of the Suez Canal. modify the data stored on it unilaterally without
ri

● Statement 2 is incorrect: Indian Air Force and the consensus of its peers.
DRDO successfully tested SAAW. SAAW is ● Statement 3 is incorrect: Blockchain can be used in
te

indigenously designed smart weapon, with 100 km Permissioned and Permission less models. Most of
range, that can target enemy airfield assets such as the first generation blockchains are permissionless
a

radars, bunkers, taxiways and runways. Electro such as Bitcoin and Ethereum. Permissionless
m

optical seeker based flight test of this class of bomb blockchain systems are highly transparent, as
c

has been conducted for the first time in the everyone can see all the transactions on the ledger.
country. It has been equipped with Imaging Infra- Permissioned model has applications in various
s

Red Seeker technology to enhance precision strike domains such as healthcare, cyber security,
p

capability Governance, media, logistics & hospitality,


.u

● Statement 3 is correct: US Approved Proposed Sale education, legal, energy, smart cities etc.
Of Six P-8I Patrol Aircraft To India. P-8I is a long-
w

range, multi-mission maritime patrol aircraft References:


w

offered by Boeing for the Indian Navy is based on https://www.meity.gov.in/writereaddata/files/Nation


the Boeing Next-Generation 737-800 aircraft. P-8I al_BCT_Strategy.pdf
w

replaced the aging fleet of the Indian Navy’s

DPP 2023 DAY 167 32


https://upscmaterial.online/
Download From - https://upscmaterial.online/

Contact us :info@onlyias.com

OnlyIAS Nothing Else Visit :dpp.onlyias.in


Contact : +91-7007 931 912

https://www.euromoney.com/learning/blockchain- LED light bulb emit pulses of light that are


explained/what-is-blockchain undetectable to the human eye and within those
emitted pulses, data can travel to and from
Q.32) Ans: C receivers. Light bounces off of surfaces and
Exp: therefore LiFi is not strictly a line-of-sight
● Statement 1 is incorrect: Sandesh is an instant technology i.e. data rate is not dependent on the
messaging platform launched by National line of sight but on the signal quality at the device
Informatics Center (NIC), Ministry of Electronics Advantages of LiFi systems includes Providing
and Information Technology. Initially it was open ultra-fast data connections, Useful in urban areas
only to government officers, and now has been where radio spectra are congested and also in rural
released for the common public. It was launched as areas wherein Fiber Optic Cables or networks are
part of the government strategy to push for use of not reachable etc.Unlike Wi-Fi that uses radio
India-made software and build an ecosystem of waves for data transmission, LiFi uses light waves.
indigenously developed products. Its light waves signal coverage distance is about 10
● Statement 2 is correct: ‘’DearCry '’, is a new family meters.
of ransomware, detected by Microsoft, attacking
business email servers. It is being used after an References: https://www.jagranjosh.com/general-
initial compromise of unpatched on-premises knowledge/li-fi-light-fidelity-advantages-
Exchange Servers. Ransomware is malware that disadvantages-applications-and-more-1611311388-1
employs encryption to hold a victim's information
at ransom. Malware is the collective name for a Q.34) Ans: C

e
number of malicious software variants, including Exp:

n
viruses, ransomware and spyware. ● Statement 1 is correct: The Department of

li
● Statement 3 is incorrect: ISRO demonstrated free- Biotechnology conducted the first Chimeric
space Quantum Key Distribution (QKD) over 300 Antigen Receptor T-cell (CAR-T) therapy. It is a way

n
meter. Experiment is a major breakthrough to get immune cells called T cells (a type of white
o
towards ISRO’s goal of Satellite Based Quantum blood cell) to fight cancer by editing them in the lab
l.
Communication, where ISRO will demonstrate the so they can find and destroy cancer cells. CAR T-cell
a

technology between two Indian ground therapy is used to treat certain blood cancers, and
ri

stations.Quantum key distribution (QKD) is a it is being studied in the treatment of other types
provably secure method of communication of cancer.
te

(depending only on the laws of quantum ● Statement 2 is correct: T cells are taken from a
mechanics) that would not be vulnerable to patient’s blood then the gene for a special receptor
a

attacks from any amount of computational that binds to a certain protein on the patient’s
m

power. cancer cells is added to the T cells in the laboratory.


c

The special receptor is called a chimeric antigen


Reference: https://news.sophos.com/en- receptor (CAR). Large numbers of the CAR T cells
s

us/2021/03/15/dearcry-ransomware-attacks-exploit- are grown in the laboratory and given to the


p

exchange-server-vulnerabilities/ patient by infusion.


.u

● Statement 3 is incorrect: T cells are taken from the


Q.33) Ans: D patient's blood and are changed in the lab by
w

Exp: adding a gene for a man-made receptor (called


w

● All Statements are incorrect: LiFi is a wireless CAR). This helps them better identify specific
optical networking technology, which uses light cancer cell antigens. The CAR T cells are then given
w

emitting diodes (LEDs) to transmit data. It makes a back to the patient. It is also sometimes talked

DPP 2023 DAY 167 33


https://upscmaterial.online/
Download From - https://upscmaterial.online/

Contact us :info@onlyias.com

OnlyIAS Nothing Else Visit :dpp.onlyias.in


Contact : +91-7007 931 912

about as a type of cell-based gene editing, because Q.36) Ans: D


it involves altering the genes inside T cells to help Exp:
them attack the cancer. ● All Statements are correct: It is India’s first Cattle
Genomic Chip for the conservation of pure
References: varieties of indigenous cattle breeds and helps
https://www.cancer.gov/publications/dictionaries/ca towards doubling farmers' income by 2022. The
ncer-terms/def/car-t-cell-therapy manufacturing of this chip is in collaboration with
Rashtriya gokul mission and a great example of
Q.35) Ans: A Atma nirbhar Bharat. And also the chip exemplifies
Exp: the application of scientific knowledge and
● Statement 1 is correct: Bt cotton is the only GM innovations for “Ease of Living” for all sections of
crop that has been approved for commercial society. The milk of indigenous animals is high in fat
cultivation (in 2002) by the Government of India. and SNF content. It is the largest cattle chip in the
GM mustard Dhara Mustard Hybrid 11 (DMH 11) world with 11,496 markers. Till now India’s dairy
developed by Delhi University is pending for development program has been referring to chips
commercial release. Genetic Engineering Appraisal which are developed for foreign western breeds of
Committee (GEAC), has allowed biosafety research cattle. This indigenous chip was developed by the
field trials of two new transgenic varieties of National Institute of Animal Biotechnology
indigenously developed Bt Brinjal. (Hyderabad), an autonomous institution under the
● Statement 2 is incorrect: FSSAI also stated that the aegis of the Department of Biotechnology under
tolerance limit for “adventitious presence” of Ministry of science and technology.

e
genetically modified organisms (GMOs) at 1% will

n
be permissible in the imported food crops References:

li
consignments. Adventitious presence refers to https://pib.gov.in/PressReleasePage.aspx?PRID=1745
unintentional or incidental presence of trace 479.

n
amounts of GM material in non-GM crops. o
● Statement 3 is correct: Globally GM crops were Q.37) Ans: B
l.
commercially introduced in 1996. Crops such as Exp:
a

corn, cotton, and soybean have been engineered ● Statement 1 is correct: RNA interference (RNAi)
ri

to resist insect pests and herbicides and are now technology has been suggested for manufacturing
planted widely in many parts of the world. GMOs pesticides.As proteins necessary for survival of
te

are organisms in which the genetic material (DNA) pests at cellular level. Using RNAi
has been altered in a way that does not occur technology,scientists created a specific RNA to
a

naturally by mating and/or natural recombination. prevent those proteins from being formed in the
m

Foods produced from or using GM organisms are pest. This can help in targeting only specific pests
c

referred to as GM foods. Bt cotton is the only and avoid damage to friendly insects such as bees,
genetically modified (GM) crop that has been earthworms etc. RNA molecules can be externally
s

approved for commercial cultivation in 2002 by the delivered to plants, through applications like spray,
p

Government of India. stem injection, root drenching, or seed treatment.


.u

Thus, by vaccinating the plants, i.e., delivering the


References: specific RNA molecule to pests from plants, they
w

https://www.downtoearth.org.in/news/health/fssai-s- are protected from infestation.


w

gmo-threshold-for-imported-food-crops- ● Statement 2 is incorrect: RNA is more resistant to


unacceptably-high-says-letter-to-health-minister- damage from UV light than DNA and DNA is
w

75675 vulnerable to damage by ultraviolet light. UV

DPP 2023 DAY 167 34


https://upscmaterial.online/
Download From - https://upscmaterial.online/

Contact us :info@onlyias.com

OnlyIAS Nothing Else Visit :dpp.onlyias.in


Contact : +91-7007 931 912

radiation causes two classes of DNA lesions: a global mandate to catalyze solar growth by
cyclobutane pyrimidine dimers (CPDs,)and 6-4 helping to reduce the cost of financing and
photoproducts (6-4 PPs,). Both of these lesions technology. ISA is also the nodal agency for
distort DNA's structure, introducing bends or kinks implementing One Sun One World One Grid
and thereby impeding transcription and initiative, which seeks to transfer solar power
replication. generated in one region to feed the electricity
● Statement 3 is correct: RNA converts the genetic demands of others.
information contained within DNA to a format ● Statement 3 is incorrect: The United States of
used to build proteins, and then moves it to America has now become the 101st member
ribosomal protein factories. DNA replicates and country to join the International Solar Alliance (ISA)
stores genetic information. It is a blueprint for all Earlier it declined to join ISA but now it joined this
genetic information contained within an organism. alliance. The Headquarters is in India with its
It is a hereditary material in humans and almost all Secretariat being set up in Gurugram.
other organisms. RNA only has one strand, but like
DNA, is made up of nucleotides. DNA consists of References:
two strands, arranged in a double helix. RNA https://indianexpress.com/article/world/climate-
nucleotides contain the nitrogenous bases change/usa-joins-international-solar-alliance-as-
adenine, cytosine, and guanine. However, they do 101st-member-country-7617127/
not contain thymine, which is instead replaced by
uracil, symbolized by a “U.” RNA exists as a single- Q.39) Ans: B
stranded molecule rather than a double-stranded Exp:

e
helix. Molecular biologists have named several ● Statement 1 is incorrect: A hydrogen fuel cell is an

n
kinds of RNA on the basis of their function. These electrochemical device that converts chemical

li
include messenger RNA (mRNA), transfer RNA energy of a fuel hydrogen into electrical energy.
(tRNA), and ribosomal RNA (rRNA)—molecules Galvanic cells that are designed to convert the

n
that are involved in the production of proteins energy of combustion of fuels like hydrogen,
o
from the DNA code. methane, methanol, etc. directly into electrical
l.
energy are called fuel cells. One of the most
a

References: https://www.thehindu.com/sci- successful fuel cells uses the reaction of hydrogen


ri

with oxygen to form water.


tech/science/new-techniques-to-study-individual-
● Statement 2 is correct: The fuel cell solves dual
te

cells/article26429416.ece purpose as it can run the vehicle and can generate


standby power in case of emergencies as well. Only
a

Q.38) Ans: B water is the residue of HFCs which can be used in


m

Exp: the desert areas.A hydrogen based fuel is highly


c

● Statement 1 is incorrect: It is an Indian initiative inflammable, the production, transport and


storage of the fuel is a possible challenge
s

that was launched by the Prime Minister of India


● Statement 3 is correct: Vehicles running on HFCs
p

and the President of France on 30th November


2015 in Paris, France on the side-lines of the are best for heavy duty vehicles as electric vehicles
.u

are only limited to lighter vehicles. Hydrogen is also


UNFCCC Cop-21 with 121 solar resource rich
a clean fuel, no emission of carbon monoxide,
w

countries lying fully or partially between the tropic hydrocarbons and particulate matter is there.
of Cancer and tropic of Capricorn as prospective
w

members. References:
● Statement 2 is correct: The ISA is an https://pib.gov.in/PressReleasePage.aspx?PRID=1806
w

intergovernmental treaty-based organization with 563


DPP 2023 DAY 167 35
https://upscmaterial.online/
Download From - https://upscmaterial.online/

Contact us :info@onlyias.com

OnlyIAS Nothing Else Visit :dpp.onlyias.in


Contact : +91-7007 931 912

Q.40) Ans: D is an ionized state of matter similar to a gas. A gas


Exp: becomes plasma at extreme temperatures. The
● All statements are incorrect: Indian Oil machine has been designed specifically to produce
Corporation Ltd. entered into a joint venture with 500 MW of fusion power to demonstrate the
Israel-based battery technology startup Phinergy integrated operation of technologies for a fusion
to develop aluminium-air technology-based power plant such as heating, control, diagnostics,
battery systems for electric vehicles and stationary cryogenics and remote maintenance.
storage, as well as hydrogen storage solutions. Its
a battery that uses aluminium alloy plates as anode References:
,air as electrolyte and air electrode as cathode. https://www.thehindu.com/sci-
Aluminium-air batteries has big advantages over tech/science/scientists-in-britain-smash-fusion-
Lithium-ion batteries is a significantly cheaper, energy-record/article38403008.ece
lighter and more energy-dense alternative.
Expected to offer a much greater range of 400 km Q.42) Ans: B
or more per battery compared to lithium-ion Exp:
batteries (150-200 kilometers) per full charge.One ● Statement 1 is incorrect: AMR is the ability of
of the key downsides of aluminium-air batteries is a microorganism (like bacteria, viruses, and
that they cannot be recharged like lithium-ion some parasites) to stop an antimicrobial (such
batteries. Apart from aluminium-air, other metal– as antibiotics, antivirals and antimalarials)
air batteries that have been extensively studied are from working against it. The World Health
sodium (Na)–air, potassium (K)–air, zinc (Zn)–air, Organisation (WHO) has declared that AMR is

e
magnesium (Mg)–air etc. one of the top 10 global public health threats

n
facing humanity. Antibiotic resistance has been

li
References: found in all regions of the world. AMR
https://www.pib.gov.in/PressReleasePage.aspx?PRID= Surveillance Network: ICMR has established

n
1794781 AMR surveillance and research network
o
(AMRSN) in 2013, to generate evidence and
l.
capture trends and patterns of drug resistant
a

Q.41) Ans: A infections in the country. This network


ri

Exp: comprises 30 tertiary care hospitals, both


● Statement 1 is correct: Scientists in the UK have private and government.
te

managed to produce the largest amount of energy ● Statement 2 is correct: Following are the
so far from a nuclear fusion reaction. The energy causes of AMR:
a

was produced in a machine called a tokamak, a 1. Poor hygiene and infection prevention
m

doughnut-shaped apparatus. The tokamak is an and control.


c

experimental magnetic fusion device designed to 2. Overuse of antimicrobials


harness the energy of fusion. Inside a tokamak, the 3. Over prescription by doctors and
s

energy produced through the fusion is absorbed as Nonprescription purchase in many


p

heat in the walls of the vessel, which will be used countries.


.u

by a fusion power plant to produce steam and then 4. If a person does not complete a course
electricity by way of turbines and generators. of antimicrobial drugs, some microbes
w

● Statement 2 is incorrect: The device uses magnetic may survive and develop resistance to
w

fields to contain and control the hot plasma, which the drug.
enables the fusion between deuterium and tritium
w

nuclei to produce great amounts of energy. Plasma

DPP 2023 DAY 167 36


https://upscmaterial.online/
Download From - https://upscmaterial.online/

Contact us :info@onlyias.com

OnlyIAS Nothing Else Visit :dpp.onlyias.in


Contact : +91-7007 931 912

5. Practice of adding antibiotics to SOPs) for enabling comparison of


agricultural feed promotes drug antimicrobial resistance patterns in
resistance. animals and humans.
6. Environment plays a significant role 6. FSSAI has set certain guidelines
wherein waste from farms, factories, limiting the antibiotics in food
communities (such as mass bathing) products such as fish and honey.
and healthcare settings can contribute 7. In 2012, India’s medical societies
to AMR through environmental routes. adopted the Chennai Declaration, a set
● Statement 3 is correct: Initiative taken in India of national recommendations to
to combat AMR are as following: promote antibiotic stewardship.
1. National programme on AMR 8. A separate Schedule H-1 has been
containment was launched during incorporated in Drug and Cosmetic
12th FYP in 2012-17. Under this rules to regulate the sale of
programme, AMR Surveillance antimicrobials in the country. sale of
Network has been strengthened by certain antimicrobials included in
establishing labs in State Medical schedule H1 without prescription is
Colleges. unlawful.
2. National Action Plan on Antimicrobial
Resistance (NAP-AMR) to establish and Reference:
strengthen governance mechanisms, https://pib.gov.in/PressReleaseIframePage.aspx?PRID
change prescription practices and =1738156

e
consumer behavior and to scale up

n
infection control and antimicrobial Q.43) Ans: D

li
surveillance. Exp:
3. The Red Line campaign demands that ● Statement 1 is correct: The Defence Research

n
prescription-only antibiotics be and Development Organisation (DRDO)
o
marked with a red line, to discourage successfully test-fired the new generation
l.
the over-the-counter sale of nuclear-capable ballistic missile ‘Agni Prime’. It
a

antibiotics. is a canisterised missile and can be


ri

4. National Health Policy, 2017 terms transported easily and fired at very short
antimicrobial resistance as one of the notice. It will replace Agni-1 and Agni-2
te

key healthcare issues and prioritizes missiles in India’s arsenal. It is a two-stage


the development of guidelines canisterised solid propellant missile with dual
a

regarding antibiotic use and check on redundant navigation and guidance system. It
m

restricting the growth of antibiotics. has been termed as a new generation


c

5. ICMR has undertaken a project on an advanced variant of the Agni class of missiles
“Integrated One Health Surveillance with improved parameters, including
s

Network for Antimicrobial Resistance” maneuvering and accuracy. The surface-to-


p

in collaboration with Indian Council of surface ballistic missile has a range of 1,000 to
.u

Agriculture research (ICAR) to assess 2,000 km. Canisterisation of missiles reduces


the preparedness of Indian Veterinary the time required to launch the missile while
w

laboratories to participate in improving the storage and ease of handling.


w

integrated AMR surveillance network. ● Statement 2 is correct: The Agni-P and Agni-5
ICMR has also created a veterinary ballistic missiles trace their origins back to the
w

standard operating procedure (Vet- Integrated Guided Missile Development

DPP 2023 DAY 167 37


https://upscmaterial.online/
Download From - https://upscmaterial.online/

Contact us :info@onlyias.com

OnlyIAS Nothing Else Visit :dpp.onlyias.in


Contact : +91-7007 931 912

Programme (IGMDP), which was spearheaded is able to defeat the interceptor missiles and
by former DRDO chief and ex-Indian president also has the ability to change its path after
Dr APJ Abdul Kalam in the early 1980s. Agni covering a certain range mid-air. It is powered
class of missiles are the mainstay of India’s by a solid-propellant rocket motor and many
nuclear launch capability, which also includes new technologies. The missile guidance system
the Prithvi short-range ballistic missiles, includes a state-of-the-art navigation system
submarine launched ballistic missiles and and integrated avionics. The missile has a
fighter aircraft. Agni-V, an Inter-Continental range of 150-500 kilometers and can be
Ballistic Missile (ICBM) with a range of over launched from a mobile launcher. Pralay will
5,000 km, had been tested several times and be the longest-range surface-to-surface missile
validated for induction. in the inventory of the Army.
IGMDP (Integrated Guided Missile ● Statement 2 is correct: It is a derivative of the
Development Program) was conceived by Dr Prahaar missile programme, which was tested
A.P.J. Abdul Kalam to enable India to attain for the first time in 2011. Prahaar is a 150-
self-sufficiency in the field of missile kilometre-range surface-to-surface missile.
technology. It was approved by the The main goal is to close the gap between the
Government of India in 1983 and completed in Pinaka multi-barrel rocket launcher and guided
March 2012. The 5 missiles (P-A-T-N-A) Prithvi missile variants. The missile can be
developed under this program are: launched from a mobile launcher and has a
1. Prithvi: Short range surface to surface range of 150-500 kilometres. The Pralay missile
ballistic missile. will be the Army's longest-range surface-to-

e
2. Agni: Ballistic missiles with different surface missile.

n
ranges, i.e. Agni (1,2,3,4,5)

li
3. Trishul: Short range low level surface Reference:
to air missile. https://www.pib.gov.in/PressReleasePage.aspx?PRID=

n
4. Nag: 3rd generation anti-tank missile. 1784136 o
5. Akash: Medium range surface to air
l.
missile.
a

Reference: Q.45) Ans: B


ri

https://indianexpress.com/article/india/drdo-tests- Exp:
nuclear-capable-agni-prime-missile-7679198/ ● Statement 1 is correct: Hydrogen is considered
te

a secondary source of energy, commonly


Q.44) Ans: B referred to as an energy carrier. It can be
a

Exp: stored physically as a gas or liquid. Coal is a


m

● Statement 1 is incorrect: the Defence mixture of two components i.e., Carbon based
c

Research and Development Organisation matter (the decayed remains of prehistoric


(DRDO) has successfully conducted a maiden vegetation), and Mineral matter (which comes
s

flight test of a new indigenously developed from the ground from which the coal is dug).
p

surface-to-surface missile ‘Pralay’. It is India's Coal (one of the Hydrocarbon Fuels) is one of
.u

first conventional quasi-ballistic missile and is the important sources of hydrogen making
an answer to any conventional missile attack apart from natural gas and renewable energy
w

from northern or western borders. A quasi- through Electrolysis.


w

ballistic missile has a low trajectory, and while ● Statement 2 is correct: Coal has not been
it is largely ballistic, it can maneuver in flight. encouraged in hydrogen production because
w

The missile has been developed in a way that it of the fear of Carbon Emission while extracting

DPP 2023 DAY 167 38


https://upscmaterial.online/
Download From - https://upscmaterial.online/

Contact us :info@onlyias.com

OnlyIAS Nothing Else Visit :dpp.onlyias.in


Contact : +91-7007 931 912

hydrogen via coal. Almost 100% of hydrogen Security, Accountability & Governance, and
produced in India is through natural gas (Grey Agency & Interoperability. The principles are
Hydrogen). Grey Hydrogen is the hydrogen not legally binding and are intended to be
extracted from natural gas using steam- aspirational.
methane reforming.
● Statement 3 is incorrect: The benefit of it is Reference: https://github.com/WEF-
that the cost of hydrogen produced from coal Blockchain/Presidio-Principles
can be cheaper and less sensitive to imports.
The global emphasis is on substituting liquid
fuels with hydrogen (as fuel in vehicles), Q.47) Ans: C
storage of surplus renewable power as Exp:
hydrogen (as power cannot be stored at a cost ● Statement 1 is incorrect: Metaverse is a
effective price), and cutting down emission. combination of multiple elements of
● Statement 4 is correct: Currently, much of the technology, including virtual reality,
hydrogen produced is used for oil refining augmented reality, and video where users
(33%), ammonia (27%), methanol production “live” within a digital universe. The metaverse
(11%), steel production (3%) and others. At is not a new idea, science fiction writer Neal
present, the current global demand for Stephenson coined the term in 1992, and the
hydrogen is 70 million metric tons, most of concept is commonplace among video game
which is being produced from fossil fuels– 76% companies. Metaverse is the next version of
from natural gas and 23% from coal and the Internet focused on social connection. It

e
remaining from the electrolysis of water. can be defined as a simulated digital

n
environment that uses Augmented Reality

li
Reference: (AR), Virtual Reality (VR), and blockchain, along
https://economictimes.indiatimes.com/topic/coal- with concepts from social media, to create

n
based-hydrogen spaces for rich user interaction mimicking the
o
real world.
l.
● Statement 2 is correct: Metaverse can be
a

Q.46) Ans: A imagined as a 3D virtual world, with ever-


ri

Exp: evolving aspects which are collectively shared


● Option A is correct: The “Presidio Principles: by its inhabitants - a virtual world with real-
te

Foundational Values for a Decentralized time events and an online infrastructure. In


Future” (the "Principles") is an initiative from theory, it encapsulates everything that’s
a

the World Economic Forum's Global happening into the real world and will bring
m

Blockchain Council intended to lay out a real-time events and updates going forward.
c

foundational set of principles for those The user exists in a virtual world without
building with blockchain technology and borders.
s

decentralized infrastructure. The document ● Statement 3 is correct: Metaverse provides


p

calls on all actors to uphold these tenets as the following opportunities:


.u

they build blockchain applications – and to self- 1. Virtual communities, activities, events,
direct their ecosystems in using these all seamlessly accessible without the
w

principles as a foundational vision for how need to sign into multiple apps.
w

users can and should be protected. The 2. For a user-centric approach, a key
Presidio Principles are grouped into four broad aspect of the Metaverse that will work
w

pillars: Transparency & Accessibility, Privacy & in its favour is the effortless transition

DPP 2023 DAY 167 39


https://upscmaterial.online/
Download From - https://upscmaterial.online/

Contact us :info@onlyias.com

OnlyIAS Nothing Else Visit :dpp.onlyias.in


Contact : +91-7007 931 912

from one to the next step without known as thermocouple. The natural decay of
discrepancies. One can sign into his plutonium-238 produces heat that is then
virtual office as a virtual avatar of transferred to one side of the thermocouple.
himself, meet a client, take a break or ● Statement 3 is correct: The Radioisotope
play a sport - virtually all in one place. Thermoelectric Generator can prove to be a
3. With the Covid-19 pandemic confining good alternative to solar energy in space as
us to our homes for work, the solar energy is not an option for space objects
Metaverse takes it to another level. It meant to operate on the dark sides of planets
facilitates seamless cross-platform where sunlight is obscured. They are used on
interaction with one’s friends, family, NASA missions where other options such as
colleagues across the world. solar power are impractical or incapable of
4. Cross-platform interaction is in its providing the power that a mission may need
infancy even in the gaming industry. to accomplish its scientific or operational
With the Metaverse, cross-platform goals.
interaction will be the gold standard
for seamless virtual interaction across Reference:
the globe. Addresses and pin codes https://solarsystem.nasa.gov/missions/cassini/radiois
won’t be a mandate anymore. otope-thermoelectric-generator/

Reference: Q.49) Ans: B


https://indianexpress.com/article/technology/tech- Exp:

e
news-technology/canadian-singer-grimes-calls-mark- ● Option B is correct:

n
zuckerberg-under-qualified-to-launch-the-metaverse- 1. Hormone therapy is a treatment that slows or

li
8147866/ stops the growth of breast and prostate
cancers that use hormones to grow. Hormone

n
therapy is also called hormonal therapy,
o
Q.48) Ans: D hormone treatment, or endocrine therapy.
l.
Exp: Because hormone therapy blocks your body’s
a

● Statement 1 is correct: RTG is a type of ability to produce hormones or interferes with


ri

Nuclear-based power system that is generally how hormones behave, it can cause unwanted
used for power generation and thermal side effects. The side effects you have will
te

management of space missions. Radioisotope depend on the type of hormone therapy you
Thermoelectric Generator (RTG). It provides receive and how your body responds to it
a

electrical power for spacecraft by converting 2. Photodynamic therapy uses a drug activated
m

the heat generated by the decay of plutonium- by light to kill cancer and other abnormal
c

238 (Pu-238) fuel into electricity using devices cells. Photodynamic therapy uses a drug that is
called thermocouples. Flight-proven systems activated by light, called a photosensitizer or
s

that provide power and heat to a spacecraft. photosensitizing agent, to kill cancer cells. The
p

RTGs were first used in space during the Cold light can come from a laser or other source,
.u

War in 1961 for the US’s Transit 4A Mission. such as LEDs. Photodynamic therapy is also
They are lightweight, compact spacecraft called PDT. Photodynamic therapy is most
w

power systems that are extraordinarily often used as a local treatment, which means
w

reliable. it treats a specific part of the body.


● Statement 2 is incorrect: RTG converts thermal 3. The Chimeric Antigen Receptor T-cell (CAR-T)
w

energy into electrical energy though the device therapy has emerged as a breakthrough in

DPP 2023 DAY 167 40


https://upscmaterial.online/
Download From - https://upscmaterial.online/

Contact us :info@onlyias.com

OnlyIAS Nothing Else Visit :dpp.onlyias.in


Contact : +91-7007 931 912

cancer treatment. It is a way to get immune coordinate, guide and oversee the
cells called T cells (a type of white blood cell) to implementation and future development of
fight cancer by editing them in the lab so they IPRs in India. The ‘Cell for IPR Promotion &
can find and destroy cancer cells. T cells are Management (CIPAM)’, setup under the aegis
taken from the patient's blood and are of DIPP, is to be the single point of reference
changed in the lab by adding a gene for a man- for implementation of the objectives of the
made receptor (called CAR). This helps them National IPR Policy.
better identify specific cancer cell antigens. ● Statement 3 is correct: India has climbed 2
The CAR-T cells are then given back to the spots and has been ranked 46th by the World
patient. It is also sometimes talked about as a Intellectual Property Organization (WIPO) in
type of cell-based gene editing, because it the Global Innovation Index 2021
involves altering the genes inside T cells to help rankings.India has been on a rising trajectory,
them attack the cancer. In order to promote over the past several years in the Global
and support development of CAR-T cell Innovation Index (GII), from a rank of 81 in
technology, 2015 to46in 2021.

Reference: Reference:
https://pib.gov.in/PressReleasePage.aspx?PRID=1725 https://pib.gov.in/PressReleasePage.aspx?PRID=1756
254 465
https://www.cancer.gov/about-
cancer/treatment/types Q.51) Ans: D

e
Exp:

n
● Statement 1 is correct: Instead of injecting a

li
Q.50) Ans: C weakened form of a virus or bacteria into the
Exp: body, Nucleic acid vaccines use genetic

n
● Statement 1 is correct: India is a member of material from a disease-causing virus or
o
the World Trade Organisation and committed bacterium (a pathogen) to stimulate an
l.
to the Agreement on Trade Related Aspects of immune response against it. That immune
a

Intellectual Property (TRIPS Agreement). India response, which produces antibodies, is what
ri

is also a member of World Intellectual Property protects us from getting infected if the real
Organization, a body responsible for the virus enters our bodies.
te

promotion of the protection of intellectual ● Statement 2 is correct: Advantages of mRNA


property rights throughout the world. India’s acid vaccines are as following:
a

IPR regime is in compliance with the WTO's 1. are considered safe as mRNA is non-
m

agreement on Trade-Related Aspects of infectious, non-integrating in nature,


c

Intellectual Property Rights (TRIPS). and degraded by standard cellular


● Statement 2 is incorrect: The National mechanisms.
s

Intellectual Property Rights (IPR) Policy 2016 2. are highly efficacious because of their
p

was adopted in May 2016 as a vision document inherent capability of being


.u

to guide future development of IPRs in the translatable into the protein structure
country. Its clarion call is “Creative India; inside the cell cytoplasm.
w

Innovative India”. Department of Industrial 3. Additionally, mRNA vaccines are fully


w

Policy & Promotion (DIPP), Ministry of synthetic and do not require a host for
Commerce, Government of India, has been growth, e.g., eggs or bacteria.
w

appointed as the nodal department to Therefore, they can be quickly

DPP 2023 DAY 167 41


https://upscmaterial.online/
Download From - https://upscmaterial.online/

Contact us :info@onlyias.com

OnlyIAS Nothing Else Visit :dpp.onlyias.in


Contact : +91-7007 931 912

manufactured inexpensively to ensure to anyone through close, personal, often skin-


their "availability" and "accessibility" to-skin contact, Hugging, massage, and kissing,
for mass vaccination on a sustainable Prolonged face-to-face contact and sex.
basis.
● Statement 3 is correct: ZyCov-D would be the Reference:
world’s first DNA vaccine against COVID-19. It https://indianexpress.com/article/explained/explaine
is the first indigenously developed COVID d-what-is-monkeypox-a-smallpox-like-disease-from-
vaccine. It has been developed in partnership africa-that-has-been-reported-in-the-uk-7907071/
with the Department of Biotechnology under
the ‘Mission COVID Suraksha'. This DNA-based
vaccine can be administered without a needle. Q.53) Ans: D
It can be administered to adults as well as Exp:
those 12 and above. ● Statement 1 is Incorrect: -
○ WHO defines rare disease as often
Reference: debilitating lifelong disease or
https://www.ncbi.nlm.nih.gov/pmc/articles/PMC1746 disorder with a prevalence of 1 or
32/ less, per 1000 population. However,
different countries have their own
definitions to suit their specific
Q.52) Ans: B requirements and in context of their
Exp: own population, health care system

e
● Statement 1 is correct: Monkeypox is caused and resources.

n
by monkeypox virus, a member of the ○ There is a lack of awareness about rare
diseases in the general public as well as

li
Orthopoxvirus genus in the family Poxviridae.
Monkeypox is usually a self-limited disease in the medical fraternity. Many

n
with the symptoms lasting from 2 to 4 weeks. doctors lack appropriate training and
o
Severe cases can occur. awareness to be able to correctly and
l.
● Statement 2 is incorrect: Monkeypox is a viral timely diagnose and treat these
a

zoonosis (a virus transmitted to humans from conditions. Availability and access to


ri

animals) with symptoms very similar to those medicines are important to reduce
seen in the past in smallpox patients, although morbidity and mortality associated
te

it is clinically less severe. with rare diseases.


● Statement 3 is correct: Monkeypox is a ● Statement 2 is incorrect:-
a

zoonosis, that is, a disease that is transmitted ○ Following a demand from activists, the
m

from infected animals to humans. According to Union Health Ministry has increased
the financial support from 20 lakh to
c

the WHO, cases occur close to tropical


rainforests inhabited by animals that carry the 50 lakh for treatment of patients
s

virus. Monkeypox virus infection has been suffering from rare diseases under the
p

detected in squirrels, Gambian poached rats, umbrella scheme of Rashtriya Arogya


.u

dormice, and some species of monkeys. Nidhi (RAN).


Human-to-human transmission can be through ○ Rashtriya Arogya Nidhi is a Scheme
w

contact with bodily fluids, lesions on the skin or which provides financial assistance to
w

on internal mucosal surfaces, such as in the patients, living below poverty line
mouth or throat, respiratory droplets and (BPL) and who are suffering from
w

contaminated objects. Monkeypox can spread major life threatening diseases, to

DPP 2023 DAY 167 42


https://upscmaterial.online/
Download From - https://upscmaterial.online/

Contact us :info@onlyias.com

OnlyIAS Nothing Else Visit :dpp.onlyias.in


Contact : +91-7007 931 912

receive medical treatment at any of a patent over a product by introducing


the super specialty Government minor changes to it.
hospitals / institutes. ○ It has been an issue of concern for
pharmaceutical companies . Ex:
Additional Information Novartis issue on the issue of patent
The policy has categorised rare diseases in three of cancer drug Glivec.
groups: ○ In 2006, the Indian Patent Office
● Group 1: Disorders amenable to one-time rejected Novartis patent application
curative treatment. for Glivec under Section 3(d) of the
● Group 2: Those requiring long term or lifelong Indian Patents Act, stating that the
treatment. drug was a modification of an existing
● Group 3: Diseases for which definitive substance, imatinib, and therefore
treatment is available but challenges are to represented a case of ‘evergreening’.
make optimal patient selection for benefit,
very high cost and lifelong therapy. Q.55) Ans: D
Exp:
Source:
https://indianexpress.com/article/opinion/columns/a ● Statement 1 is Incorrect: - World’s first
nother-chance-at-life-7611890/ genetically modified (GM) rubber sapling was
https://main.mohfw.gov.in/sites/default/files/Final%2 planted at Rubber Board’s Sarutari Research
0NPRD%2C%202021.pdf farm on the outskirts of Guwahati in Assam.

e
The GM rubber plant, which is the first of its
kind developed specifically for the northeast

n
and is expected to flourish in the region's

li
Q.54) Ans: B
Exp: climatic conditions. The plant was developed

n
● Statement 1 is Correct: - The Compulsory at the Kerala-based Rubber Research Institute
licensing (CL) enables a competent of India (RRII).
o
l.
government authority to licence the use of a ● Statement 2 is incorrect:- With additional
a

patented invention to a third party or copies of the gene MnSOD (manganese-


containing superoxide dismutase) inserted in
ri

government agency without the consent of


the patent holder. Section 92 of the Patent it, the GM rubber is expected to tide over the
te

Act, 1970, provides for issuing CL by the central severe cold conditions during winter, which is
government. a major factor affecting the growth of rubber
a

● Statement 2 is Incorrect : - Compulsory saplings. Natural rubber is a native of warm


m

licensing is permitted under the WTO’s TRIPS humid Amazon forests and is not naturally
(IPR) Agreement) in circumstances of “national suited for the colder conditions in the
c

emergency or in circumstances of extreme Northeast, which is one of the largest


s

urgency or in case of public non-commercial producers of rubber in India. Growth of young


p

use. rubber plants remains suspended during the


.u

● Statement 3 is Correct: - winter months, which are also characterised by


○ Evergreening of patents deals with progressive drying of the soil. This is the reason
w

renewal of patents with minor for the long immaturity period of this crop in
w

changes. Section 3(d) Indian Patent the region. The MnSOD gene has the ability to
Act 1970 doesn’t allow the renewal of protect plants from the adverse effects of
w

DPP 2023 DAY 167 43


https://upscmaterial.online/
Download From - https://upscmaterial.online/

Contact us :info@onlyias.com

OnlyIAS Nothing Else Visit :dpp.onlyias.in


Contact : +91-7007 931 912

severe environmental stresses such as cold and comes from ruminants (cows and
drought. sheep).
Source: ○ Industrially produced trans-fat (have a
https://www.thehindu.com/news/national/other- longer shelf life and are cheaper) are
states/first-ever-genetically-modified-rubber-planted- found in hardened vegetable fats such
in-assam/article34901294.ece as margarine and are often present in
snack foods, baked goods and fried
foods.
Q.56) Ans: B ● Statement 2 is Incorrect: -
Exp:

● Statement 1 is Incorrect: - Synthetic biology


refers to the science of using genetic
sequencing, editing, and modification to create
unnatural organisms or organic molecules that
can function in living systems (Not for Non- ○ WHO has launched a comprehensive
living systems). Synthetic biology enables plan to eliminate industrially-
scientists to design and synthesise new produced artificial trans fats from the
sequences of DNA from scratch. global food supply by 2023 called
● Statement 2 is Correct: -In the pharmaceutical REPLACE.
industry it can be used to make natural ○ Intake of TFA is associated with

e
compounds such as artemisinin used for the increased risk of heart attacks and
treatment of malaria and Car T cell therapy

n
death from coronary heart disease.
for cancer treatment.

li
● Statement 3 is Incorrect: A draft foresight ● Trans fatty acids, more commonly called trans

n
paper on synthetic biology has been released fats, are made by heating liquid vegetable oils
by the Department of Biotechnology has
o
in the presence of hydrogen gas and a catalyst,
l.
stressed the need for a national policy that can a process called hydrogenation. Partially
a

consolidate India’s stand on the issue. National hydrogenating vegetable oils makes them
policy needed for synthetic biology, says DBT
ri

more stable and less likely to become rancid.


paper. This process also converts the oil into a solid,
te

which makes them function as margarine or


Source: shortening. Partially hydrogenated oils can
a

https://indianexpress.com/article/cities/delhi/nationa withstand repeated heating without breaking


m

l-policy-needed-for-synthetic-biology-says-dbt-paper- down, making them ideal for frying fast foods.


7777678/
c

For these reasons, partially hydrogenated oils


became a mainstay in restaurants and the food
s

Q.57) Ans: D industry – for frying, baked goods, and


p

Exp:
processed snack foods and margarine. Partially
.u

● Statement 1 is Incorrect: -
hydrogenated oil is not the only source of trans
○ Trans-fat or trans-fatty acids are
fats in our diets. Trans fats are also naturally
w

unsaturated fatty acids that come


found in beef fat and dairy fat in small
from either natural or industrial
w

amounts. Trans fats are the worst type of fat


sources. Naturally occurring trans-fat
for the heart, blood vessels, and rest of the
w

body.

DPP 2023 DAY 167 44


https://upscmaterial.online/
Download From - https://upscmaterial.online/

Contact us :info@onlyias.com

OnlyIAS Nothing Else Visit :dpp.onlyias.in


Contact : +91-7007 931 912

Source: Q.59) Ans: A


Exp:
https://www.thehindu.com/news/national/af
ter-oils-fssai-caps-transfats-in- ● Statement 1 is correct: -
foods/article61751295.https://www.hsph.har ○ Additive manufacturing (AM) is a
vard.edu/nutritionsource/what-should-you- process in which a three dimensional
eat/fats-and-cholesterol/types-of-fat/ object is built from a computer aided
design (CAD) model, usually by
successively adding materials in a
layer-by-layer fashion. products with
Q.58) Ans: D
complex features, which could not
Exp:
easily have been produced via
● Statement 1 is Incorrect: - Soyameal is a subtractive or other traditional
protein rich solid leftover raw material after manufacturing processes.
extracting oil from soyabean seed. It is a major ○ Subtractive manufacturing involves
ingredient of poultry feed. India will import removing parts of a material involved
genetically modified (GM) soyameal for the in traditional manufacturing methods.
first time to be used as livestock feed. ○ The AM market globally focused on
● Statement 2 is Incorrect: - Ministry of the sectors including automotive,
Environment, Forest, and Climate Change consumer products, medical, business
(MoEFCC) cleared the proposal, As, it belongs machines, aerospace,

e
to the non-living nature of the material and government/military, academic and

n
hence, Genetic Engineering Appraisal others.
● Statement 2 is Incorrect: - National Strategy on

li
Committee (GEAC) was passed. The
Additive Manufacturing was released by the

n
environment ministry clarified since the de-
oiled cake per se obtained after crushing GM Ministry of Electronics & Information
o
soyabean does not contain any living modified Technology (MeitY). There are following
l.
organism, it has no objection to import of soya features of the policy -
a

cake or meal from an environmental angle. The ○ Addition to the GDP: It hopes that it
ri

official said there are three types of GM foods, could likely add $ 1 billion to the gross
domestic product by that time.
te

one with Genetically Modified Organism


(GMO), second with Living Modified Organism ○ Employment opportunities: These
new start-ups and opportunities will
a

(LMO) and third with Non-Living Modified


Organism (NLMO). "The soyameal is NLMO. give jobs to at least 1 lakh new skilled
m

The Environment Ministry has said it has no workers over the next three years.
○ Overcome technical and economic
c

negative impact on human and animal health


barriers: IT ministry is readying a
s

as well as on the environment.


national policy on promoting 3D
p

Source: printing on an industrial scale and


.u

https://www.thehindu.com/news/national/ helping domestic companies


overcome technical and economic
w

centre-allows-import-of-gm-soymeal-to-
support-poultry- barriers so that they can build
w

industry/article35901075.ece supportive and ancillary facilities for


world leaders in the technology, such
w

as the US and China.

DPP 2023 DAY 167 45


https://upscmaterial.online/
Download From - https://upscmaterial.online/

Contact us :info@onlyias.com

OnlyIAS Nothing Else Visit :dpp.onlyias.in


Contact : +91-7007 931 912

○ The government would address key batteries have no memory effect, a


sector specific technical challenges for detrimental process where repeated partial
making 3D printing economically discharge/charge cycles can cause a battery to
viable for MSMEs. ‘remember’ a lower capacity. This is an
advantage over both Ni-Cd and Ni-MH, which
Source: display this effect
https://www.thehindu.com/news/cities/Hyde
rabad/city-in-race-to-get-national-centre-for- Source:
additive-manufacturing/article33804568.ece
https://www.meity.gov.in/content/national- https://indianexpress.com/article/india/india-china-
strategy-additive-manufacturing relation-argentina-lithium-reserves-economic-growth-
rate-7131626/

Q.60) Ans: B
Exp: Q.61) Ans: D
● Statement 1 is Correct: - It is lightweight and Exp:
has high energy density i.e., stores more ● Statement 1 is incorrect: -
energy per unit of weight. They also have a ○ It is a colourless, odourless, tasteless,
high power-to-weight ratio, high energy non-toxic and highly combustible
efficiency and good high-temperature gaseous substance.
performance. Most components of lithium-ion ○ Hydrogen is the lightest, simplest and
is the most abundant chemical

e
batteries can be recycled, but the cost of
substance in the universe
material recovery remains a challenge for the

n
○ Although hydrogen is the most
industry. abundant element in the universe

li
● Statement 2 is Incorrect: - Li-ion batteries also (three times as abundant as helium,

n
have low self-discharge rate of around 1.5-2% the next most widely occurring
per month. They do not contain toxic
o
element), it makes up only about 0.14
percent of Earth’s crust by weight.
l.
cadmium, which makes them easier to dispose
○ It occurs, however, in vast quantities
a

of than Ni-Cd batteries.


as part of the water in oceans, ice
● Statement 3 is Incorrect: - Lithium metal is
ri

packs, rivers, lakes, and the


extremely reactive and can potentially short- atmosphere. As part of innumerable
te

circuit the cell. Thermal runaway occurs due to carbon compounds, hydrogen is
melting of the electrolyte, high operational present in all animal and vegetable
a

temperature, poor quality of the battery cells tissue and in petroleum.


m

● Statement 2 is Incorrect: - Hydrogen gas


and battery pack assemblies, and lack of
generated through Electrolysis is called Green
active cells assemblies. Thermal runaway
c

Hydrogen. Electrolysis employs an electric


leads to catching fire.
s

current to split water into hydrogen and


● Statement 4 is Correct: Li-ion battery cells can
p

oxygen in an electrolyzer. If the electricity is


deliver up to 3.6 Volts, 3 times higher than
produced by renewable power, such as solar or
.u

technologies such as Ni-Cd or Ni-MH. This


wind, the resulting pollutant-free hydrogen is
means that they can deliver large amounts of
w

called green hydrogen.


current for high-power applications, which has
Li-ion batteries are also comparatively low
w

maintenance, and do not require scheduled


w

cycling to maintain their battery life. Li-ion

DPP 2023 DAY 167 46


https://upscmaterial.online/
Download From - https://upscmaterial.online/

Contact us :info@onlyias.com

OnlyIAS Nothing Else Visit :dpp.onlyias.in


Contact : +91-7007 931 912

explainsnuclear-fusion-reactions

Q.63) Ans: A
Exp:
● Statement 1 is Correct:- India's three-stage
nuclear power programme was formulated by
● Statement 3 is Correct: - Homi Bhabha, the well-known physicist, in the
○ India’s first 99.999% pure Green 1950s to secure the country's long term energy
Hydrogen pilot plant has been independence, through the use of uranium and
commissioned by the Oil India Limited thorium reserves found in the monazite sands
(OIL) at its Jorhat Pump Station in of coastal regions of South India. India is
currently on the second stage of its ambitious
Assam. The plant has an installed
nuclear program. The country is planning to
capacity of 10 kg per day. construct 12 new nuclear power reactors by
○ The plant produces Green Hydrogen 2024
from the electricity generated by a ● Statement 2 is Correct:- India's 1962 Atomic
500kW Solar plant using a 100 kW Energy Act says nothing about liability or
Anion Exchange Membrane (AEM) compensation in the event of an accident. Also,
India was not a party to the relevant
Electrolyser array. The use of AEM
international nuclear liability conventions (the
technology is being used for the first
IAEA's 1997 Amended Vienna Convention and
time in India. 1997 Convention on Supplementary
Compensation for Nuclear Damage – CSC).

e
Source :- https://www.thehindu.com/sci-tech/energy- ● Statement 3 is Incorrect:- The plutonium-239

n
and-environment/green-hydrogen-fuel-of-the- breeder reactor is commonly called a fast

li
future/article65463098.ece breeder reactor, and the cooling and heat
transfer is done by a liquid metal.Fast breeder

n
reactor does not use moderator, while liquid
Q.62) Ans: C sodium is used as a Coolant.
o
Exp:
l.
● Statement 1 is Incorrect:- Fusion process
a

produces several times greater energy than


Source:-
ri

fission and generates minimal waste. Fusion


doesn't produce harmful long-term https://www.thehindu.com/news/national/telangana
te

radioactive waste as a by-product like fission /nuclear-energy-can-meet-indias-energy-


does. A fusion reactor produces helium, which needs/article65507022.ece
a

is an inert gas. https://world-nuclear.org/information-


m

● Statement 2 is correct:- Fusion is among the library/country-profiles/countries-g-n/india.aspx


most environmentally friendly sources of
energy. There are no CO2 or other harmful
c

atmospheric emissions from the fusion


s

process, which means that fusion does not Q.64) Ans: D


p

contribute to greenhouse gas emissions or Exp:


.u

global warming. ● Both Statement 1 and Statement 2 are


● Statement 3 is Incorrect:- It is difficult to Incorrect: - Neutrinos are tiny, neutral,
w

sustain the Fusion process for long periods of elementary particles which interact with
time due to the tremendous amount of matter with the weak force via the weak
w

pressure and temperature needed.


interaction and gravity. The weakness of this
force gives neutrinos the property that matter
w

Reference: - https://www.energy.gov/science/doe-
is almost transparent to them. The Sun, and all
DPP 2023 DAY 167 47
https://upscmaterial.online/
Download From - https://upscmaterial.online/

Contact us :info@onlyias.com

OnlyIAS Nothing Else Visit :dpp.onlyias.in


Contact : +91-7007 931 912

other stars, produce neutrinos copiously due nano nitrogen particles (from 20-50 nm).
to nuclear fusion and decay processes within ● Statement 2 is Incorrect:- It is sprayed directly
their core. Since they rarely interact, these on the leaves of plants and gets absorbed by
the stomata- pores found on the epidermis of
neutrinos pass through the Sun, and even the
leaves. 2-4 ml of nano urea should be mixed a
Earth, unhindered. . There are many other litre of water and sprayed on crop leaves at
natural sources of neutrinos including active growth stages. Liquid nano urea
exploding stars (supernovae), relic neutrinos contains 4 per cent total nitrogen (w/v) evenly
(from the birth of the universe), natural dispersed in water. The size of a nano nitrogen
radioactivity, and cosmic ray interactions in the particle varies from 20-50 nm.
atmosphere of the Earth. For example, the Sun ● Statement 3 is incorrect:- It has higher shelf
life & higher efficiency as compared to
produces over two hundred trillion trillion
conventional Urea. Being in liquid state is
trillion neutrinos every second, and a advantageous for the nano urea as it will avoid
supernova blast can unleash 1000 times more ‘caking’ of fertilisers.
neutrinos than our Sun will produce in its 10- It has also other many benefits, such as -
billion year lifetime. Billions of neutrinos I. Improved agricultural produce with
stream through our body every second, yet reduced urea consumption as the
only one or two of the higher energy neutrinos efficiency of liquid nano urea is as high
as 85-90% in comparison to 25% of
will scatter from you in your lifetime.
conventional urea.
II. Significant reduction in urea imports,
Additional Information government subsidies and logistics
● Indian Neutrino Observatory is a multi- cost.

e
institutional effort aimed at building a world- III. Reduced soil, water and air pollution
from urea, improving quality of

n
class underground laboratory and an Iron
underground water and reduction in

li
Calorimeter (ICAL) detector for studying
global warming.
neutrinos.

n
IV. Higher shelf life, as no issue of caking
● It aims to assess the properties of neutrino on coming into contact with moisture.
o
particles and to make precision measurements Caking occurs when fertiliser particles
l.
of the parameters related to neutrino create contact points with each other
a

oscillations. that are typically formed by salt


bridges, adhesion, or surface diffusion
ri

● It is jointly funded by the Dept. of Atomic


from porous granules. A product with
Energy and the Dept. of Science and
te

a high amount of dust can lead to


Technology. caking when the dust comes into
a

contact with moisture, such as through


Reference- ambient humidity.
m

https://www.scientificamerican.com/article/what-is-
c

a-neutrino/
Source:
s

https://indianexpress.com/article/explained/everyday
p

-explainers/pm-modi-gujarat-visit-india-first-nano-
urea-plant-kalol-farms-iffco-7946163/
.u

Q.65) Ans: D https://theprint.in/india/what-is-nano-urea-indias-


Exp: 21st-century-product-aiming-to-revolutionise-world-
w

● Statement 1 is Incorrect:- Liquid nano Urea is agriculture/673151/


w

a patented chemical nitrogen fertiliser


developed by Indian Farmers Fertiliser PYQ Reference: Q. No. - 91 Set A (UPSC 2020)
w

Cooperative Limited IFFCO’s Nano


Biotechnology Research Centre at Kalol with
DPP 2023 DAY 167 48
https://upscmaterial.online/
Download From - https://upscmaterial.online/

Contact us :info@onlyias.com

OnlyIAS Nothing Else Visit :dpp.onlyias.in


Contact : +91-7007 931 912

Q.66) Ans: B immediate effect, the import of drones in


Exp: Completely-Built-Up, Semi-knocked-down or
● Statement 1 is Incorrect:- Conversion of a Completely-Knocked-down forms. Import of
material at nanoscale results in alternation of drones for R&D, defence and security purposes
Physicochemical, Biochemical, Optical,
have been exempted from the ban but such
Electronic and mechanical properties. The
properties of materials can be different on a imports will require due clearances.
nanoscale for two main reasons - ● Statement 2 is Correct: - Drones have been
○ Nanomaterials have, relatively, a classified based upon the maximum all-up
larger surface area than the same weight including payload There are total five
mass of material produced in a larger times of drones -
form.
○ Below 50 nm, the laws of classical
physics give way to quantum effects, Types of Drones Load (Drones Plus
provoking optical, electrical and Weight)
magnetic behaviours different from
those of the same material at a larger Nano <= 250g
scale.
Micro 250g - 2kg
● Statement 2 is Correct: - Recently, Scientists
used Machine Learning to predict the match of
Small 2 kg - 25 kg
pairs of metals that can form bimetallic
nanoalloys. Medium 25 kg - 150 kg
❖ Nano alloys, also called core-shell
nanocluster alloys, are alloys with one Large >150 kg

e
metal forming core and another stays

n
on the surface as a shell.
● Statement 3 is Correct: - Drones rule shall not
❖ The formation depends on the

li
cohesive energy difference, atomic apply to drones used by the naval, military or

n
radius difference, surface energy Air Force of India
difference and electronegativity of the Source: - o
two atoms.
l.
https://www.thehindu.com/news/national/ministry
❖ Nanoalloys can have other -of-civil-aviation-notifies-drone-rules-2021-aim-to-
a

configurations as well such as help-start-ups-smes/article36114141.ece


ri

homogeneously mixed alloy and


cluster-in-cluster
te

Reference - Q.68) Ans: D


a

https://www.ncbi.nlm.nih.gov/pmc/articles/PMC5905 Exp:
m

● Statement 1 is Incorrect: - Computer storage


289/
and memory capacities are expressed in units
c

called bits and bytes. A bit is the smallest unit


Q.67) Ans: C
s

of measurement for digital information in


Exp:
p

computing. A byte is the number of bits a


.u

● Statement 1 is Incorrect:- Import of drones particular computing architecture needs to


and drone components shall be regulated by store a single text character. Consequently, the
w

the Directorate General of Foreign Trade. The number of bits in a byte can differ between
Directorate General of Foreign Trade or DGFT computing platforms. Supercomputers are
w

under the Ministry of Commerce and Industry large systems that are specifically designed to
w

issued an order which prohibited, with solve complex scientific and industrial
challenges. The performance capabilities of
DPP 2023 DAY 167 49
https://upscmaterial.online/
Download From - https://upscmaterial.online/

Contact us :info@onlyias.com

OnlyIAS Nothing Else Visit :dpp.onlyias.in


Contact : +91-7007 931 912

supercomputers are expressed using a Internet opportunities and address risks and
standard rate for indicating the number of challenges that arise.
floating-point arithmetic calculations systems ● Statement 2 is Correct: - India Internet
can perform on a per-second basis. The rate, Governance Forum (IIGF) has been constituted
floating-point operations per second, is in conformance to the Tunis Agenda of UN
abbreviated as FLOPS. Internet Governance Forum (IGF). IIGF
● Statement 2 is Incorrect:- facilitates discussion between
○ NSM, launched in 2015, is being intergovernmental organisations, private
steered jointly by the Ministry of companies, civil society organisations etc. who
Electronics & Information Technology all deal with or are involved in Internet
and Department of Science and governance related public policy issues.
Technology and implemented by C-
DAC and Indian Institute of Science, Source:-
Bangalore. https://www.pib.gov.in/PressReleasePage.aspx?PRI
○ Mission envisages empowering our D=1765476
national academic and R&D
institutions spread over the country by
installing a vast supercomputing grid Q.70) Ans: A
comprising more than 70 high- Exp:
performance computing (HPC) ● Statement 1 is Incorrect: -
facilities. ❖ Project tiger is a Centrally Sponsored

e
Scheme of the Ministry of
Environment, Forests and Climate

n
Source: -
Change which provides central

li
assistance to tiger range States for in-
https://www.thehindu.com/news/cities/bangalore/iis

n
situ (not ex-situ) conservation of
c-commissions-one-of-indias-most-powerful- tigers in designated tiger reserves.
o
supercomputers/article38369811. ❖ It was initially launched in 1973 in 9
l.
https://kb.iu.edu/d/apeq Tiger reserves (TRs)
❖ Implementing Agency is National Tiger
a

Conservation Authority (NTCA) which


ri

was established through WildLife


te

(Protection) Amendment Act, 2006 to


Q.69) Ans: B provide statutory authority to Project
Exp:
a

Tiger.
● Statement 1 is Incorrect: - Internet ❖ Funding pattern: Centre provides
m

Governance Forum (IGF) is a UN-based multi- financial assistance to States of 60%


stakeholder platform bringing representatives and 50% for expenditure on all
c

together from various groups to discuss public nonrecurring items and expenditure
s

policy issues related to the Internet. IGF on recurring items respectively.


p

● Statement 2 is Correct: - The tiger inhabited


informs and inspires those with policy-making
.u

countries pledged to double the number of


power in both the public and private sectors. tigers by 2022 (Chinese Year of the Tiger)
At their annual meeting delegates discuss,
w

under the St.Petersburg Declaration. There are


exchange information and share good a total of tigers inhabiting 13 countries –
w

practices with each other. The IGF facilitates a Bangladesh, Bhutan, Cambodia, China, India,
common understanding of how to maximize Indonesia, Lao PDR, Malaysia, Myanmar,
w

Nepal, Russia, Thailand, and Vietnam. They

DPP 2023 DAY 167 50


https://upscmaterial.online/
Download From - https://upscmaterial.online/

Contact us :info@onlyias.com

OnlyIAS Nothing Else Visit :dpp.onlyias.in


Contact : +91-7007 931 912

have formed the Global Tiger Initiative (GTI) . Q.72) Ans: D


● Statement 3 is Incorrect:- CATS are a globally Exp:
accepted conservation tool that sets best ● Statement 1 is correct: BECs have been
practices and standards to manage tigers and
produced in a variety of experiments on Earth
assessments to benchmark progress. It is not
related to translocation of Tigers from one since 1995, but these are hindered by gravity,
tiger reserve to another tiger reserve which which collapses the clouds in a split second. To
are decided under the guidelines of National make a BEC, scientists must first corral and
Tiger Conservation Authority (NTCA). At then supercool atoms. In the near-zero gravity
present, 14 out of India’s 51 tiger reserves are in space, they can mix the ingredients in a
CATS accredited. much smaller catchment “trap.” On Earth’s
surface, the atoms begin to repel each other
Source:
and fly apart almost instantaneously. This
https://frontline.thehindu.com/environment/conserv exotic material only exists when atoms of
ation/india-at-75-epochal-moments-1973-project- certain elements are cooled to temperatures
tiger/article65721318.ece near absolute zero.
● Statement 2 is correct: NASA scientists on
PYQ Reference: Q. No. - Set A (UPSC 2020)
Earth have collaborated with astronauts on the
International Space Station (ISS) to corral the
Q.71) Ans: C first ever Bose-Einstein condensate (BEC)- the
Exp: fifth state of matter- outside of Earth’s gravity.
● Statement 1 is Incorrect: - Central Ground The matter has been created in one of the

e
Water Board (CGWB) is a subordinate office coldest places in the universe- the Cold Atom

n
(Not a statutory body) of the Ministry of Jal Laboratory– a device on board the
Shakti (Department of Water Resources, River

li
International Space Station (ISS).
Development and Ganga Rejuvenation),
● Statement 3 is correct: There are four natural

n
Government of India.
● Statement 2 is Incorrect: - states of matter: Solids, liquids, gases and
o
❖ It is a National Apex Agency entrusted plasma. The fifth state is the man-made Bose-
l.
with the responsibilities of providing Einstein condensates. A Bose-Einstein
a

scientific inputs for management, condensate is so named because its existence


ri

exploration, monitoring, assessment, was posited almost a century ago by Albert


augmentation and regulation of Einstein and Indian mathematician Satyendra
te

groundwater resources of the Nath Bose. This exotic material only exists
country.
when atoms of certain elements are cooled to
a

❖ Central Ground Water Authority (a


temperatures near absolute zero. At that
m

Statutory body) has been entrusted


with the responsibility of regulating point, clusters of atoms begin functioning as a
c

and controlling ground water single quantum object with both wave and
development and management in the particle properties. It is used to study quantum
s

country. It has been established under mechanics on a macroscopic level.


p

the Environmental (Protection) Act - https://physicsworld.com/a/bose-einstein-


.u

1986.
condensation/
Source:
w

https://www.thehindu.com/news/national/tamil-
nadu/cgwa-notice-on-groundwater-extraction-does-
w

not-apply-to-tn/article65608236.ece Q.73) Ans: C


http://cgwb.gov.in/aboutcgwb.html Exp:
w

DPP 2023 DAY 167 51


https://upscmaterial.online/
Download From - https://upscmaterial.online/

Contact us :info@onlyias.com

OnlyIAS Nothing Else Visit :dpp.onlyias.in


Contact : +91-7007 931 912

● Statement 1 is incorrect: The Hasdeo Aranya Company (APMDC) and Chhattisgarh State
forests are called the lungs of Chhattisgarh. Power Generation Company (CSPGC)
The Hasdeo Aranya Region (Aranya means respectively, clearances were withdrawn.
forest) lies in the catchment area of the Reference:
Hasdeo river in North-Central Chhattisgarh. https://www.thehindu.com/news/explained-the-coal-
The Hasdeo River is a tributary of the mining-protests-in-the-hasdeo-aranya-
Mahanadi river which originates in region/article65726814.ece
Chhattisgarh and flows through Odisha into
the Bay of Bengal. The Hasdeo forests are also
the catchment area for the Hasdeo Bango Dam Q.74) Ans: D
built across the Hasdeo river which irrigates six Exp:
lakh acres of land, crucial to a State with paddy ● Option D is correct: The Peninsular Rock
as its main crop. Agama (Psammophilus dorsalis) is a type of
● Statement 2 is correct: The forests are garden lizard that has a strong presence in
ecologically sensitive due to the rich southern India. This lizard is a large animal,
biodiversity and presence of a large migratory strikingly coloured in orange and black. They
corridor for elephants (Lemru Elephant do not generate their own body heat, so they
Reserve) stretching from supporting the need to seek warmth from external sources
migration of wild elephants from Gumla like a warm rock or a sunny spot on the wall.
district in Jharkhand to Korba district of Recently, a study has been carried out by
Chhattisgarh. It lies on top of the Hasdeo researchers from Indian Institute of Science

e
coalfield, which represents one of the largest (IISc), Bengaluru to understand several

n
coal reserves in India, having estimated environmental factors (including urbanization)

li
reserves of 5.18 billion tonnes of coal. It is one that could affect the presence of the
of the largest intact forest areas in Central Peninsular Rock Agama/ South Indian Rock

n
India outside of the protected area system. Agama. It is majorly found in India (Asia).
o
● Statement 3 is correct: In the year 2010, the Indian states of Tamil Nadu, Chhattisgarh,
l.
Center categorized Hasdeo Aranya to be a Kerala, Andhra Pradesh, Karnataka, Bihar hosts
a

“no-go” zone for mining. Hasdeo represents the population of the lizard. Rock Agama can
ri

one of the largest coal reserves in India, having indicate which parts of the city are warming,
estimated reserves of 5.18 billion tonnes of and their numbers show how the food web is
te

coal. There are more than 20 known coal mines changing. These lizards eat insects and are in
on the Hasdeo reserve. After a joint study in turn eaten by raptors, snakes and dogs, they
a

2010, the Ministry of Coal and the Ministry of cannot live in places where there are no
m

Forest and Environment deemed the Hasdeo insects. Insects are critical components of a
c

reserve a ‘No Go Area,’ prohibiting any sort of healthy ecosystem as they provide many
mining due to its rich biodiversity and services, including pollination. Hence, the
s

exceptional ecology. Interestingly, the Parsa presence of rocky agamas presents a good
p

East Kente Basan coal mine was permitted model system to understand other aspects of
.u

within a year after the prohibition, based on a the ecosystem.


goal of extracting as much coal as possible at Reference: https://www.thehindu.com/sci-
w

the lowest possible cost. After the gram sabhas tech/study-of-rock-agama-gives-insights-into-


w

opposed mining in the Madanpur South and urbanisation-conservation/article65728840.ece


Gidmedi Paturia blocks that were allotted to
w

the Andhra Pradesh Mineral Development Q.75) Ans: B

DPP 2023 DAY 167 52


https://upscmaterial.online/
Download From - https://upscmaterial.online/

Contact us :info@onlyias.com

OnlyIAS Nothing Else Visit :dpp.onlyias.in


Contact : +91-7007 931 912

Exp: decalcification. Major Effects of pollution in


● Statement 1 is incorrect: Loktak is the largest Loktak lake are:
freshwater lake in Manipur, Northeast India. 1. Calcium anomalies: There are signs of
Lake is one of the most popular tourist calcium anomalies in some of the
attractions in Manipur. The lake invites tourists molluscs and other aquatic life in the
from far and wide for its ethereal beauty. One lake.This is similar to the phenomenon
of the largest water bodies in Asia and its of coral bleaching in oceans, where
beauty, greenery, marine life, surrounded by rising sea surface temperature causes
blue mountains stand out. Loktak lake was organisms that live on corals to
initially designated as a wetland of disengage, thereby killing the corals
international importance under the Ramsar themselves.
Convention in 1990. Later it was also listed 2. Effect on Phumdis: The health of the
under the Montreux Record in 1993. The lake also affects the Phumdis (the
etymology of Loktak is Lok = “stream” and tak unique ‘floating islands’) of the Loktak
= “the end”. This ancient lake plays an lake. These islands are made of a mix
important role in the economy of Manipur. It of vegetation and soil. These coalesce
serves as a source of water for hydropower to form a thick mat that, for centuries,
generation, irrigation and drinking water have hosted huts and fishing
supply. settlements.
● Statement 2 is correct: The lake houses the Reference:
only floating national park in the world, the https://www.downtoearth.org.in/news/environment/

e
Keibul Lamjao National Park, which is the last manipur-govt-orders-floating-homestays-off-loktak-

n
refuge of the endangered brow-antlered deer locals-demand-rollback-84047

li
or sangai, Manipur's state animal. It is famous
for the phumdis floating over it. Phumdis are a

n
series of floating islands of entangled Q.76) Ans: D o
vegetation formed by the accumulation of Exp:
l.
organic debris and biomass with soil, exclusive ● Statement 1 is correct: The Godavari River is
a

to the Loktak Lake in Manipur. Its thickness the largest Peninsular River system, also
ri

varies from a few centimeters to two meters. known as the Dakshin Ganga. It originates in
The humus of phumdi is black in colour and Brahmagiri Mountain at Trimbakeshwar in
te

very spongy with a large number of pores. It Maharashtra (Nashik region) and empties into
floats with 4/5 parts under water. In addition, the Bay of Bengal in Andhra Pradesh. The
a

the lake shelters about 230 species of aquatic Godavari basin spans Maharashtra, Andhra
m

plants, 100 types of birds, and 400 species of Pradesh, Chhattisgarh, and Odisha, as well as
c

fauna like barking deer, sambar, and Indian minor sections of Madhya Pradesh,
python. Karnataka, and the UT of Puducherry (Yanam)
s

● Statement 3 is correct: Instead of Carbon having a total area of approximately 3 lakh


p

Dioxide, Nitrogen is a major pollutant. The pH Sq.km. The river is 1,465 km long and
.u

of the lake, as per measurements so far, varies considered the second-longest river in the
from 6.8-7.2 (ideally the pH of a healthy lake country (after the Ganges). The Godavari River
w

should be slightly below 7). However, studies drains about 10% of India’s total geographical
w

of ocean acidification have shown that even a area.


0.1 increase in pH can cause (harmful) ● Statement 2 is correct: The Godavari Water
w

Dispute Tribunal headed by Justice Bachawat

DPP 2023 DAY 167 53


https://upscmaterial.online/
Download From - https://upscmaterial.online/

Contact us :info@onlyias.com

OnlyIAS Nothing Else Visit :dpp.onlyias.in


Contact : +91-7007 931 912

was constituted by the Government in April, Exp:


1969. The tribunal was tasked to look after the ● Statement 1 is incorrect: Indian
dispute over Godavari river between Andhra Meteorological Department (IMD) is a
Pradesh, Madhya Pradesh, Chhattisgarh, government agency that deals with related to
Odisha, and Karnataka over the sharing of the meteorology and other associated fields of
Godavari river water. The Bachawat Tribunal study. The branch of science dealing with the
gave its final award in 1980. Accordingly, each atmosphere and its various phenomena, along
State was free to utilize the flow in Godavari with weather and weather forecasting can be
and its tributaries up to a certain level. Thus, defined as Meteorology. Other subjects
Andhra Pradesh decided to divert 80 tmcft of related to meteorology are Seismology,
Godavari water from Polavaram to Krishna Geography, Geology, etc. Formed in 1875, the
river, upstream of Vijayawada, so that it could India Meteorological Department (IMD) is the
be shared with Karnataka and Maharashtra. national meteorological service of the country
Once Telangana came into existence in 2014, and it is the chief government agency dealing
the Godavari water and, more specifically, the in everything related to meteorology,
Polavaram project became the bone of seismology, and associated subjects. The
contention between Telangana and Andhra administrative responsibilities of the
Pradesh. While the project will take care of the Department are under the supervision of the
irrigation needs of the Godavari districts of Ministry of Earth Sciences of the Indian
Andhra Pradesh, Telangana fears it would Government. The IMD is headquartered in
submerge many villages in its Khammam New Delhi.

e
district. Odisha too has expressed its ● Statement 2 is correct: IMD said that Climate

n
reservations over the Polavaram dam's design. Change has hampered the ability of forecasting

li
● Statement 3 is correct: Apart from Ganga and agencies to make predictions accurately.
Yamuna, Godavari also holds special religious Tools used for forecasting:

n
importance in India. Godavari is considered as 1. Radars (The number of radars will
o
one of the sacred rivers in India. The State increase from 34 at present to 67 by
l.
Government of Telangana on 2 February 2014 2025): Radars are preferred because
a

announced its decision to organize Godavari they have a higher resolution and can
ri

Pushkaralu on a large scale on the lines of provide observations every 10


Kumbh Mela. It is believed that bathing in the minutes.
te

sacred river during Pushkaram cleanses the 2. Automatic weather stations and rain
devotees of their sins. Godavari is one of the gauges and satellites.
a

12 sacred rivers, and the Godavari Pushkaram 3. The Ministry of Earth Sciences (MoES)
m

takes place once every 12 years, when Jupiter also plans to upgrade its high-
c

is in the Leo sign. The Maha Pushkaram ("Great performance computing system —
Pushkaram") takes place once every 144 years. from a capacity of 10 petaflops
s

currently to 30 petaflops in the next


p

Reference: two years.


.u

https://indianexpress.com/article/india/flood-surge- IMD uses the Long Period Average (LPA) as a


continues-across-godavari-krishna-rivers-in-ap- base to predict the expected amount of
w

8086848/ Monsoon rainfall in a particular year. The IMD


w

predicts a “normal”, “below normal”, or


“above normal” monsoon in relation to a
w

Q.77) Ans: B benchmark “long period average” (LPA).

DPP 2023 DAY 167 54


https://upscmaterial.online/
Download From - https://upscmaterial.online/

Contact us :info@onlyias.com

OnlyIAS Nothing Else Visit :dpp.onlyias.in


Contact : +91-7007 931 912

According to the IMD, the “LPA of rainfall is the are responsible for 40% and 15% of
rainfall recorded over a particular region for a polar amplification respectively.
given interval (like month or season) average 4. Some additional factors include
increases in radiation from
over a long period like 30 years, 50 years, etc”.
greenhouse gasses, atmospheric
Along with the countrywide figure, the IMD water vapor, and cloud cover as well as
also maintains LPAs for every meteorological changes in ocean heat content and
region of the country. atmospheric circulation patterns.
● Statement 3 is incorrect: According to the
Reference: World Meteorological Organization’s (WMO)
https://economictimes.indiatimes.com/news/india/cli report, ‘State of Global Climate in 2021’, sea
level along the Indian coast is rising faster
mate-change-making-prediction-difficult-for-weather-
than the global average rate. One of the
agencies-across-world-imd- primary reasons for this rise is the melting of
dg/articleshow/93406401.cms sea ice in the polar regions, especially the
Arctic. A group of Indian and Norwegian
Q.78) Ans: A scientists published a study titled ‘A possible
Exp: relation between Arctic Sea ice and late season
● Statement 1 is correct: Polar amplification Indian Summer Monsoon Rainfall extremes.’
happens when changes to the earth’s The study found that the reduced sea ice in the
atmosphere lead to a larger difference in Barents-Kara Sea region can lead to extreme
temperature near the north and south poles rainfall events.
than to the rest of the world. This
phenomenon is measured against the average Reference: https://www.thehindu.com/specials/text-

e
temperature change of the planet. These and-context/explained-what-is-causing-arctic-
warming-should-india-be-

n
changes are more pronounced at the northern
latitudes and are known as the Arctic worried/article65778586.ece

li
amplification. It occurs when the

n
atmosphere’s net radiation balance is
affected by an increase in greenhouse gasses. Q.79) Ans: B o
The Arctic is warming 2-3 times as fast as the Exp:
l.
global rate due to the unique features in the ● Option B is correct: ONGC started drilling its
a

Arctic climate system. first well at Puga (Ladakh) to generate


ri

● Statement 2 is correct: Main factors electricity using geothermal power to reduce


contributing to Arctic Amplification are as the dependence of Ladakh on diesel-run
te

following: generators. It will be India’s first geothermal


1. Sea ice loss is the dominant driver: Sea energy project and also the world’s highest at
a

ice and snow have high albedo 14,000ft. ONGC has started drilling its first well
m

implying that they are capable of for the project and encountered high-pressure
reflecting most of the solar radiation steam at 100 degrees Celsius with a discharge
c

as opposed to water and land. As the rate of 100 tonne geothermal energy per hour,
sea ice melts,resulting in the low considered as a good sign for the project. It will
s

albedo, the Arctic Ocean will be more boost Ladakh’s potential to emerge as one of
p

capable of absorbing solar radiation, the country’s clean energy bowl by expanding
.u

thereby driving the amplification. the area’s horizon beyond solar or wind power.
2. The lapse rate or the rate at which the The pilot plant provides power and heating
w

temperature drops with elevation, needs of the nearby settlements of Tibetan


decreases with warming. pastoralist refugee settlements at Sumdo and
w

3. Studies show that the ice-albedo nearby areas. A bigger plant will provide 24X7
feedback and the lapse rate feedback supply for the far-flung settlements and the
w

large defense establishment in the eastern

DPP 2023 DAY 167 55


https://upscmaterial.online/
Download From - https://upscmaterial.online/

Contact us :info@onlyias.com

OnlyIAS Nothing Else Visit :dpp.onlyias.in


Contact : +91-7007 931 912

sector, reducing their dependence on diesel deers and wild sheep and caused salt storms
for running generators. The plant can also play that pollute nearby cities and farms. It has
a vital role as a stabilizer for the 15-gigawatt started stabilizing in recent times after the
solar/wind project being planned in the nearby implementation of a joint program between
Morey plains in the southwest. Iran and the UNDP.
Puga Valley is situated in the Changthang ● Lake Superior, most northwesterly and largest
Valley in the south-eastern part of Ladakh, of the five Great Lakes of North America and
about 22 km away from the Salt Lake Valley. It one of the world's largest bodies of fresh
is a region of great significance known for its water.
natural beauty and geothermal activities. Puga ● Situated in south-east Siberia, the 3.15-
is also visited for its hot sulphur spring. million-ha Lake Baikal is the oldest (25 million
Geothermal Power refers to the heat stored years) and deepest (1,700 m) lake in the
beneath the earth’s surface that is carried to world.
the surface by water and/or steam. It can be
directly used or converted into electricity. Reference:
Other places with potential geothermal energy https://www.ncbi.nlm.nih.gov/pmc/articles/PMC6556
are Manikaran in Himachal Pradesh, Jalgaon in 201/
Maharashtra, Tapovan in Uttarakhand,
Bakreshwar in West Bengal and Tuwa in
Gujarat. Q.81) Ans: B
Exp:
Reference: ● Pair 1 is correctly matched: The Narmada is a
https://timesofindia.indiatimes.com/business/india- river located in central India. It rises to the
business/ongc-readies-to-pump-geothermal-power- summit of the Amarkantak Hill in Madhya

e
at-14000-feet-in-ladakh/articleshow/93695731.cms Pradesh state. It outlines the traditional

n
frontier between North India and South India.
It is one of the major rivers of peninsular India.

li
Q.80) Ans: A Only the Narmada, the Tapti, and the Mahi

n
Exp: rivers run from east to west. The river flows
● Option A is correct: This year, for the first time o
through the states of Madhya Pradesh,
in its centuries-long history, Iraq Lake Sawa Gujarat, and Maharashtra. It drains into the
l.
dried up. A combination of mismanagement by Arabian Sea in the Bharuch district of Gujarat.
a

local investors, government neglect and ● Pair 2 is incorrectly matched: Chambal river is
ri

climate change has ground down its azure 960 km. long river that originates at the Singar
shores to chunks of salt. The Sawa Lake, a Chouri peak in the northern slopes of the
te

biodiverse wetland situated near the city of Vindhya mountains (Indore, Madhya Pradesh).
Samawa, south of the capital Baghdad, Iraq. From there, it flows in the North direction in
a

● Lake Urmia is a saltwater lake. It is situated in Madhya Pradesh for a length of about 346 km
and then follows a north-easterly direction for
m

the mountains of northwest Iran i.e the west of


the southern portion of the Caspian Sea and is a length of 225 km through Rajasthan. It enters
U.P. and flows for about 32 km before joining
c

fed by 13 rivers. It is designated as a site of


international importance under the UN the Yamuna River in Etawah District. It is a
s

Convention on Wetlands. The lake has been rainfed river and its basin is bounded by the
p

shrinking since 1995, due to a combination of Vindhyan mountain ranges and the Aravallis.
.u

prolonged drought, elevated summer The Chambal and its tributaries drain the
temperatures that speed up evaporation, over- Malwa region of northwestern Madhya
w

farming and dams. It became one of the worst Pradesh.


ecological disasters of recent decades as the ● Pair 3 is correctly matched: The Yamuna River
w

lake's surface, which was 2,366 km2 in 2011 is the largest tributary of the Ganga River. It
shrank to just 700 km2 in 2013. This has originates from the Yamunotri glacier, at the
w

threatened the habitat of shrimp, flamingos, Bandarpoonch peak in Uttarakhand in the

DPP 2023 DAY 167 56


https://upscmaterial.online/
Download From - https://upscmaterial.online/

Contact us :info@onlyias.com

OnlyIAS Nothing Else Visit :dpp.onlyias.in


Contact : +91-7007 931 912

Shivalik Ranges. The main tributaries joining the Indian Ocean Rim Association (IORA) that
the river include the Sin, Hindon, Betwa Ken, India is reviving. It is also a member of the Arab
and Chambal. The Tons is the largest tributary League and the Organization of Islamic
of the Yamuna. The catchment of the river Cooperation (OIC).
extends to the states of Delhi, Himachal ● Statement 3 is correct: In the colonial era, the
Pradesh, Uttar Pradesh, Haryana, Rajasthan, Vanilla Islands were the object of rivalry
and Madhya Pradesh. among the European powers. All the sea lines
● Pair 4 is incorrectly matched: The Mahanadi of communication between Europe and the
originates from the Satpura Range of central Indian Ocean came round Africa and went
India and it is a river in eastern India. It flows through the Mozambique channel – the Vanilla
east to the Bay of Bengal. The river drains of islands became attractive way stations. The
the state of Maharashtra, Chhattisgarh, Comoros was of special importance as it sits at
Jharkhand, and Orissa.The largest dam, the the northern end of the Mozambique channel
Hirakud Dam is built on the river. and provided a strong base to control the
channel.
Reference- France gained the upper hand among the
https://www.nrsc.gov.in/KR_Atlas_RiverBasin?langua European powers in the Vanilla Islands in the
ge_content_entity=en 18th century. The construction of the Suez
Canal obviated the need for European shipping
to go round Africa. This reduced the strategic
Q.82) Ans: C significance of the Vanilla islands.
Exp:
● Statement 1 is incorrect: Vanilla Islands are Reference: https://www.chronicleindia.in/current-
formed by the islands of Seychelles, affairs/609-lsquo-vanilla-islands-rsquo

e
Madagascar, Reunion (France), Mauritius,
Q.83) Ans: D

n
Comoros, and Mayotte (France) in the Indian
Ocean. They joined hands to promote tourism. Exp:

li
Together they account for more than 40 ● Statement 1 is correct: Electromagnetic fields

n
members. Their large numbers and impact on are a combination of invisible electric and
the voting patterns in the UN and other o
magnetic fields of force. They are present
multilateral forums had made island states of everywhere in our environment but are
l.
interest to major powers. Island states have invisible to the human eye. Electric fields are
a

also taken the lead in developing the concept produced by the local build-up of electric
ri

of “blue economy” focused on sustainable use charges in the atmosphere associated with
and development of ocean resources. thunderstorms. The electricity that comes out
te

● Statement 2 is correct: The Vice President of of every power socket is associated with low
India is the first senior figure from the Indian frequency electromagnetic fields.
a

leadership to visit the Comoros. India is ● Statement 2 is correct: The International EMF
Project of the World Health Organisation
m

devoting greater attention to the Indian


Ocean. Many places that have long fallen off (WHO) published an information sheet in 2005
on the effect of EMF emissions on animals,
c

Delhi’s political radar are coming into view.


India is witnessing a growing interaction with insects, vegetation, and aquatic life and has
s

the island states. The Prime Minister met concluded that the exposure limits in the
p

leaders of the Caribbean Community International Commission on Non-Ionizing


.u

(CARICOM) and the Pacific Islands Forum (PIF) Radiation Protection (ICNIRP) guidelines for
on the margins of the UNGA last month. India’s the protection of human health are also
w

activism on countering climate change and protective of the environment. The present
promoting the blue economy have made the norms for Electromagnetic Field (EMF)
w

island states special partners for India. emissions from mobile towers in India are
Comoros is a founding member of the already ten times more stringent (even lower)
w

International Solar Alliance. It is a member of

DPP 2023 DAY 167 57


https://upscmaterial.online/
Download From - https://upscmaterial.online/

Contact us :info@onlyias.com

OnlyIAS Nothing Else Visit :dpp.onlyias.in


Contact : +91-7007 931 912

than the safe limits prescribed by ICNIRP and tropical forest. The volcano, which has the
recommended by WHO. same name as the island, is the sub-aerial part
● Statement 3 is correct: Impacts on Humans: of a large frequently active stratovolcano
Numerous worldwide studies link EMFs to which nowadays remains active and still ejects
serious health problems such as leukemia, magma and releases large smoke plumes full of
miscarriages, chronic fatigue, weakened sulfur.
immune system, forgetfulness, depression, ● Pair 3 is incorrectly matched: Etna is located
nausea and loss of libido. on the Italian island of Sicily. It is locally called
Impacts on the Environment: Radars are used "Mongibello", is Europe's largest and most
for navigation, weather forecasting emit active volcano. Its frequent eruptions are
pulsed microwave signals, which are often accompanied by large lava flows, but
detrimental to health of flora and fauna rarely pose danger to inhabited areas. Etna is
present around these radars. one of the volcanoes with the longest historic
records of eruptions, going back more than
2000 years. Historical lava flows cover much of
Reference: the surface of this massive volcano, which
https://pib.gov.in/PressReleaseIframePage.aspx?PRID makes it the highest and most voluminous in
=1848739 Italy. Ever since ancient times, the volcano
https://www.who.int/news-room/questions-and- seems to have been in near-constant activity.
answers/item/radiation-electromagnetic-fields It is considered, after Kilauea in Hawaii, the
second most active, i.e. productive volcano on
earth.
Q.84) Ans: A ● Pair 4 is incorrectly matched: Mount Semeru
Exp: is the tallest and most active volcano on the

e
● Pair 1 is correctly matched: Kavachi is an Indonesian island of Java. It is one of the most
active volcanoes in Indonesia. Typically

n
active submarine volcano in the South-West
Pacific, located in the Solomon Islands south eruptions occur every 20-30 minutes, with ash

li
of Gatokae and Vangunu islands. Since its first emissions to 100-400 m above the crater, and

n
recorded eruption in 1939, Kavachi has created projectiles ejected onto the crater rim. The
ephemeral islands on several occasions. But present activity center is at Jonggring Saloko, a
o
the islands, up to a kilometer long, have been crater that appears in the southeast part of
l.
eroded and washed away by wave action. Mahameru, and it is separated by a narrow
a

Kavachi formed in a tectonically active area—a saddle. Large eruptions are often followed by
generation of pyroclastic flows. Semeru
ri

subduction zone lies 30 kilometers (18 miles)


to the southwest. The volcano produces lavas volcano is the highest in Java and one of the
te

that range from basaltic, which is rich in most active volcanoes in the world.
magnesium and iron, to andesitic, which
a

contains more silica. It is known for having Reference:


phreatomagmatic eruptions in which the https://volcano.si.edu/gvp_currenteruptions.cfm
m

interaction of magma and water cause


explosive eruptions that eject steam, ash,
c

volcanic rock fragments, and incandescent Q.85) Ans: A


s

bombs. Exp:
p

● Pair 2 is incorrectly matched: Tofua is a ● Statement 1 is incorrect: Recently, Indian


.u

remote volcano in the central part of the Tonga Meteorological Department (IMD) has
Islands group and it is part of the Ha’apai island released the Aridity Anomaly Outlook (AAO)
w

group, within the archipelago of the Kingdom Index of July 2022, which says at least 85% of
of Tonga in the southern Pacific Ocean. Having districts face arid conditions across India. The
w

an oval shape, the land area of Tofua has 64 Aridity Index (AI) is computed for weekly or
square km (25 square mi), where around sixty two-week periods. For each period, the actual
w

percent of the island is covered by semi- aridity for the period is compared to the

DPP 2023 DAY 167 58


https://upscmaterial.online/
Download From - https://upscmaterial.online/

Contact us :info@onlyias.com

OnlyIAS Nothing Else Visit :dpp.onlyias.in


Contact : +91-7007 931 912

normal aridity for that period. Negative values ● Statement 1 is incorrect: It is a primitive and
indicate a surplus of moisture while positive hazardous method of mining for coal. It
values indicate moisture stress. involves digging of very small narrow tunnels
● Statement 2 is correct: The index monitors (hence called rat holes), usually 3-4 feet high,
agricultural drought, a situation when rainfall which workers (more often children)enter and
and soil moisture are inadequate to support extract coal. The rat-hole mining is broadly of
healthy crop growth till maturity, causing crop two types - side-cutting and box-cutting. In a
stress. An anomaly from the normal value side-cutting procedure, narrow tunnels are
signifies a water shortage in these districts that dug on the hill slopes and workers go inside
could directly impact agricultural activity. It is a until they find the coal seam. The coal seam in
real-time drought index in which water the hills of Meghalaya is very thin, less than 2
balance is considered. It assesses the impacts m in most cases. In the box-cutting type, a
of drought in agriculture, especially in the rectangular opening is made, varying from 10
tropics where defined wet and dry seasons are to 100 sq m. Through this, a vertical pit, 100 to
part of the climate regime. Both winter and 400 feet deep, is dug. Once the coal seam is
summer cropping seasons can be assessed found, rat-hole-sized tunnels are dug
using this method. horizontally through which workers can extract
● Statement 3 is correct: The following are the the coal. The Mines and Minerals
key findings of the index: (Development and Regulation) Amendment
1. Only 63 of 756 districts are non-arid, Bill, 2021 was passed in both Houses of
while 660 are facing different degrees Parliament. The Bill amends the Mines and
of aridity — mild, moderate and Minerals (Development and Regulation) Act,
severe. 1957. The Act regulates the mining sector in
2. Some 196 districts are in the grip of a India.

e
‘severe’ degree of dryness and 65 of ● Statement 2 is correct: Rat-hole mining
these are in Uttar Pradesh (highest). degrades water quality with a high

n
Bihar had the second highest number concentration of sulfates, iron and toxic heavy

li
of districts (33) experiencing arid metals, low dissolved oxygen (DO), and high

n
conditions. The state also has a high BOD. Areas around mines are used for piling of
rainfall deficit of 45%. coal, which has become a major source of air,
o
3. Other districts facing ‘severe arid’ water, and soil pollution. Off-road movement
l.
conditions are in Jharkhand, Haryana, of coal trucks and other vehicles in the mining
a

Madhya Pradesh, Delhi, Telangana, area causes further damage to the ecology of
Maharashtra, Andhra Pradesh, Jammu the area. The National Green Tribunal (NGT)
ri

and Kashmir, Punjab, West Bengal, banned it in 2014.


te

Rajasthan, Karnataka and Tamil Nadu. ● Statement 3 is correct: It is practiced mostly in


4. Arid conditions have impacted the Northeastern states, especially in Meghalaya
a

ongoing kharif sowing, as the area because coal seam found in the hilly region is
sown under different kharif crops as of very thin, less than 2 m in most cases. In
m

July, 2022, was less by 13.26 million Jharkhand, the coal layer is extremely thick,
hectares compared to the where open-cast mining can be done. But no
c

corresponding period in 2021. other method would be economically viable in


s

Meghalaya, where the coal seam is extremely


p

Reference: thin. Removal of rocks from the hilly terrain


https://www.imdpune.gov.in/hydrology/Drought/met and putting up pillars inside the mine to
.u

hodology.html prevent collapse would be costlier. So despite


a ban, rat-hole mining remains the prevalent
w

procedure for coal mining in Meghalaya. Rat-


w

Q.86) Ans: B hole mining is the locally developed technique


Exp: and the most commonly used one. It is not
w

regulated by any law, and coal extraction has


been made by unscrupulous elements in a
DPP 2023 DAY 167 59
https://upscmaterial.online/
Download From - https://upscmaterial.online/

Contact us :info@onlyias.com

OnlyIAS Nothing Else Visit :dpp.onlyias.in


Contact : +91-7007 931 912

most illegal and unscientific manner. utilized only 14 percent of its funds
Meghalaya’s annual coal production of nearly because of the lack of coordination
6 million tonnes is mostly said to have come with the ASI.
through rat-hole mining. ○ Violations of Various Provisions: The
report reveals that the ASI has been
Reference: https://prsindia.org/billtrack/the-mines- working in violation of the provisions
and-minerals-development-and-regulation- of the Monuments (AMASR) Act. At
amendment-bill-2021 Humayun’s tomb, a CAG inspection
revealed commercial construction
being undertaken in the prohibited
Q.87) Ans: B area by an “agency other than the
Exp: ASI”.
● Statement 1 is incorrect: Comptroller and ○ Dumping of garbage and sewage: The
Auditor General’s performance audit on auditing team’s visit revealed that
Preservation and Conservation of Monuments sewage from nearby areas“was being
and Antiquities that was tabled in Parliament discharged into many lakes”.
singled out the abysmal (poor) state of
heritage conservation across India. The CAG Reference:
reports on the state of India’s heritage https://indianexpress.com/article/opinion/columns/n
released over the years related with different ayanjot-lahiri-writes-cag-report-abysmal-state-
issues such as CAG report on museums (2011), heritage-conservation-anang-tal-mehrauli-8123799/
on findings of ASI (2013) are the most
comprehensive public documents. These
reports along with reports of Parliamentary Q.88) Ans: A

e
Committees are the only sources today for Exp:

n
understanding the state of Indian archaeology, ● Option A is correct: 9th August is celebrated as
monuments and museums. Therefore these the International Day of the World’s

li
documents are essential because after 2007, Indigenous Peoples. It aims to highlight the

n
there has been no internal audit of the ASI role of indigenous people and the importance
conducted by the Cultural ministry. o
of preserving their rights, communities and
● Statement 2 and statement 3 are correct: knowledge they gathered and passed down
l.
Issues Highlighted in the report are as over centuries. This year’s Theme: “The Role of
a

following: Indigenous Women in the Preservation and


ri

○ Lack of proper National Policy: There Transmission of Traditional Knowledge.” In


is no national policy on archaeological 1994, the UNGA, passed a resolution, declaring
te

exploration and excavation which August 9 as the International Day of the


seems to be true for antiquities as World’s Indigenous People as it was on 9th
a

well. August that the UN Working Group on


m

○ Less allocation of Funds: The ASI Indigenous Populations held its first meeting.
budget for exploration and Taking cognisance of the knowledge acquired
c

excavations is less than 1 per cent even by indigenous people is vital culturally and also
though it intended to raise the scientifically. Understanding and preservation
s

allocation to 5 percent of the total of indigenous languages, their spiritual


p

budget. practices, and philosophies can help in the


.u

○ Lack of Coordination: There is no conservation and upliftment of Tribals without


synergy between the different compromising their identity. Status of Tribes in
w

agencies involved in heritage India: Tribal Population constitutes 8.6% (or 11


conservation. The National Culture cr) of the total population (the second largest
w

Fund, established in November 1996, number of tribal people in any country in the
to rope in individuals and corporate world). 89.97% of them live in rural areas and
w

groups to fund conservation, has 10.03% in urban areas. According to Lokur

DPP 2023 DAY 167 60


https://upscmaterial.online/
Download From - https://upscmaterial.online/

Contact us :info@onlyias.com

OnlyIAS Nothing Else Visit :dpp.onlyias.in


Contact : +91-7007 931 912

Committee (1965), the essential chieftains, known as Wangham or Wangsa.


characteristics to be recognized by Scheduled Like most neighboring tribes, the Wancho
Tribe are: Indication of Primitive Traits, constructed houses made out of wood and
Distinctive Culture, Shyness of Contact with bamboo, and roofs were thatched with dry
the Community at Large, Geographical leaves. Dormitories, known as Murung, where
Isolation, Backwardness Constitution: the boys are trained to be men by their fathers.
Constitution of India does not define the term Although the girls do not have dormitories like
‘tribe’, however, the term Scheduled Tribe’ the boys, they sleep in one big, single house,
was inserted in the Constitution through with the care of an old lady. Until 1991, human
Article 342 (i). headhunting was practiced among the Naga
tribes, and both the government and
Reference: missionaries have taken steps to ban the
https://www.news18.com/news/lifestyle/internation practice of headhunting, which is now
al-day-of-the-worlds-indigenous-peoples-2022- restricted to animals.
history-theme-and-significance-5702131.html
Reference:
https://www.outlookindia.com/national/arunachal-s-
Q.89) Ans: B wancho-community-sets-out-to-digitise-ancient-
Exp: folktales-news-217865
● Statement 1 is incorrect: The Wancho are a https://longding.nic.in/tribe/
tribal people inhabiting the Patkai hills of
Longding District in the state of Arunachal
Pradesh, India. They have a population of
35,000. Culturally Naga, they are ethnically Q.90) Ans: C

e
related to the Nocte and Konyak Naga of the Exp:

n
Mon and Tirap districts. The Wancho language ● Both Statements 1 and 2 are correct: Navroz
belongs to the Tibeto-Burman family. Festival is celebrated at the time of the vernal

li
● Statement 2 is correct: Tattooing plays a major equinox (marking the start of spring) in the

n
role among the Wancho tribe. According to Northern Hemisphere. It is being celebrated in
tradition, a man is tattooed on his four limbs o
India on 16th August in India. ‘Nav’= new and
and his entire face, with the exception of ‘Roz’= day, meaning ‘new day’ (in the Persian
l.
certain regions around the eyes and the lips. language). It is celebrated in March globally (as
a

The women adorn themselves with necklaces per the Iranian calendar), Navroz arrives 200
ri

and bangles, along with some light tattooing as days later in India and is celebrated in the
well. The prime festival of the Wancho is month of August as the Parsis here follow the
te

Oriah, a festival between March to April, for a Shahenshahi calendar which doesn’t account
period of six to twelve days interspersed with for leap years. The Persian king Jamshed is
a

prayer, songs and dances. Villagers exchange credited with having created the Shahenshahi
m

bamboo tubes filled with rice beer as a mark of calendar. Navroj is on the list of UNESCO
greeting and goodwill. Pork skin is then offered Intangible Cultural Heritage of Humanity of
c

to the village chief as a mark of respect. This India.


festival continues for several days just sowing
s

of Jhum paddy and Pigs, buffaloes and mithuns Reference:


p

are sacrificed and feasts are arranged in each https://economictimes.indiatimes.com/magazines/pa


.u

and every morungs (dormitories). Boys and nache/happy-parsi-new-year-2022-from-food-to-gifts-


girls, wearing ceremonial costumes, sing and a-look-at-the-origin-of-
w

dance during this Oriah. People dance around navroz/articleshow/93584057.cms


a “Jangban”, a long ceremonial pole planted
w

during Oriah.
● Statement 3 is correct: The Wancho are Q.91) Ans: D
w

traditionally governed by a council of elderly Exp:

DPP 2023 DAY 167 61


https://upscmaterial.online/
Download From - https://upscmaterial.online/

Contact us :info@onlyias.com

OnlyIAS Nothing Else Visit :dpp.onlyias.in


Contact : +91-7007 931 912

● Option D is correct: The Temple of the Vedic for the year’s rice harvest. This is one of India’s
Planetarium in Mayapur, West Bengal, is a most colorful harvest festivities, lasting four
temple currently under construction. It is days. The first day is the Bhogi Festival, which
slated to be ready by 2023 and when it does, it is dedicated to Lord Indra in exchange for an
will be the world’s largest Hindu temple and abundance of rain. On the second day, newly
also one of the tallest in the world. The Temple harvested rice and milk are prepared and
of the Vedic Planetarium will be the presented to the Sun God outside. The third
headquarters of the International Society of day is dedicated to cow worship. The fourth
Krishna Consciousness (ISKCON). It is located day is dedicated to Pongal, or traditional
approximately 130 km from Kolkata. The coloured rice flavored with turmeric, betel leaf,
purpose behind the construction of this temple and betel nuts. Tamilians mark the event by
is to make people aware of the Vedic culture. creating traditional patterns known as kolams
The temple is based on the vision of Acharya using rice powder in their houses. Pongal
Prabhupada, the founder of ISKCON, which celebration highlights also include bull-taming
would help in the spread of Vedic knowledge contests, a bonfire made of agricultural waste
in the world. Special Blue Bolivian Marble has and prayers for the prosperity of the family.
been used in the 380 feet high temple, which ● Pair 3 is incorrectly matched: Nabanna: This
shows the influence of Western architecture in harvest festival is celebrated in West Bengal.
the temple. Mayapur is also known as the Land This is one of Bengal’s most recognised
of the Golden Avatar, this site is about 140 km celebrations, in which fresh rice is harvested
from Calcutta. An incarnation of Lord Krishna with glee and stored in households. Farmers
(incarnation of Lord Vishnu), Sri Chaitanya from Bengal joyfully engage in this harvest rite
Mahaprabhu was born in this city in the year during the Bengali month of Agrahayan,
1486. offering the first grains to Goddess Lakshmi

e
and thanking her for all benefits. The main
Reference: attractions of the Nabanna celebration are

n
https://www.hindustantimes.com/cities/kolkata- Nabanna fair and Payesh (Kheer) cooked from

li
news/vedic-planetarium-world-s-largest-temple-to- freshly harvested rice.

n
open-in-bengal-10-things-to-know- ● Statement 4 is correctly matched: Onam is a
101661579886577.html historical harvest festival in Kerala that is
o
celebrated with tremendous zeal in various
l.
regions of the state. With the advent of
a

Q.92) Ans: B Mahabali, the festival lasts ten days. People


celebrate the successful harvest by decorating
ri

Exp:
● Pair 1 is incorrectly matched: Nuakhai is an their house entry with flower rangoli, wearing
te

ancient harvest festival celebrated in Orissa. new traditional costumes, cooking wonderful
Nuakhai Parab or Nuakhai Bhetghat are other food, and celebrating with traditional music
a

names for the harvest festival. ‘Nua’ means and dancing. Traditional Malayalee foods such
as Rasam, Payasam, Avial, brown rice, and
m

new in the native language, while ‘Khai’


indicates food. This is not only a famous parippu curry are served to guests in
traditional green leaf, and the snake boat race
c

harvest festival but it is also held to honor the


passage of the old and wicked days while and tiger dance are extremely enjoyable to
s

embracing the new and lovely with open arms. witness.


p

The delectable Arsaa Pitha (sweet pancakes)


.u

are prepared to celebrate the festival.


● Pair 2 is correctly matched: Pongal is another Q.93) Ans: D
w

name for Makar Sankranti, which is celebrated Exp:


in many cities in Tamil Nadu at the same time. ● Statement 1 is correct: Mandala literally
w

Pongal means ‘overflow‘ or ‘boiling over.’ This means “circle” or “center” in Sanskrit. It is
is a thanksgiving festival in which people show defined by a geometric configuration that
w

their heartfelt appreciation to Mother Nature usually incorporates the circular shape in some

DPP 2023 DAY 167 62


https://upscmaterial.online/
Download From - https://upscmaterial.online/

Contact us :info@onlyias.com

OnlyIAS Nothing Else Visit :dpp.onlyias.in


Contact : +91-7007 931 912

form. While it can also be created in the shape supports unique and diverse aquatic
of a square, a mandala pattern is essentially ecosystems and serves as a vital habitat for
interconnected. It is believed that by entering some of the world's richest coral reefs. The
the mandala and moving towards its center,
Gulf houses more than 100 coral species, 800
one is guided through the cosmic process of
transforming the universe from one of fish species as well as several species of
suffering to that of joy. crustaceans and mollusks.
● Statement 2 is correct: It is believed to be
rooted in Buddhism, appearing in the 1st
century BC in India. Over the next couple
centuries, Buddhist missionaries traveling
along the silk-road took it to other regions. By
the 6th century, mandalas have been recorded
in China, Korea, Japan, Indonesia and Tibet. In
Hinduism, the mandala imagery first
appeared in Rig Veda (1500 – 500 BCE). In Reference:
Hinduism, a mandala or yantra is in the shape https://economictimes.indiatimes.com/news/india/eg
of a square with a circle at its center.
ypt-releases-postage-stamp-celebrating-75th-
● Statement 3 is correct: Mandala patterns have
been adapted by artists the world over, each of anniversary-of-diplomatic-ties-with-
whom have added their own interpretation india/articleshow/93669503.cms
and painted it as their own. Recently, a
mandala, the size of one and a half football
pitches in length was created by artist James Q.95) Ans: B

e
Brunt, in Liverpool, UK with materials such as Exp:
leaves and rocks.

n
● Pair 1 is correctly matched: Odisha’s
Kandhamal Haldi (turmeric), famous for its

li
Reference: healing properties, has received GI Tag. The

n
https://indianexpress.com/article/explained/explaine golden yellow spice, named after the district
d-culture/mandala-art-history-buddhism-james-brunt- o
where it is produced, has been cultivated since
8101223/ time immemorial and is known for its
l.
medicinal value. Turmeric is the main cash crop
a

of tribal people in Kandhamal. Apart from


ri

domestic use, turmeric is also used for


Q.94) Ans: A cosmetic and medicinal purposes. More than
te

Exp: 60,000 families (nearly 50% of Kandhamal


● The Gulf of Aqaba or Gulf of Eilat is a large gulf population) are engaged in growing the
a

variety. The crop is sustainable in adverse


at the northern tip of the Red Sea, east of the
climatic conditions.
m

Sinai Peninsula and west of the Arabian


● Pair 2 is incorrectly matched: The government
Peninsula. The Gulf of Aqaba connects the red has recently awarded the Geographical
c

sea with Israel, the Gulf of Suez connects the Indication (GI) tag to Mithila Makhana of
s

red sea with the Mediterranean sea. Four Bihar. Mithila Makhana or Makhan (botanical
p

countries share its northern section: Egypt name: Euryale ferox Salisb.) is a special variety
.u

controls the western shore; Saudi Arabia the of aquatic fox nut cultivated in Mithila region
of Bihar and Nepal. Makhana is the one of the
eastern shore; Jordan possesses a 25-
w

three prestigious cultural identities of Mithila.


kilometre corridor along the Gulf in which is It is also very famous in the Kojagara festival of
located the city of Aqaba (the country's only
w

Maithil Brahmins celebrated for newly married


maritime port); Israel has a 10-kilometre strip couples. Makhana contains protein and fiber,
w

that includes the city of Eilat. The Gulf of Aqaba

DPP 2023 DAY 167 63


https://upscmaterial.online/
Download From - https://upscmaterial.online/

Contact us :info@onlyias.com

OnlyIAS Nothing Else Visit :dpp.onlyias.in


Contact : +91-7007 931 912

along with micronutrients like calcium, Representation of the People Act (RPA), 1951 (43
magnesium, iron, and phosphorus. of 1951) and which secured not less than one
● Pair 3 is correctly matched: The ‘Gulbarga tur percent of the votes polled in the last General
dal’ (Kalaburagi red gram) of Karnataka,
Election to the House of the People or the
received a Geographical Indication (GI) tag
from the government. Red gram is the main Legislative Assembly of the State, shall be
Kharif crop in the Hyderabad Karnataka region. eligible to receive the Electoral Bonds. The RPA
The high calcium and potassium content of the (Representation of People Act 1951) although
soil in the Gulbarga tur dal-growing region has makes it mandatory for the political parties to
been one of the major factors for its superior disclose donations over Rs 20000.
quality. Gulbarga tur dal has a good taste and ● Statement 3 is incorrect: A person being an
aroma compared to that grown elsewhere. It
individual can buy Electoral Bonds, either singly
also takes less time to cook.
● Pair 4 is correctly matched: Odisha’s Rasagola or jointly with other individuals.Donor’s name is
has received GI Tag. It is a sweet made of not mentioned on the bond. The bonds are like
chhena (cottage cheese) cooked in sugar syrup banknotes that are payable to the bearer on
using the principle of caramelisation of sugar demand and are interest-free.State Bank of
(caramelization is the browning of sugar, a India is authorised to issue and encash these
process used extensively in cooking for the
bonds, which are valid for fifteen days from the
resulting sweet nutty flavor and brown color).
date of issuance.One can purchase these bonds
It is associated with the world famous Puri
Jagannath Temple. only digitally or through cheques.The Electoral
Bonds can be encashed by an eligible Political
Reference- https://www.thehindu.com/life-and- Party only through a Bank account with the

e
style/food/the-odisha-rasgolla-receives-the-gi- Authorized Bank.The Electoral Bond deposited
tag/article28757036.ece

n
by an eligible Political Party in its account is

li
credited on the same day.
Q.96) Ans: B

n
Exp:
Reference: o
● Statement 1 is correct: Electoral Bond is a
https://indianexpress.com/article/business/banking-
l.
financial instrument for making donations to
and-finance/electoral-bonds-parties-mop-up-over-rs-
a

political parties.The bonds are issued in


10000-crore-since-2018-8062883
multiples of Rs. 1,000, Rs. 10,000, Rs. 1 lakh, Rs.
ri

10 lakh and Rs. 1 crore without any maximum


Q.97) Ans: C
te

limit.These bonds are redeemable in the


Exp:
designated account of a registered political
● Counterfeiting, manufacture of false money
a

party.The bonds are available for purchase for a


for gain, a kind of forgery in that something is
m

period of ten days each in the months of January,


copied so as to defraud by passing it for the
April, July and October as may be specified by
c

original or genuine article. counterfeit


the Central Government.An Electoral Bond is
currency leads to an increase in prices
s

like a promissory note that may be purchased


(inflation) due to more money getting
p

by a person who is a citizen of India or company


circulated in the economy. It causes an
.u

incorporated or established in India.A


artificial increase in money supply in the
promissory note is a written promise by one
economy. It leads to reduction in the value of
w

party to make a payment of money at a date in


real money and a decrease in the acceptability
the future.
w

of paper money. It results in losses for traders,


● Statement 2 is incorrect: Only the Political
as they are not reimbursed for counterfeit
w

Parties registered under Section 29A of the


money detected by banks, even if it is

DPP 2023 DAY 167 64


https://upscmaterial.online/
Download From - https://upscmaterial.online/

Contact us :info@onlyias.com

OnlyIAS Nothing Else Visit :dpp.onlyias.in


Contact : +91-7007 931 912

confiscated. Hence, It gives rise to illegal ● Statement 2 is incorrect: Anti-radiation missiles


transactions where no legal money exchange are designed to detect, track and neutralize the
happens.Other effects include the loss of adversary’s radar, communication assets and
public confidence, black marketing of products other radio frequency sources, which are
and illegal stocking of products.If the value of generally part of their air defence systems.Such
the currency is weakened through a missile’s navigation mechanism comprises an
administrative action, it is called inertial navigation system, a computerised
devaluation.The circulation of a large amount mechanism that uses changes in the object’s
of fake currency increases the amount of own position , coupled with GPS, which is
money in circulation, which may lead to high satellite-based. For guidance, it has a “passive
demand for goods and commodities. The rise homing head” , a system that can detect, classify
in demand in turn creates a scarcity of goods, and engage targets (radio frequency sources in
leading to a rise in the price of the goods. This this case) over a wide band of frequencies as
leads to currency devaluation. programmed.

Reference: Reference:
https://www.thehindu.com/news/national/parliamen https://indianexpress.com/article/explained/explaine
tary-proceedings-value-of-fake-notes-in-banking- d-sci-tech/explained-what-is-agm-88-harm-the-new-
system-recorded-over-80-decline-from-2016-17-to- anti-radar-missile-supplied-ukraine-us-8080573/
2021-22-pankaj-choudhary/article65746940.
https://www.livemint.com/news/india/seized-fake-

e
currency-worth-rs-43-47-crore-in-2016-17-now- Q.99) Ans: C

n
valued-at-rs-8-26-crore-11659947730635.html Exp:

li
● The Indian Army reportedly tested the
operational readiness of its satellite-based

n
Q.98) Ans: B systems deployed across the country under
o
Exp: operation ‘Skylight’. Satellite-based systems
l.
● Statement 1 is correct: It is an air-to-surface provide resilience to communication
a

missile, designed and developed by the DRDO. capabilities in case terrestrial connectivity is
ri

DRDO conducted a successful test of the New disrupted in conflicts. Unlike the Indian Air
Generation Anti Radiation Missile (NGRAM) also Force and the navy, the army currently does
te

called the Rudram-1 at the Integrated Test not have a dedicated satellite. Currently, the
Range (ITR) in Balasore (Odisha). Rudram-1 is Army is using 30% of the communication
a

the first indigenous anti-radiation missile of the capabilities of the GSAT 7A. The Defence
m

country. Rudram has been developed for the Acquisition Council has cleared an army
c

Indian Air Force - IAF’s requirement to enhance proposal for a GSAT-7B satellite to sharpen its
its Suppression of Enemy Air Defence (SEAD) operational capabilities. GSAT 7 series
s

capability. Recently The United States confirmed satellites are advanced satellites developed by
p

that it has supplied “anti-radiation missiles” to ISRO to meet the communication needs of the
.u

Ukraine in ongoing conflict with Russia.This defence services.Various agencies responsible


confirmation has strengthened Russian for space and ground segments, including the
w

allegations that an American anti-radar missile, Indian Space Research Organisation (ISRO)
w

AGM-88 HARM, which is part of NATO’s participated in the exercise. The Army has
inventory, has been used in the theatre of carried out detailed studies of cyber and
w

conflict. electromagnetic effects in the war in Ukraine.

DPP 2023 DAY 167 65


https://upscmaterial.online/
Download From - https://upscmaterial.online/

Contact us :info@onlyias.com

OnlyIAS Nothing Else Visit :dpp.onlyias.in


Contact : +91-7007 931 912

Electronic warfare has played a significant role contrary to the fundamental laws of demand
in Ukraine. which are based on a downward sloping
demand curve.
Reference: https://theprint.in/defence/amid-india-
china-tensions-army-conducts-exercise-skylight-to- Reference:
test-satellite-based-systems/1070655/ https://www.thehindu.com/business/Economy/cantill
https://www.thehindu.com/news/national/indian- on-effect/article65746967.ece
army-conducts-exercise-skylight-to-testresilience-of-
itssatellite-communications/article65733174.ece

Q.100) Ans: A
Exp:
● The Cantillon effect refers to the idea that
changes in the money supply in an economy
causes redistribution of purchasing power
among people, disturbs the relative prices of
goods and services, and leads to the
misallocation of scarce resources. Although,
money has largely been considered to be
neutral by economists today, in the sense that

e
changes in its supply have no real effect on the

n
economy. It is named after 18th century
French economist Richard Cantillon who

li
published his ideas in the 1755 book Essay on

n
Nature of Trade in General.It explains how o
changes in the supply of money in an economy
l.
can affect the prices of different goods in
a

different proportions. This is because the real


ri

purchasing power of individuals is not affected


uniformly by the changes in money supply. The
te

Cantillon effect has been widely cited by


economists who are critical of expansionary
a

central bank policy to tackle economic


m

downturns.
● Beveridge curve - This refers to a graphical
c

representation that shows the relationship


s

between the unemployment rate (on the


p

horizontal axis) and the job vacancy rate (on


.u

the vertical axis) in an economy. It is named


after British economist William Beveridge
w

● A Giffen good is a low-income, non-luxury


w

product for which demand increases as the


price increases and vice versa. A Giffen good
w

has an upward-sloping demand curve which is

DPP 2023 DAY 167 66


https://upscmaterial.online/

You might also like